You are on page 1of 246

Financial Reporting

(International Stream)
PART 2 THURSDAY 5 DECEMBER 2002

QUESTION PAPER Time allowed 3 hours This paper is divided into two sections Section A This ONE question is compulsory and MUST be answered THREE questions ONLY to be answered

Section B

Paper 2.5(INT)

Section A This ONE question is compulsory and MUST be attempted 1 Hydrate is a public company operating in the industrial chemical sector. In order to achieve economies of scale, it has been advised to enter into business combinations with compatible partner companies. As a first step in this strategy Hydrate acquired all of the ordinary share capital of Sulphate by way of a share exchange on 1 April 2002. Hydrate issued five of its own shares for every four shares in Sulphate. The market value of Hydrates shares on 1 April 2002 was $6 each. The share issue has not yet been recorded in Hydrates books. The summarised financial statements of both companies for the year to 30 September 2002 are: Income statement year to 30 September 2002: Hydrate $000 24,000 (16,600) 7,400 (1,600) 5,800 (2,000) 3,800 $000 64,000 nil 64,000 $000 Sulphate $000 20,000 (11,800) 8,200 (1,000) 7,200 (3,000) 4,200 $000 35,000 12,800 47,800

Sales revenue Cost of sales Gross profit Operating expenses Operating profit Taxation Profit after tax Balance Sheet as at 30 September 2002 Non-current assets $000 Property, plant and equipment Investment Current Assets Inventory Accounts receivable Bank Total assets Equity and liabilities Ordinary shares of $1 each Reserves: Share premium Accumulated profits Non-current liabilities 8% Loan note Current liabilities Accounts payable Taxation

22,800 16,400 500

39,700 103,700 20,000

23,600 24,200 200

48,000 95,800 12,000

4,000 57,200

61,200 81,200 5,000

2,400 42,700

45,100 57,100 18,000

15,300 2,200

17,500 103,700

17,700 3,000

20,700 95,800

The following information is relevant: The fair value of Sulphates investment was $5 million in excess of its book value at the date of acquisition. The fair values of Sulphates other net assets were equal to their book values. Consolidated goodwill is deemed to have a five-year life, with time apportioned charges (treated as an operating expense) in the year of acquisition. No dividends have been paid or proposed by either company.

Required: (a) (i) Prepare the consolidated income statment and balance sheet of Hydrate for the year to 30 September 2002 using the purchase method of accounting (acquisition accounting); and (13 marks)

(ii) Prepare a consolidated income statement and the consolidated SHAREHOLDERS FUNDS section of the balance sheet of Hydrate for the year to 30 September 2002 using the uniting of interests method of accounting (merger accounting). (7 marks) (b) Describe the distinguishing feature of a uniting of interests, and discuss whether the business combination in (a) should be accounted for as a uniting of interests. (5 marks) (25 marks)

[P.T.O.

Section B THREE questions ONLY to be attempted 2 The following figures have been extracted from the accounting records of Bloomsbury on 30 September 2002: $000 $000 Sales revenue (note (i)) 98,880 Cost of sales 56,000 Joint venture account (note (ii)) 1,200 Operating expenses 14,000 Loan interest paid 1,800 Investment income 700 Investment property at valuation 10,000 25-year leasehold factory at cost (note (iii)) 50,000 15-year leasehold factory at cost (note (iii)) 30,000 Plant and equipment at cost (note (iii)) 49,800 Depreciation 1 October 2001 25 year leasehold 10,000 Depreciation 1 October 2001 15 year leasehold 10,000 Depreciation 1 October 2001 plant and equipment 19,800 Accounts receivable (note (i)) 16,700 Inventory 30 September 2002 7,500 Cash and bank 500 Accounts payable 9,420 Deferred tax 1 October 2001 (note (iv)) 2,100 Ordinary shares of 25 cents each 40,000 10% Redeemable (in 2005 at par) preference shares of $1 each 10,000 12% Loan note (issued in 2000) 30,000 Accumulated profits 1 October 2001 6,100 Investment property revaluation reserve 1 October 2001 2,000 Interim dividends (note (v)) 1,500 239,000 239,000 The following notes are relevant: (i) On 1 January 2002, Bloomsbury agreed to act as a selling agent for an overseas company, Brandberg. The terms of the agency are that Bloomsbury receives a commission of 10% on all sales made on behalf of Brandberg. This is achieved by Bloomsbury remitting 90% of the cash received from Brandbergs customers one month after Bloomsbury has collected it. Bloomsbury has included in its sales revenue $72 million of sales on behalf of Brandberg of which there is one months outstanding balances of $12 million included in Bloomsburys accounts receivable. The cash remitted to Brandberg during the year of $54 million (i.e. 90% of $6 million) in accordance with the terms of the agency, has been treated as the cost of the agency sales. (ii) The joint venture account represents the net balance of Bloomsburys transactions in a joint venture with Waterfront which commenced on 1 October 2001. Each venturer contributes their own assets and pays their own expenses. The revenues for the venture are shared equally. The joint venture is not a separate entity. Details of Bloomsburys joint venture transactions are: Plant and equipment at cost Share of joint venture sales revenues (50% of total sales revenues) Related cost of sales excluding depreciation Accounts receivable Accounts payable Net balance of joint venture account $000 1,500 (800) 400 200 (100) 1,200

Plant and equipment should be depreciated in accordance with the companys policy in note (iii).

(iii) On 1 October 2001 Bloomsbury had its two leasehold factories revalued (for the first time) by an independent surveyor as follows: 25 year leasehold $52 million 15 year leasehold $18 million Bloomsbury depreciates its leaseholds on a straight-line basis over the life of the lease. The directors of Bloomsbury are disappointed in the value placed on the 15-year leasehold. The surveyor has said that the fall in its value is due mainly to its unfavourable location, but in time the surveyor expects its value to increase. The directors are committed to incorporating the revalued amount of the 25-year leasehold into the financial statements, but wish to retain the historic cost basis for the 15-year leasehold. Revaluation surpluses are transferred to accumulated realised profits in line with the realisation of the related assets. Prior to the current year, Bloomsbury had adopted a policy of carrying its investment property at fair value, with the surplus being credited to reserves. For the current year it will be applying the fair value method of accounting for investment properties in IAS 40 Investment Property. The value of the investment property had increased by a further $500,000 in the year to 30 September 2002. Plant and equipment is depreciated at 20% per annum on the reducing balance basis. (iv) A provision for income tax for the year to 30 September 2002 of $5 million is required. Temporary differences (related to the difference between the tax base of the plant and its balance sheet written down value) on 1 October 2001 were $7 million and on 30 September 2002 they had declined to $5 million. Assume a tax rate of 30%. Ignore deferred tax on the property revaluations. (v) The interim dividends paid include half of the full years preference dividend. On 25 September 2002 the directors declared a final ordinary dividend of 3 cents per share. Required: Prepare the financial statements for the year to 30 September 2002 for Bloomsbury in accordance with International Accounting Standards as far as the information permits. They should include: An Income Statement; A Statement of Changes in Equity; and A Balance Sheet. (9 marks) (3 marks) (13 marks)

Notes to the financial statements are not required. (25 marks)

[P.T.O.

The principle of recording the substance or economic reality of transactions rather than their legal form lies at the heart of the Framework for Preparation and Presentation of Financial Statements (Framework) and several International Accounting Standards. The development of this principle was partly in reaction to a minority of public interest companies entering into certain complex transactions. These transactions sometimes led to accusations that company directors were involved in creative accounting. Required: (a) (i) Explain, with relevant examples, what is generally meant by the term creative accounting; (5 marks)

(ii) Explain why it is important to record the substance rather than the legal form of transactions and describe the features that may indicate that the substance of a transaction is different from its legal form. (5 marks) (b) (i) Atkinss operations involve selling cars to the public through a chain of retail car showrooms. It buys most of its new vehicles directly from the manufacturer on the following terms: Atkins will pay the manufacturer for the cars on the date they are sold to a customer or six months after they are delivered to its showrooms whichever is the sooner. The price paid will be 80% of the retail list price as set by the manufacturer at the date that the goods are delivered. Atkins will pay the manufacturer 15% per month (of the cost price to Atkins) as a display charge until the goods are paid for. Atkins may return the cars to the manufacturer any time up until the date the cars are due to be paid for. Atkins will incur the freight cost of any such returns. Atkins has never taken advantage of this right of return. The manufacturer can recall the cars or request them to be transferred to another retailer any time up until the time they are paid for by Atkins.

Required: Discuss which party bears the risks and rewards in the above arrangement and come to a conclusion on how the transactions should be treated by each party. (6 marks) (ii) Atkins bought five identical plots of development land for $2 million in 1999. On 1 October 2001 Atkins sold three of the plots of land to an investment company, Landbank, for a total of $24 million. This price was based on 75% of the fair market value of $32 million as determined by an independent surveyor at the date of sale. The terms of the sale contained two clauses: Atkins can re-purchase the plots of land for the full fair value of $32 million (the value determined at the date of sale) any time until 30 September 2004; and On 1 October 2004, Landbank has the option to require Atkins to re-purchase the properties for $32 million. You may assume that Landbank seeks a return on its investments of 10% per annum.

Required: Discuss the substance of the above transactions; and (3 marks)

Prepare extracts of the income statement and balance sheet (ignore cash) of Atkins for the year to 30 September 2002: if the plots of land are considered as sold to Landbank; and reflecting the substance of the above transactions. (2 marks) (4 marks) (25 marks)

This is a blank page. Question 4 begins on page 8.

[P.T.O.

The financial statements of Nedberg for the year to 30 September 2002, together with the comparative balance sheet for the year to 30 September 2001 are shown below: Income Statement year to 30 September 2002: $m Sales revenue Cost of sales (note (1)) Gross profit for period Operating expenses (note (1)) Interest Loan note Profit before tax Taxation Net profit for the period Dividends: ordinary Interim Dividends: ordinary Final Net profit for period Balance sheets as at 30 September: Non-current assets Property, plant and equipment Intangible assets (note (2)) Current assets Inventory Accounts receivable Cash Total assets Equity and liabilities Ordinary Shares of $1 each Reserves Share premium Revaluation Accumulated profits Less dividends paid and declared Non-current liabilities (note (3)) Current liabilities (note (4)) Total equity and liabilities $m (120) (280) $m 3,820 (2,620) 1,200 (300) 900 (30) 870 (270) 600 (400) 200 2002 $m 1,890 650 2,540

$m

2001 $m 1,830 300 2,130

1,420 990 70

2,480 5,020 750 350 140

940 680 nil

1,620 3,750 500 100 nil

2,010 (400)

1,610 2,850 870 1,300 5,020

1,700 (300)

1,400 2,000 540 1,210 3,750

Notes to the financial statements: (1) Cost of sales includes depreciation of property, plant and equipment of $320 million and a loss on the sale of plant of $50 million. It also includes a credit for the amortisation of government grants. Operating expenses include a charge of $20 million for the amortisation of goodwill. (2) Intangible non-current assets: 2002 $m 470 180 650 300 260 310 870 875 nil 15 280 130 1,300 2001 $m 100 200 300 100 300 140 540 730 115 5 200 160 1,210

Deferred development expenditure Goodwill

(3) Non-current liabilities: 10% loan note Government grants Deferred tax

(4) Current liabilities: Accounts payable Bank overdraft Accrued loan interest Declared dividends unpaid Taxation

The following additional information is relevant: (i) Intangible fixed assets: The company successfully completed the development of a new product during the current year, capitalising a further $500 million before amortisation charges for the period. (ii) Property, plant and equipment/revaluation reserve: The company revalued its buildings by $200 million on 1 October 2001. The surplus was credited to a revaluation reserve. New plant was acquired during the year at a cost of $250 million and a government grant of $50 million was received for this plant. On 1 October 2001 a bonus issue of 1 new share for every 10 held was made from the revaluation reserve. $10 million has been transferred from the revaluation reserve to realised profits as a year-end adjustment in respect of the additional depreciation created by the revaluation. The remaining movement on property, plant and equipment was due to the disposal of obsolete plant. (iii) Share issues: In addition to the bonus issue referred to above Nedberg made a further issue of ordinary shares for cash. Required: (a) A cash flow statement for Nedberg for the year to 30 September 2002 prepared in accordance with IAS 7 Cash Flow Statements. (20 marks) (b) Comment briefly on the financial position of Nedberg as portrayed by the information in your cash flow statement. (5 marks) (25 marks)

[P.T.O.

You are a partner in a small audit and accounting practice. You have just completed the audit and finalised the financial statements of a small family owned company in discussion with its managing director Mrs Harper. After the meeting Mrs Harper has asked for your help. She has obtained the published financial statements of several quoted companies in which she is considering buying some shares as a personal investment. She presents you with the following information: (a) In the year to 30 September 2002, two companies, Gamma and Toga, reported identical profits before tax of $100 million. Information in the Chairmens reports said both companies also expected profits from their core activities (to be interpreted as from continuing operations) to grow by 10% in the following year. Mrs Harper has extracted information from the income statements and made the following summary: Operating profit: Continuing activities Acquisitions Discontinued activities Gamma $ million 70 nil 30 100 Toga $ million 90 50 (40) 100

A note to the financial statements of Toga said that both the discontinuation and acquisition occurred on 1 April 2002 and were part of an overall plan to focus on its traditional core activities after incurring large losses on a new foreign venture. Required: (i) Briefly explain to Mrs Harper why information on discontinued operations is useful; (3 marks)

(ii) Calculate the expected operating profit for both companies for the year to 30 September 2003 (assuming the Chairmens growth forecasts are correct): in the absence of information of the discontinued operations; and based on the information provided above. (4 marks)

(b) Taylor is another company about which Mrs Harper has obtained the following information from its published financial statements: Earnings per share: Year to 30 September Basic earnings per share 2002 25 cents 2001 20 cents

The earnings per share is based on attributable earnings of $50 million ($30 million in 2001) and 200 million ordinary shares in issue throughout the year (150 million weighted average number of ordinary shares in 2001). Balance sheet extracts: 8% Convertible loan stock $ million 200 $ million 200

The loan stock is convertible to ordinary shares in 2004 on the basis of 70 new shares for each $100 of loan stock. Note to the financial statements: There are directors share options (in issue since 1999) that allow Taylors directors to subscribe for a total of 50 million new ordinary shares at a price of $150 each. (Assume the current rate of income tax for Taylor is 25% and the market price of its ordinary shares throughout the year has been $250) Mrs Harper has read that the trend of the earnings per share is a reliable measure of a companys profit trend. She cannot understand why the increase in profits is 67% ($30 million to $50 million), but the increase in the earnings per share is only 25% (20 cents to 25 cents). She is also confused by the company also quoting a diluted earnings per share figure, which is lower than the basic earnings per share.

10

Required: (i) Explain why the trend of earnings per share may be different from the trend of the reported profit, and which is the more useful measure of performance; (3 marks)

(ii) Calculate the diluted earnings per share for Taylor based on the effect of the convertible loan stock and the directors share options for the year to 30 September 2002 (ignore comparatives); and (5 marks) (iii) Explain the relevance of the diluted earnings per share measure. (4 marks)

(c) Mrs Harper has noticed that the tax charge for a company called Stepper is $5 million on profits before tax of $35 million. This is an effective rate of tax of 143%. Another company Jenni has an income tax charge of $10 million on profit before tax of $30 million. This is an effective rate of tax of 333% yet both companies state the rate of income tax applicable to them is 25%. Mrs Harper has also noticed that in the cash flow statements each company has paid the same amount of tax of $8 million. Required: Advise Mrs Harper of the possible reasons why the income tax charge in the financial statements as a percentage of the profit before tax may not be the same as the applicable income tax rate, and why the tax paid in the cash flow statement may not be the same as the tax charge in the income statement. (6 marks) (25 marks)

End of Question Paper

11

Answers

Part 2 Examination Paper 2.5(INT) Financial Reporting (International Stream) 1 (a) (i) Using the purchase (acquisition) method of accounting. Hydrate Consolidated Income Statement year to 30 September 2002 Sales revenue (24,000 + (6/12 x 20,000)) Cost of sales (16,600 + (6/12 x 11,800)) Gross profit Operating expenses (1,600 + (6/12 x 1,000) + 3,000 goodwill depreciation (w (ii))) Taxation (2,000 + (6/12 x 3,000)) Profit for the year Consolidated balance sheet at 30 September 2002: Non-current assets Property, plant and equipment (64,000 + 35,000) Investments (12,800 + 5,000 fair value adjustment) Goodwill (30,000 3,000 (w (ii))) Current Assets Inventory (22,800 + 23,600) Accounts receivable (16,400 + 24,200) Bank (500 + 200) Total assets Equity and liabilities: Ordinary shares of $1 each (20,000 + 15,000 (w (ii))) Reserves: Share premium (4,000 + 75,000 (w (ii))) Accumulated profits (w (i)) Non-current liabilities 8% Loan notes (5,000 + 18,000) Current liabilities Accounts payable (15,300 + 17,700) Taxation (2,200 + 3,000)

December 2002 Answers

$000 34,000 (22,500) 11,500 (5,100) 6,400 (3,500) 2,900 $000 99,000 17,800 27,000 143,800

$000

46,400 40,600 700

87,700 231,500 35,000

79,000 56,300

135,300 170,300 23,000

33,000 5,200

38,200 231,500

(ii)

Using the uniting of interest (merger) method of accounting. Hydrate Consolidated Income Statement year to 30 September 2002 Sales revenue (24,000 + 20,000) Cost of sales (16,600 + 11,800) Gross profit Operating expenses (1,600 + 1,000) Taxation (2,000 + 3,000) Profit for the year Hydrate Share capital and reserves as at 30 September 2002: $000 Ordinary shares of $1 each (20,000 + 15,000 (w (ii))) Reserves: Share premium Capital reserve (reclassification of Sulphates share premium) Accumulated profits (w (iii)))

44,000 (28,400) 15,600 (2,600) 13,000 (5,000) 8,000 $000 35,000

4,000 2,400 96,900

103,300 138,300

15

(b)

The distinguishing features of a uniting of interests are: It is not possible to identify an acquirer. Instead of a dominant party, the shareholders of the joining entities unite in a substantially equal arrangement to share control over the combined entity. All parties to the combination participate in the management of the combined business. The sizes of the combining entities should be broadly similar leading to a substantially equal exchange of voting common shares. This should ensure that no one party is in a position to dominate the combined business due to its previous relative size. There must not be a significant reduction in the rights attaching to the shares of one of the combining parties, as this would weaken the position of that party. The above is generally evidenced by: The substantial majority of (if not all) of the voting shares of the combining parties are exchanged or pooled. The fair value of one entity is not significantly different from that of the other parties. The shareholders of each party maintain substantially the same voting rights and interests, relative to each other, in the combined entity. No partys share of the equity of the combining entities should depend on the performance of their previous business. In effect, all parties must share fairly (i.e. in proportion to their previous holdings) in the future prosperity (or otherwise) of the whole of the combined business.

It is not possible to be absolutely certain from the limited information given in the question whether all of the above criteria for a uniting of interest have been satisfied, but it does appear likely. The following observations can be made: Although Hydrate is acting on a strategy of acquisition to achieve economies of scale, the use of the phrase compatible partner companies may be indicative of a uniting of interests rather than an acquisition approach. The sizes of the companies are broadly similar (20,000 : 15,000 shares). There is no guidance in IAS 22 Business Combinations of how the term substantially equal should be interpreted. The consideration is all in the form of equity and satisfies the share exchange criterion. The composition of the new management and whether all shares rank equally would need to be determined from the details and terms of the combination agreement. Workings Acquisition accounting: (i) Consolidated accumulated profits: Hydrates reserves per question Sulphates post acquisition reserves (6/12 x 4,200) Goodwill amortisation (see below)

$000s 57,200 2,100 (3,000) 56,300

(ii)

Goodwill/Cost of control in Sulphate: Hydrate issued 5 shares for every 4 in Sulphate. Therefore Hydrate issued 15 million (12 million/4 x 5) shares at a value of $6 each to the shareholders of Sulphate. This would be recorded in Hydrates books as ordinary share capital of $15 million and share premium of $75 million. $000s Investment at cost (15 million x $6) Less ordinary shares of Sulphate Less share premium Less pre-acquisition reserves (42,700 2,100 see (i)) Less revaluation of investment Goodwill on consolidation 12,000 2,400 40,600 5,000 $000s 90,000

(60,000) 30,000

Amortisation of goodwill for the year to 30 September 2002 will be $30 million/5 years x 6/12 = $3 million (iii) Uniting of interests: A feature of a uniting of interests is that most, if not all, of the subsidiarys reserves will be included as group reserves. For Hydrate the consolidated reserves will be: Hydrates reserves per question Sulphates reserve per question Adjustment re share capital (15,000 issued 12,000 acquired) Accumulated profits at 30 September 2002 $000 57,200 42,700 (3,000) 96,900

16

Bloomsbury Income Statement Year to 30 September 2002 $000 Sales revenue (98,880 7,200 (w (i)) + 800 (w (ii))) Cost of sales (w (i)) Gross profit Other operating income: Agency commission (w (i)) Investment income surplus on investment property (iii) Invement ince other Operating expenses Loss on revaluation of property (w (iii)) Loan interest (1,800 + 1,800 accrued) Preference dividends (500 + 500 accrued (w (v))) Operating profit Taxation (5,000 600 deferred tax (w (iv))) Net profit for the period Bloomsbury Statement of Changes in Equity Year to 30 September 2002 Share capital Revaluation Investment reserve prop. revaln $000 $000 $000 Balance at 1 October 2001 40,000 nil 2,000 Surplus on revaluation of property (w (iii)) 12,000 Net profit for the period Ordinary dividends (w (v)) Transfer to realised profits (w (iii)) (600) (2,000) Balance at 30 September 2002 40,000 11,400 nil Bloomsbury Balance Sheet as at 30 September 2002 Tangible non-current assets Property, plant and equipment (w (iii)) Investments investment property (10,000 + 500 revaluation) Current Assets Inventory Accounts receivable (16,700 1,200 + 120 (w (i)) + 200 (w (ii))) Cash Total Assets Equity and liabilities: Ordinary shares of 25 cents each Reserves: Accumulated profits ((b) above) Revaluation reserve (12,000 600 (w (iii))) Non-current liabilities (w (vii)) Current liabilities Trade and other payables (w (vi)) Taxation Proposed dividends (w (v)) Total equity and liabilities $000 92,480 (61,700) 30,780 720 500 700 (14,000) (2,000) (3,600) (1,000) 1,200

(20,600) 12,100 (4,400) 7,700

Accumulated profits $000 6,100 7,700 (5,800) 2,600 10,600

Total $000 48,100 12,000 7,700 (5,800) nil 62,000

$000

$000 90,800 10,500 101,300

7,500 15,820 500

23,820 125,120 40,000

10,600 11,400

22,000 62,000 41,500

11,820 5,000 4,800

21,620 125,120

17

Workings (i) Cost of sales: Per question Brandberg adjustment see below Depreciation leaseholds (w (iii)) Depreciation plant and equipment (w (iii)) Joint venture expenses (see (ii))

$000

$000 56,000 (5,400) 10,700 400 61,700

4,400 6,300

The companys treatment of the transactions in relation to the agreement with Brandberg is incorrect. Bloomsbury has treated the sales and expenses as if they were its own sales rather than acting as an agent and receiving commission. The entries required to correct the error are: Sales revenue Cost of sales Accounts receivable Commission receivable (10% x $72 million) Due from Brandberg (Accounts receivable) Dr 7,200 Cr 5,400 1,200 720 120 7,320 7,320

(ii)

The joint venture with Waterfront qualifies to be treated under IAS 31 Financial Reporting of Interests in Joint Ventures as a jointly controlled operation. The Standard requires that each party should account for its own assets, liabilities and results according to the terms of the agreement. Thus Bloomsbury transactions with the joint venture will be treated as if they were Bloomsburys own transactions and included in the appropriate line items together with other similar transactions e.g. sales revenues will include $800,000 in respect of the joint venture.

(iii) Tangible non-current assets leaseholds Where a company chooses to revalue a non-current asset, it must revalue all the assets of the same class. Thus, in this case, Bloomsbury must recognise the fall in the value of the 15-year leasehold factory. 25-year leasehold revaluation surplus is $12 million (52m (50m 10m)) 15-year leasehold revaluation deficit is $2 million (18m (30m 10m) The revaluation loss must be charged to income; it cannot be offset against the surplus on the 25-year leasehold. A transfer from the revaluation reserve to retained profits must be made. This will represent the partial realisation of the surplus on the 25-year leasehold. It is realised at $600,000 per annum ($12 million/20 years) in line with the remaining life of the leasehold. The balance sheet values of the properties will be: at revalued amount $000 52,000 18,000 70,000 depreciation $000 2,600 1,800 4,400 NBV $000 49,400 16,200 65,600

25-year leasehold 15-year leasehold

The accumulated depreciation on the 25-year leasehold of $10,000 represents five years depreciation, thus after its revaluation it would have a remaining life of 20 years. A similar exercise with the 15-year leasehold gives a remaining life of 10 years. These figures are used to calculate the depreciation charges, which are charged to cost of sales. Investment property Under IAS 40 movements in the fair value of investment properties must be taken to income. Also on the first adoption of the Standard any previous surplus on an investment property revaluation reserve is transferred to realised profits. Plant and equipment: The plant used as part of the joint venture is included with other plant: Plant at cost (49,800 + 1,500 joint venture) Accumulated depreciation 1 October 2001 Net book value before depreciation for year Depreciation for year (charged to cost of sales) (20% x 31,500) Net book value at 30 September 2002 $000 51,300 (19,800) 31,500 (6,300) 25,200

18

(iv) As the temporary differences have fallen by $2 million this will cause a reversal of deferred tax of $2 million x 30% = $600,000. This will reduce the tax charge for the year and the deferred tax balance will be $2,100,000 $600,000 = $1,500,000. (v) The interim dividends are half of the preference dividend of $500,000 (10% x $10 million x 6/12) and the balance must be an interim ordinary dividend of $1 million. The final proposed dividend is another $500,000 preference and $48 million ordinary (40 million x 4 x 3 cents). Under IAS 32 Financial Instruments: Disclosure and Presentation redeemable preference shares have the characteristics of debt and must be treated as such. The preference dividends will be treated as interest costs and the shares will appear under non-current liabilities, not equity.

(vi) Current liabilities Accounts payable per question Joint venture creditor Accrued loan interest Accrued preference dividend

9,420 100 1,800 500 11,820 30,000 10,000 1,500 41,500

(vii) Non-current liabilities 12% Loan note 10% Redeemable preference shares (w (v)) Deferred tax (2,100 600 (w (iv)))

(a)

(i)

Creative accounting is a term in general use to describe the practice of applying inappropriate accounting policies or entering into complex or special purpose transactions with the objective of making a companys financial statements appear to disclose a more favourable position, particularly in relation to the calculation of certain key ratios, than would otherwise be the case. Most commentators believe creative accounting stops short of deliberate fraud, but is nonetheless undesirable as it is intended to mislead users of financial statements. Probably the most criticised area of creative accounting relates to off balance sheet financing. This occurs where a company has financial obligations that are not recorded on its balance sheet. There have been several examples of this in the past: finance leases treated as operating leases borrowings (usually convertible loan stock) being classified as equity secured loans being treated as sales (sale and repurchase agreements) the non-consolidation of special purpose vehicles (quasi subsidiaries) that have been used to raise finance offsetting liabilities against assets (certain types of accounts receivable factoring) The other main area of creative accounting is that of increasing or smoothing profits. Examples of this are: the use of inappropriate provisions (this reduces profit in good years and increases them in poor years) not providing for liabilities, either at all or not in full, as they arise. This is often related to environmental provisions, decommissioning costs and constructive obligations. restructuring costs not being charged to income (often related to a newly acquired subsidiary the costs are effectively added to goodwill) It should be noted that recent International Accounting Standards have now prevented many of the above past abuses, however more recent examples of creative accounting are in use by some of the new Internet/Dot.com companies. Most of these companies do not (yet) make any profit so other performance criteria such as site hits, conversion rates (browsers turning into buyers), burn periods (the length of time cash resources are expected to last) and even sales revenues are massaged to give a more favourable impression.

(ii)

One of the primary characteristics of financial statements is reliability i.e. they must faithfully represent the transactions and other events that have occurred. It can be possible for the economic substance of a transaction (effectively its commercial intention) to be different from its strict legal position or form. Thus financial statements can only give a faithful representation of a companys performance if the substance of its transactions is reported. It is worth stressing that there will be very few transactions where their substance is different from their legal form, but for those where it is, they are usually very important. This is because they are material in terms of their size or incidence, or because they may be intended to mislead.

19

Common features which may indicate that the substance of a transaction (or series of connected transactions) is different from its legal form are: Where the ownership of an asset does not rest with the party that is expected to experience the risks and reward relating to it (i.e. equivalent to control of the asset). Where a transaction is linked with other related transactions. It is necessary to assess the substance of the series of connected transactions as a whole. The use of options within contracts. It may be that options are either almost certain to be (or not to be) exercised. In such cases these are not really options at all and should be ignored in determining commercial substance. Where assets are sold at values that differ from their fair values (either above or below fair values). Many complex transactions often contain several of the above features. Determining the true substance of transactions can be a difficult and sometimes subjective procedure. (b) (i) This is an example of consignment inventory. From Atkinss point of view the main issue is whether or at what point in time the goods have been purchased and should therefore be recognised. As is often the case in these types of agreement there is conflicting evidence as to which party bears the risks and rewards relating to the vehicles. The manufacturer retains the legal right of ownership until the goods are paid for by Atkins. Consistent with this the manufacturer also has the right to have the goods returned or passed on to another supplier. The fact that Atkins may choose to return the goods to the manufacturer is also indicative that the manufacturer is exposed to the risk of obsolescence or falling values. These factors would seem to suggest that the vehicles have not been sold and should therefore remain in the inventory of the manufacturer and not be recognised in the accounting records of Atkins. There are, however, some contrary indications to this view. The price for the goods is fixed as of the date of transfer, not the date that they are deemed sold. This means that Atkins is protected from any price increases by the manufacturer. The 15% paid to the manufacturer appears to be in substance a finance charge, despite it being described as a display charge. A finance charge indicates that Atkins must have a liability to the manufacturer; in effect this liability is the account payable in respect of the cost of the vehicles. Although Atkins has a right of return, it cannot exercise this without a cost. There is an explicit freight cost, but this may not be the only cost. It could well be that Atkins may suffer poor future supplies from the manufacturer if it does return goods. The question says that Atkins has never taken advantage of this option, which would seem to suggest that it should be ignored. Conclusion: The substance of this transaction appears to suggest that the goods have been purchased by Atkins and the company is paying a finance cost. Therefore the vehicles should be recognised on Atkinss balance sheet, together with the respective liability. It would seem logical that if Atkins considers the goods as purchased, then the manufacturer should consider them as sold. The problem is that prudence may prevent the manufacturer from recognising the profit on the sale, as the period for the right to return the goods has not expired. Therefore, either the sales are not recognised by the manufacturer (the goods would remain in its inventory), or if they are, a provision should be made in respect of the unrealised profits. This could lead to the unusual situation that the goods may appear on both companies balance sheets. (ii) Although the question says that Atkins has sold the land to Landbank and even though there will be a legal transfer of the land, the substance of this transaction is that of a secured loan. The two clauses in combination mean that in practice Atkins will repurchase the land on or before 1 October 2004. This is because if its value is above $32 million Atkins will exercise its option to purchase, conversely if the value is below $32 million Landbank plc will exercise its option to require a repurchase. Either way Atkins will repurchase the land. When this is understood it becomes clear that the difference between the sale price of $24 million and the repurchase price of $32 million represents a finance charge on a secured loan. Assuming the land is sold: Income statement year to 30 September 2002 Sales Cost of sales (3/5 x $2 million) Profit on sale of land Balance sheet as at 30 September 2002 Non-current assets Development land ($2 million $12 million above) Assuming the arrangement is secured loan: Income statement year to 30 September 2002 Interest on loan (10% of in substance loan of $24 million) Balance sheet as at 30 September 2002 Non-current assets Development land at cost Non-current liabilities Secured loan Accrued interest 2,400,000 240,000 $ $ 2,400,000 1,200,000 1,200,000

800,000

(240,000)

2,000,000

(2,640,000)

20

Cash Flow Statement of Nedberg for the Year to 30 September 2002: Cash flows from operating activities Net profit before interest and tax Adjustments for: Amortisation development expenditure (w (i)) Amortisation goodwill (200 180) Depreciation property, plant and equipment Amortisation of government grant (w (ii)) Loss on sale of plant Increase in inventory (1,420 940) Increase in accounts receivable (990 680) Increase in accounts payable (875 730) Cash generated from operations Interest paid (30 (15 5 accrual adjustments)) Income tax paid (w (iii)) Net cash from operating activities Cash flows from investing activities Purchase property, plant and equipment (w (iv)) Capitalised development costs (w (i)) Receipt of government grant Proceeds of sale of plant (w (iv)) Net cash from operating activities Cash flows from financing activities Issue of ordinary shares (w (v)) Issue of loan notes (300 100) Dividends paid (200 final for 2001 + 120 interim for 2002) Net cash generated from financing activities Net increase in cash and cash equivalents Cash and cash equivalents at beginning of period Cash and cash equivalents at end of period Workings (i) Development expenditure: Balance b/f Amount capitalised Amortisation balancing figure Balance c/f (ii) Government grant: Balance b/f Cash received Amortisation Balance c/f (iii) Income tax: Tax provision b/f Deferred tax b/f Charged to income statement Tax provision c/f Deferred tax c/f Difference cash paid (250) (500) 50 20 $m $m 900

130 20

150 320 (90) 50 (480) (310) 145 685 (20) (130) 535

(680)

450 200 (320) 330 185 (115) 70

100 500 (130) 470 300 50 (90) 260 160 140 270 (130) (310) 130

21

$m (iv) Property, plant and equipment: Balance b/f Revaluation surplus Plant acquired Depreciation Disposal at net book value balancing figure Balance c/f Disposal of plant: Net book value from above Loss on sale (from question) Difference is sale proceeds (v) Share capital: Ordinary shares b/f Bonus issue 1 for 10 (from revaluation reserve) Ordinary shares c/f Difference issue for cash Plus increase in share premium (350 100) Total cash proceeds of issue of ordinary shares (vi) Reconciliation of reserve movements Revaluation reserve: Balance b/f Revaluation of buildings Bonus issue Transfer to realised profits Balance c/f Accumulated profits: Balance b/f Net profit for period Dividends paid Dividends declared Transfer from revaluation reserve Balance c/f (b)

$m 1,830 200 250 (320) (70) 1,890 70 (50) 20 (500) (50) 750 200 250 450

nil 200 (50) (10) 140 1,400 600 (120) (280) (400) 10 1,610

The cash flows generated from operations of $685 million are relatively healthy and more than adequate to pay the interest costs and taxation, but not as large as the equivalent profit figure. For most companies the operating cash flows tend to be higher than the profit before interest and tax due to the effects of depreciation/amortisation charges (which are not cash flows). In the case of Nedberg the depreciation/amortisation effect has been more than offset by a much higher investment in working capital of $645 million. Inventory has increased by over 50% and accounts receivable by 45%. This may be an indication of expanding activity, but it could also be an indication of poor inventory management policy and poor credit control, or even the presence of some obsolete inventory or unprovided bad accounts receivables. A cause of concern is the size of the dividends, at $400 million they represent 67% of the profit for the period and cash flows for dividends (last years final plus this years interim) are also high at $320 million. This is a very high distribution ratio, and it seems curious that the company is returning such large amounts to shareholders at the same time as they are raising finance. $450 million has been received from the issue of new shares and $200 million from a further issue of loan notes. The company has invested considerably in new plant ($250 million) and even more so in development expenditure ($500 million). If management has properly applied the capitalisation criteria in IAS 38 Intangible Assets, then this indicates that they expect good future returns from the investment in new products or processes. The net investment in non-current assets is $680 million which closely correlates to the proceeds from financing of $650 million. In general it is acceptable to finance increases in the capacity of non-current assets by raising additional finance, however operating cash flows should finance replacement of consumed fixed assets.

22

(a)

(i)

The requirement in IAS 35 Discontinuing Operations to provide an analysis between continuing and discontinuing operations is intended to improve the predictive usefulness of financial statements. In essence there can be no more important information when trying to assess the future performance of a company than to know which parts of it are continuing their operations and those which have ceased or been sold or are about to be in the near future. Only the results of continuing operations should be used in forecasting future results; profits or losses from discontinuing operations will not be repeated. Information on discontinued operations can also help to assess managements strategy. One would expect loss-making activities to be sold or closed down, but selling a profitable activity may indicate that a company has liquidity or debt problems.

(ii)

If no information on continuing and discontinuing activities were available then the best estimate of the future profit of both companies would be $110 million (i.e. $100 million x 110%) Utilising the available information, a very different picture emerges: Gamma $ million 77 Toga $ million 209

Forecast profit

Gammas forecast is based on profit from continuing activities of $70 million increasing by 10% to $77 million. Togas forecast is also based on its continuing activities, but it is in two parts. The existing activities that made profits of $90 million would be expected to produce profits of $99 million in 2003. Its newly acquired activities would be expected to produce profits of $110 million. The latter figure is based on the $50 million profit in 2002 being for only six months, a full year would have presumably yielded $100 million. In 2003 this would increase by 10% to $110 million. (b) (i) The trend shown by a comparison of a companys profits over time is rather a raw measure of performance and can be misleading without careful interpretation of all the events that the company has experienced. In the year to 30 September 2002, Taylors eps has increased by 25% (from 20 cents to 25 cents), whereas its profit has increased by a massive 67% (from $30 to $50 million). It is not possible to determine exactly what has caused the difference between the percentage increase in the eps and the percentage increase in the reported profit of Taylor, but a simpler example may illustrate a possible explanation. Assume company A acquired company B by way of a share exchange. Both companies had the same market value and the same profits. A comparison of As post combination profits with its pre-combination profits would be very misleading. They would have appeared to double. This is because the post combination figures incorporate both companies results, whereas the pre-combination profits would be those of company A alone (assuming it is not accounted for as a uniting of interest). The trend shown by the earnings per share goes some way to addressing such distortion. In the above the increase in post combination profit would also be accompanied by an increase in the issued share capital (due to the share exchange) thus the reported eps of company A would not be distorted by its acquisitive growth. It can therefore be argued that the trend of a companys eps is a more reliable measure of its earnings performance than the trend shown by its reported profits. Both the convertible loan stock and the directors share options will give rise to dilution: 8% Loan stock on conversion there will be 140 million new shares (200 million x 70/100) The interest saved, net of tax at 25%, will be $12 million ($200 million x 8% x 75%) The directors share options will yield income of $75 million (50 million x $150). At the market price of $250 this would be sufficient to purchase 30 million shares. As the options are for 50 million shares the dilutive effect of the options is 20 million shares. Diluted eps year to 30 September 2002: Earnings $62 million Number of shares 360 million Diluted eps 172 cents (basic $50 million + $12 million re loan stock) (basic 200 million + 140 million re loan stock + 20 million re options)

(ii)

(iii) The relevance of the diluted earnings per share measure is that it highlights the problem of relying too heavily on a companys basic eps when trying to predict future performance. There can exist certain circumstances which may cause future eps to be lower than current levels irrespective of future profit performance. These are said to cause a dilution of the eps. Common examples of diluting circumstances are the existence of convertible loan stock or share options that may cause an increase in the future number of shares without being accompanied by a proportionate increase in earnings. It is important to realise that a diluted eps figure is not a prediction of what the future eps will be, but it is a warning to shareholders that, based on the current level of earnings, the basic reported eps would be lower if the diluting circumstances had crystallised. Clearly future eps will be based on future profits and the number of shares in issue.

23

(c)

There are two main reasons why the income tax charge in the financial statements is not at the same rate as the stated percentage. The first reason is that tax is payable on the taxable profits of a company, which may differ considerably from the accounting profit. Such differences may be because some items of income or expenditure included in the financial statements may be disallowable for tax purposes (or allowed in a different accounting period) and some taxation allowances (e.g. tax depreciation allowances) are not included in the accounting profit. These differences may be mitigated by deferred tax on temporary differences. The second reason for differences is that the income tax charge does not usually consist solely of the charge on the current years profit. Commonly the tax charge also includes an element of deferred tax (this may be a debit or credit) and possibly an adjustment to the previous years tax provision (due to it being settled at an amount different to the provision). Other more complex items such as withholding taxes on income and double (dual) taxation relief may also be included in the tax charge. The main reason why the income tax charge in the income statement differs to that in the cash flow statement is that the tax charge in the financial statements is a provision for tax that is normally settled in the following period. This means that the cash flow figure for tax actually paid is the amount needed to settle the previous years tax liability. Other differences may be due to items referred to above such as deferred tax movements that are not cash flows.

24

Part 2 Examination Paper 2.5(INT) FInancial Reporting (International Stream)

December 2002 Marking Scheme

This marking scheme is given as a guide in the context of the suggested answers. Scope is given to markers to award marks for alternative approaches to a question, including relevant comment, and where well-reasoned conclusions are provided. This is particularly the case for written answers where there may be more than one definitive solution. 1 (a) (i) Income statement: all line items 1 mark each except operating expenses Balance sheet: property, plant and equipment investments goodwill inventory and accounts receivable bank and loan note accounts payable and tax share capital share premium accumulated profits available maximum (ii) Income statement: all line items 1/2 mark each profit b/f Hydrate and Sulphate, 1 mark each Equity and reserves: share premium share premium of Sulphate now classed as a capital reserve accumulated profits Hydrate plus Sulphate qccumulated profits share capital adjustment available maximum (b) 1 mark for each feature together with compliance comment Maximum for question marks 3 2 1 1 3 1 1 1 1 1 1 16 13 2 2 1 1 1 1 8 7 5 25

25

marks 2 (a) Income statement sales revenue cost of sales commission investment income plus operating expenses surplus on investment property (as income) loan interest preference dividend (as an expense) loss on revaluation of 15 year leasehold taxation available maximum (b) Changes in equity profit for period revaluation surplus of 25 year leasehold dividend transfers to realised profits available maximum (c) Balance sheet property, plant and equipment investment property inventory and cash accounts receivable trade and other payables income tax provision dividends deferred tax share capital revaluation reserves available maximum Maximum for question 2 3 1 1 1 1 1 1 1 12 9

1 1 1 1 4 3

5 1 1 2 2 1 1 1 1 1 16 13 25

(a)

(i)

description of creative accounting examples of creative accounting maximum importance of substance features indicating a difference between substance and legal form maximum

2 3 5 2 3 5 6 3 6 25

(ii)

(b)

(i) (ii)

1 mark per relevant point to a 1 mark per relevant point to a and 1 mark per correct figure in the financial statements to a

maximum maximum maximum Maximum for question

26

marks 4 (a) net cash flows from operating activities 1 mark per item except loan interest except taxation capital expenditure proceeds from the sale of the plant capital expenditure other items,1 mark per component financing equity shares financing loan note equity dividends movement in cash and cash equivalents available maximum (b) 1 mark per relevant point to a maximum Maximum for question 8 2 2 2 3 2 1 2 1 23 20 5 25

(a)

(i) (ii)

1 mark per relevant point to a $110 for both companies if no information available applying the information available $77 million for Gamma applying the information available $209 million for Toga

maximum

3 1 1 2 4 3 1 1 2 1 5 4 6 25

maximum (b) (i) (ii) 1 mark per relevant point to a number of shares re loan stock interest saved dilutive number of share re options calculation of diluted eps maximum (iii) 1 mark per relevant point to a (c) 1 mark per relevant point to a maximum maximum Maximum for question maximum

27

Financial Reporting
(International Stream)
PART 2 THURSDAY 4 DECEMBER 2003

QUESTION PAPER Time allowed 3 hours This paper is divided into two sections Section A This ONE question is compulsory and MUST be answered THREE questions ONLY to be answered

Section B

Paper 2.5(INT)

Section A This ONE question is compulsory and MUST be attempted 1 Highmoor, a public listed company, acquired 80% of Slowmoors ordinary shares on 1 October 2002. Highmoor paid an immediate $2 per share in cash and agreed to pay a further $120 per share if Slowmoor made a profit within two years of its acquisition. Highmoor has not recorded the contingent consideration. The balance sheets of the two companies at 30 September 2003 are shown below: Highmoor $ million $ million 585 225 nil 810 Slowmoor $ million $ million 172 113 140 225

Tangible non-current assets Investments (note (ii)) Software (note (iii))

Current assets Inventory Accounts receivable Tax asset Bank Total assets

185 195 1nil 120

1,200 1,010

42 36 80 nil

158 383

Equity and liabilities Capital and reserves: Ordinary shares of $1 each Accumulated profits 1 October 2002 Accumulated profits profit/loss for year

1400 230 100 1330 730 150 (35)

100 115 215

Non-current liabilities 12% loan note 8% Inter company loan (note (ii)) Current liabilities Accounts payable Taxation Overdraft Total equity and liabilities

1nil 1nil 210 170 1nil

nil

135 145 171 1nil 117

180

1,280 1,010

188 383

The following information is relevant: (i) At the date of acquisition the fair values of Slowmoors net assets were approximately equal to their book values. (ii) Included in Highmoors investments is a loan of $50 million made to Slowmoor. On 28 September 2003, Slowmoor paid $9 million to Highmoor. This represented interest of $4 million for the year and the balance was a capital repayment. Highmoor had not received nor accounted for the payment, but it had accrued for the loan interest receivable as part of its accounts receivable figure. There are no other intra group balances. (iii) The software was developed by Highmoor during 2002 at a total cost of $30 million. It was sold to Slowmoor for $50 million immediately after its acquisition. The software had an estimated life of five years and is being amortised by Slowmoor on a straight-line basis.

(iv) Due to the losses of Slowmoor since its acquisition, the directors of Highmoor are not confident it will return to profitability in the short term. (v) For the purposes of realising any negative goodwill, in its acquisition plan, Highmoor had estimated at the date of acquisition that Slowmoor would make losses of $15 million (of which $12 million would be attributable to Highmoor) before returning to profitability. The remaining weighted average useful life at the date of acquisition of the acquired depreciable non-monetary assets can be taken as four years (straight-line basis). The group accounting policy for any positive goodwill is to write it off on a straight-line basis over a period of four years. (vi) Highmoor uses the allowed alternative treatment in IAS 22 Business Combinations to account for the fair value of identifiable assets and liabilities on acquisition. Required: (a) Prepare the consolidated balance sheet of Highmoor as at 30 September 2003, explaining your treatment of the contingent consideration. (20 marks) (b) Describe the circumstances in which negative goodwill may arise. Your answer should refer to the particular issues of the above acquisition. (5 marks) (25 marks)

[P.T.O.

Section B THREE questions ONLY to be attempted 2 The following extracted balances relate to Tourmalet at 30 September 2003: $000 Ordinary shares of 20 cents each Accumulated profits at 1 October 2002 Revaluation reserve at 1 October 2002 6% Redeemable preference shares 2005 Trade accounts payable Tax Land and buildings at valuation (note (iii)) 150,000 Plant and equipment cost (note (v)) 98,600 Investment property valuation at 1 October 2002 (note (iv)) 10,000 Depreciation 1 October 2002 land and buildings Depreciation 1 October 2002 plant and equipment Trade accounts receivable 31,200 Inventory 1 October 2002 26,550 Bank 3,700 Sales revenue (note (i)) Investment income (from properties) Purchases 158,450 Distribution expenses 26,400 Administration expenses 23,200 Interim preference dividend 900 Ordinary dividend paid 2,500 531,500 $000 50,000 47,800 18,500 30,000 35,300 2,100

9,000 24,600

313,000 1,200

531,500

The following notes are relevant: (i) Sales revenue includes $50 million for an item of plant sold on 1 June 2003. The plant had a book value of $40 million at the date of its sale, which was charged to cost of sales. On the same date, Tourmalet entered into an agreement to lease back the plant for the next five years (being the estimated remaining life of the plant) at a cost of $14 million per annum payable annually in arrears. An arrangement of this type is deemed to have a financing cost of 12% per annum. No depreciation has been charged on the item of plant in the current year. (ii) The inventory at 30 September 2003 was valued at cost of $285 million. This includes $45 million of slow moving goods. Tourmalet is trying to sell these to another retailer but has not been successful in obtaining a reasonable offer. The best price it has been offered is $2 million. (iii) On 1 October 1999 Tourmalet had its land and buildings revalued by a firm of surveyors at $150 million, with $30 million of this attributed to the land. At that date the remaining life of the building was estimated to be 40 years. These figures were incorporated into the companys books. There has been no significant change in property values since the revaluation. $500,000 of the revaluation reserve will be realised in the current year as a result of the depreciation of the buildings. (iv) Details of the investment property are: Value 1 October 2002 Value 30 September 2003 $10 million $98 million

The company adopts the fair value method in IAS 40 Investment Property of valuing its investment property. (v) Plant and equipment (other than that referred to in note (i) above) is depreciated at 20% per annum on the reducing balance basis. All depreciation is to be charged to cost of sales.

(vi) The above balances contain the results of Tourmalets car retailing operations which ceased on 31 December 2002 due to mounting losses. The results of the car retailing operation, which is to be treated as a discontinuing operation, for the year to 30 September 2003 are: $000 Sales 15,200 Cost of sales 16,000 Operating expenses 13,200 The operating expenses are included in administration expenses in the trial balance. Tourmalet is still paying rentals for the lease of its car showrooms. The rentals are included in operating expenses. Tourmalet is hoping to use the premises as an expansion of its administration offices. This is dependent on obtaining planning permission from the local authority for the change of use, however this is very difficult to obtain. Failing this, the best option would be early termination of the lease which will cost $15 million in penalties. This amount has not been provided for. (vii) The balance on the taxation account in the trial balance is the result of the settlement of the previous years tax charge. The directors have estimated the provision for income tax for the year to 30 September 2003 at $92 million. Required: (a) Comment on the substance of the sale of the plant and the directors treatment of it. (b) Prepare the Income Statement; and (5 marks) (17 marks)

(c) A Statement of Changes in Equity for Tourmalet for the year to 30 September 2003 in accordance with current International Accounting Standards. (3 marks) Note: A balance sheet is NOT required. Disclosure notes are NOT required. (25 marks)

[P.T.O.

IAS 37 Provisions, Contingent Liabilities and Contingent Assets was issued in 1998. The Standard sets out the principles of accounting for these items and clarifies when provisions should and should not be made. Prior to its issue, the inappropriate use of provisions had been an area where companies had been accused of manipulating the financial statements and of creative accounting. Required: (a) Describe the nature of provisions and the accounting requirements for them contained in IAS 37. (6 marks) (b) Explain why there is a need for an accounting standard in this area. Illustrate your answer with three practical examples of how the standard addresses controversial issues. (6 marks) (c) Bodyline sells sports goods and clothing through a chain of retail outlets. It offers customers a full refund facility for any goods returned within 28 days of their purchase provided they are unused and in their original packaging. In addition, all goods carry a warranty against manufacturing defects for 12 months from their date of purchase. For most goods the manufacturer underwrites this warranty such that Bodyline is credited with the cost of the goods that are returned as faulty. Goods purchased from one manufacturer, Header, are sold to Bodyline at a negotiated discount which is designed to compensate Bodyline for manufacturing defects. No refunds are given by Header, thus Bodyline has to bear the cost of any manufacturing faults of these goods. Bodyline makes a uniform mark up on cost of 25% on all goods it sells, except for those supplied from Header on which it makes a mark up on cost of 40%. Sales of goods manufactured by Header consistently account for 20% of all Bodylines sales. Sales in the last 28 days of the trading year to 30 September 2003 were $1,750,000. Past trends reliably indicate that 10% of all goods are returned under the 28-day return facility. These are not faulty goods. Of these 70% are later resold at the normal selling price and the remaining 30% are sold as sale items at half the normal retail price. In addition to the above expected returns, an estimated $160,000 (at selling price) of the goods sold during the year will have manufacturing defects and have yet to be returned by customers. Goods returned as faulty have no resale value. Required: Describe the nature of the above warranty/return facilities and calculate the provision Bodyline is required to make at 30 September 2003: (i) for goods subject to the 28 day returns policy; and (8 marks)

(ii) for goods that are likely to be faulty.

(d) Rockbuster has recently purchased an item of earth moving plant at a total cost of $24 million. The plant has an estimated life of 10 years with no residual value, however its engine will need replacing after every 5,000 hours of use at an estimated cost of $75 million. The directors of Rockbuster intend to depreciate the plant at $24 million ($24 million/10 years) per annum and make a provision of $1,500 ($75 million/5,000 hours) per hour of use for the replacement of the engine. Required: Explain how the plant should be treated in accordance with International Accounting Standards and comment on the Directors proposed treatment. (5 marks) (25 marks)

This is a blank page. Question 4 begins on page 8.

[P.T.O.

Comparator assembles computer equipment from bought in components and distributes them to various wholesalers and retailers. It has recently subscribed to an interfirm comparison service. Members submit accounting ratios as specified by the operator of the service, and in return, members receive the average figures for each of the specified ratios taken from all of the companies in the same sector that subscribe to the service. The specified ratios and the average figures for Comparators sector are shown below. Ratios of companies reporting a full years results for periods ending between 1 July 2003 and 30 September 2003 Return on capital employed 221% Net assets turnover 18 times Gross profit margin 30% Net profit (before tax) margin 125% Current ratio 16:1 Quick ratio 09:1 Inventory holding period 46 days Accounts receivable collection period 45 days Accounts payable payment period 55 days Debt to equity 40% Dividend yield 6% Dividend cover 3 times Comparators financial statements for the year to 30 September 2003 are set out below: Income statement $000 Sales revenue 2,425 Cost of sales (1,870) Gross profit 555 Other operating expenses (215) Operating profit 340 Interest payable (34) Exceptional item (note (ii)) (120) Profit before taxation 186 Income tax (90) Profit after taxation 96 Extracts of changes in equity: Accumulated profits 1 October 2002 179 Net profit for the period 96 Dividends paid (interim $60,000; final $30,000) (90) Accumulated profits 30 September 2003 185

Balance Sheet Non-current assets (note (i)) Current Assets Inventory Accounts receivable Bank

$000

$000 540

275 320 nil

595 1,135 150 185 335 300

Share Capital and Reserves Ordinary shares (25 cents each) Accumulated profits

Non-current liabilities 8% loan notes Current liabilities Bank overdraft Trade accounts payable Taxation 65 350 85

500 1,135

Notes (i) The details of the non-current assets are: Cost $000 At 30 September 2003 3,600

Accumulated depreciation $000 3,060

Net book value $000 540

(ii) The exceptional item relates to losses on the sale of a batch of computers that had become worthless due to improvements in microchip design. (iii) The market price of Comparators shares throughout the year averaged $600 each. Required: (a) Explain the problems that are inherent when ratios are used to assess a companys financial performance. Your answer should consider any additional problems that may be encountered when using interfirm comparison services such as that used by Comparator. (7 marks) (b) Calculate the ratios for Comparator equivalent to those provided by the interfirm comparison service. (6 marks) (c) Write a report analysing the financial performance of Comparator based on a comparison with the sector averages. (12 marks) (25 marks)

[P.T.O.

(a) (i)

IAS 12 Income Tax was issued in 1996 and revised in 2000. It details the requirements relating to the accounting treatment of deferred tax.

Required: Explain why it is considered necessary to provide for deferred tax and briefly outline the principles of accounting for deferred tax contained in IAS 12 Income Tax. (5 marks) (ii) Bowtock purchased an item of plant for $2,000,000 on 1 October 2000. It had an estimated life of eight years and an estimated residual value of $400,000. The plant is depreciated on a straight-line basis. The tax authorities do not allow depreciation as a deductible expense. Instead a tax expense of 40% of the cost of this type of asset can be claimed against income tax in the year of purchase and 20% per annum (on a reducing balance basis) of its tax base thereafter. The rate of income tax can be taken as 25%. Required: In respect of the above item of plant, calculate the deferred tax charge/credit in Bowtocks income statement for the year to 30 September 2003 and the deferred tax balance in the balance sheet at that date. (6 marks) Note: work to the nearest $000. (b) Bowtock has leased an item of plant under the following terms: Commencement of the lease was 1 January 2002 Term of lease 5 years Annual payments in advance $12,000 Cash price and fair value of the asset $52,000 at 1 January 2002 Implicit interest rate within the lease (as supplied by the lessor) 8% per annum (to be apportioned on a time basis where relevant). The companys depreciation policy for this type of plant is 20% per annum on cost (apportioned on a time basis where relevant). Required: Prepare extracts of the income statement and balance sheet for Bowtock for the year to 30 September 2003 for the above lease. (5 marks) (c) (i) Explain why events occurring after the balance sheet date may be relevant to the financial statements of the previous period. (4 marks)

(ii) At 30 September 2003 Bowtock had included in its draft balance sheet inventory of $250,000 valued at cost. Up to 5 November 2003, Bowtock had sold $100,000 of this inventory for $150,000. On this date new government legislation (enacted after the year end) came into force which meant that the unsold inventory could no longer be marketed and was worthless. Bowtock is part way through the construction of a housing development. It has prepared its financial statements to 30 September 2003 in accordance with IAS 11 Construction Contracts and included a proportionate amount of the total estimated profit on this contract. The same legislation referred to above (in force from 5 November 2003) now requires modifications to the way the houses within this development have to be built. The cost of these modifications will be $500,000 and will reduce the estimated total profit on the contract by that amount, although the contract is still expected to be profitable. Required: Assuming the amounts are material, state how the information above should be reflected in the financial statements of Bowtock for the year ended 30 September 2003. (5 marks) (25 marks)

End of Question Paper

10

Answers

Part 2 Examination Paper 2.5 (INT) Financial Reporting (International Stream) 1 (a) Consolidated Balance Sheet of Highmoor as at 30 September 2003: $ million Assets Non-current assets Tangible (585 + 172) Intangible Consolidated negative goodwill (40 19 (w (ii))) Software (w (v)) Investments (225 160 shares 50 loan (w (iv)) + 13)

December 2003 Answers

$ million

757 1(21) 124 128 1788 127 127 180 129

Current assets Inventory (85 + 42) Accounts receivable (95 4 in transit (w (iv)) + 36) Tax asset Bank (20 + 9 in transit (w (iv))) Total assets Equity and liabilities Capital and reserves: Ordinary shares $1 each Accumulated profits (w (i))

363 1,151

400 305 705 1 1 1281 1117 1170 35 43

Non-current liabilities 12% loan notes Minority interest (w (iii)) Current liabilities Accounts payable (210 + 71) Overdraft Taxation Total equity and liabilities Workings (Note: all figures in $ million) (i) Unrealised profit (w (v)) Minority interest (20% x 115) Pre-acq profit (80% x 150) Post acq loss (80% x 35) Balance c/f

368 1,151

Accumulated profits Highmoor Slowmoor 116 123 120 (28) 305 321 115 Cost of control

B/f Post acq loss Realisation of negative goodwill (w (ii))

Highmoor 330 1(28) 119

Slowmoor 115

321 Ordinary shares (80% x 100) Pre acq profit (w (i))

115

(ii) Investments at cost (100 x 80% x $2) Negative goodwill 160

180 120

140 200 200 The contingent consideration has not been included in the above calculation. IAS 22 Business Combinations only requires contingent consideration to be included in the cost of an acquisition if it is probable that the amount will be paid and it can be measured reliably. The additional $96 million (i.e. $120 per share) is only payable if Slowmoor makes a profit within two years of acquisition. In the year since acquisition the company made a loss of $35 million, much higher than the $15 million in its acquisition plan, and the directors of Highmoor are now less confident of the future prospects of Slowmoor. This seems to indicate that it is unlikely that any further consideration will be paid and the above treatment is justified.

13

The negative goodwill of $40 million will be realised as follows: Attributable to: Proportion realised year to 30 September 2003 expected losses attributable to Highmoor 12 100% non-monetary assets 28 25% (four year life) Total negative goodwill 40 (iii) Minority interest Ordinary shares (20% x 100) Accumulated profits (w (i)) Balance c/f 43 43

amount 12 17 19

20 23 43

(iv) Elimination of loan and accrued interest: The investments of Highmoor will show an unadjusted amount of $50 million as a loan to Slowmoor. The cash in transit of $9 million from Slowmoor should be applied $4 million to cancel the accrued interest receivable and the balance of $5 million to the investment (loan). When this adjustment is made the investment and the loan will cancel each other out. (v) The net book value of the software in Slowmoors books is $40 million. If the software had been depreciated on its original cost of $30 million it would have a book value of $24 million ($30 less $6 million depreciation at 20% per annum). Thus there is an unrealised profit on the transfer of the software of $16 million ($40 million $24 million).

(b)

Negative goodwill arises in bookkeeping where the consideration given for a business is less than the fair value of the net assets acquired. Intuitively it does not make sense for a vendor to sell net assets for less than they are worth. This view is reflected by the IASB as they appear rather sceptical about the existence of negative goodwill. They say where an acquisition appears to create negative goodwill, a careful check of the value of the assets acquired and whether any liabilities have been omitted is required. Negative goodwill may arise for several reasons; the most obvious is that there has been a bargain purchase. This may occur through the vendor being in a poor financial position and needing to realise assets quickly, or it may be due to good negotiating skills on the part of the acquirer, or the vendor may not realise how much the assets are really worth. A more controversial occasion where negative goodwill arises is where a company, in determining the amount of consideration it is willing to pay for a business, will take into account the cost of anticipated future losses and post acquisition reorganisation expenditure that it believes will be required. The effect of this is that it would reduce the consideration offered/paid. As these costs cannot generally be recognised as a liability at the date of purchase, this can lead to the consideration being lower than the recognisable net assets. In relation to the acquisition of Slowmoor the following are questionable issues: Highmoor may be trying to deliberately create losses at Slowmoor to avoid paying the further consideration. An example of this may be the transfer price of the software. The additional consideration of $96 million, if payable, would change the negative goodwill into positive goodwill of $56 million. The tax asset of Slowmoor may be questionable. Accounting standards are quite restrictive over the recognition of tax assets.

14

(a)

The sale of the plant has been incorrectly treated on two counts. Firstly even if it were a genuine sale it should not have been included in sales and cost of sales, rather it should have been treated as the disposal of a non-current asset. Only the profit or loss on the disposal would be included in the income statement (requiring separate disclosure if material). However even this treatment would be incorrect. As Tourmalet will continue to use the plant for the remainder of its useful life, the substance of this transaction is a secured loan. Thus the receipt of $50 million for the sale of the plant should be treated as a loan. The rentals, when they are eventually paid, will be applied partly as interest (at 12% per annum) and the remainder will be a capital repayment of the loan. In the income statement an accrual for loan interest of 12% per annum on $50 million for four months ($2 million) is required. Tourmalet Income Statement Year to 30 September 2003 continuing operations $000 Sales revenues (313,000 50,000 (see above)) 247,800 Cost of sales (w (i)) (128,800) Gross profit (loss) 119,000 Distribution expenses (26,400) Administration expenses (w (iii)) (20,000) Profit (loss) on the ordinary activities before interest 72,600 Financing cost (w (iv)) Loss on investment properties ($10 million $98 million) Investment income Profit before tax Income tax (9,200 2,100) Profit for the period

(b)

discontinuing operations $000 15,200 (16,000) (800) nil (4,700) (5,500)

Total $000 263,000 (144,800) 118,200 (26,400) (24,700) 67,100 (3,800) (200) 1,200 64,300 (7,100) 57,200

Note: IAS 35 Discontinuing Operations does not require a specific presentation of the results of a discontinued operation. The above is only one of several acceptable presentations. (c) Tourmalet Statement of Changes in Equity Year to 30 September 2003 Accumulated Revaluation Profits reserve $000 $000 At 1 October 2002 47,800 18,500 Profit for period 57,200 Transfer to realised profit 500 (500) Ordinary dividends paid (2,500) At 30 September 2003 103,000 18,000

Ordinary shares $000 50,000

Total $000 116,300 57,200 (2,500) 171,000

50,000

Note: IAS 32 Financial Instruments: Disclosure and Presentation says redeemable preference shares have the substance of debt and should be treated as non-current liabilities and not as equity. This also means that preference dividends are treated as a finance cost in the income statement. Workings (i) Cost of sales: Opening inventory Purchases Transfer to plant (see (a)) Depreciation (w (ii)) Closing inventory (285 million 25 million see below) $000 26,550 158,450 (40,000) 25,800 (26,000) 144,800

The slow moving inventory should be written down to its estimated realisable value. Despite the optimism of the Directors, it would seem prudent to base the realisable value on the best offer so far received (i.e. $2 million).

15

(ii)

Non-current asset depreciation: Buildings $120/40 years Plant per trial balance ((98,600 24,600) x 20%) Plant plant treated as sold (40,000/5 years)

$000 3,000 14,800 8,000 25,800

Note: investment properties do not require depreciating under the fair value model in IAS 40. Instead they are revalued each year with the surplus or deficit being taken to income. For information only: in the balance sheet cost/valuation $000 150,000 98,600 40,000 accumulated depreciation $000 12,000 39,400 8,000 net book value $000 138,000 59,200 32,000 229,200

Land and buildings Plant per trial balance Plant incorrectly treated as sold

(iii) It would seem prudent to accrue for the penalty on the lease as it is uncertain that the permission for a change of use will be granted. In these circumstances, the payment of the penalty will be the lowest liability. This gives total administration expenses of $24,700 (23,200 + 1,500), of which $4,700 (3,200 + 1,500) is classed as discontinuing. (iv) Finance costs: income statement $000 2,000 1,800 3,800

Accrued interest on in-substance loan (see (a)) Preference dividends (30,000 x 6%)

Note this balance sheet is provided for information only. It does not form part of the answer or marking scheme. Tourmalet Balance Sheet as at 30 September 2003 $000 Non-current assets (w (ii)) Investment properties $000 229,200 9,800 239,000

Current Assets Inventory (w (i)) Trade accounts receivable Bank Total assets Equity and liabilities Capital and Reserves: Ordinary shares of 20c each Accumulated profits Revaluation reserve (see (c))

26,000 31,200 3,700

60,900 299,900

50,000 103,000 18,000 121,000 171,000

Non-current liabilities 6% Redeemable preference shares $1 each In-substance loan (see (a)) Current liabilities Trade accounts payable Accrued penalty cost (w (iii)) Accrued finance costs (2,000 + 900) Taxation

30,000 50,000 35,300 1,500 2,900 9,200

80,000

Total equity and liabilities

48,900 299,900

16

(a)

IAS 37 Provisions, Contingent Liabilities and Contingent Assets only deals with those provisions that are regarded as liabilities. The term provision is also generally used to describe those amounts set aside to write down the value of assets such as depreciation charges and provisions for diminution in value (e.g. provision to write down the value of damaged or slow moving inventory). The definition of a provision in the Standard is quite simple; provisions are liabilities of uncertain timing or amount. If there is reasonable certainty over these two aspects the liability is a creditor. There is clearly an overlap between provisions and contingencies. Because of the uncertainty aspects of the definition, it can be argued that to some extent all provisions have an element of contingency. The IASB distinguishes between the two by stating that a contingency is not recognised as a liability if it is either only possible and therefore yet to be confirmed as a liability, or where there is a liability but it cannot be measured with sufficient reliability. The IASB notes the latter should be rare. The IASB intends that only those liabilities that meet the characteristics of a liability in its Framework for the Preparation and Presentation of Financial Statements should be reported in the balance sheet. IAS 37 summarises the above by requiring provisions to satisfy all of the following three recognition criteria: there is a present obligation (legal or constructive) as a result of a past event; it is probable that a transfer of economic benefits will be required to settle the obligation; the obligation can be estimated reliably.

A provision is triggered by an obligating event. This must have already occurred, future events cannot create current liabilities. The first of the criteria refers to legal or constructive obligations. A legal obligation is straightforward and uncontroversial, but constructive obligations are a relatively new concept. These arise where a company creates an expectation that it will meet certain obligations that it is not legally bound to meet. These may arise due to a published statement or even by a pattern of past practice. In reality constructive obligations are usually accepted because the alternative action is unattractive or may damage the reputation of the company. The most commonly quoted example of such is a commitment to pay for environmental damage caused by the company, even where there is no legal obligation to do so. To summarise: a company must provide for a liability where the three defining criteria of a provision are met, but conversely a company cannot provide for a liability where they are not met. The latter part of the above may seem obvious, but it is an area where there has been some past abuse of provisioning as is referred to in (b). (b) The main need for an accounting standard in this area is to clarify and regulate when provisions should and should not be made. Many controversial areas including the possible abuse of provisioning are based on contravening aspects of the above definitions. One of the most controversial examples of provisioning is in relation to future restructuring or reorganisation costs (often as part of an acquisition). This is sometimes extended to providing for future operating losses. The attraction of providing for this type of expense/loss is that once the provision has been made, the future costs are then charged to the provision such that they bypass the income statement (of the period when they occur). Such provisions can be glossed over by management as exceptional items, which analysts are expected to disregard when assessing the companys future prospects. If this type of provision were to be incorporated as a liability as part of a subsidiarys net assets at the date of acquisition, the provision itself would not be charged to the income statement. IAS 37 now prevents this practice as future costs and operating losses (unless they are for an onerous contract) do not constitute past events. Another important change initiated by IAS 37 is the way in which environmental provisions must be treated. Practice in this area has differed considerably. Some companies did not provide for such costs and those that did often accrued for them on an annual basis. If say a company expected environmental site restoration cost of $10 million in 10 years time, it might argue that this is not a liability until the restoration is needed or it may accrue $1 million per annum for 10 years (ignoring discounting). Somewhat controversially this practice is no longer possible. IAS 37 requires that if the environmental costs are a liability (legal or constructive), then the whole of the costs must be provided for immediately. That has led to large liabilities appearing in some companies balance sheets. A third example of bad practice is the use of big bath provisions and over provisioning. In its simplest form this occurs where a company makes a large provision, often for non-specific future expenses, or as part of an overall restructuring package. If the provision is deliberately overprovided, then its later release will improve future profits. Alternatively the company could charge to the provision a different cost than the one it was originally created for. IAS 37 addresses this practice in two ways: by not allowing provisions to be created if they do not meet the definition of an obligation; and specifically preventing a provision made for one expense to be used for a different expense. Under IAS 37 the original provision would have to be reversed and a new one would be created with appropriate disclosures. Whilst this treatment does not affect overall profits, it does enhance transparency. Note: other examples would be acceptable.

17

(c)

Guarantees or warranties appear to have the attributes of contingent liabilities. If the goods are sold faulty or develop a fault within the guarantee period there will be a liability, if not there will be no liability. The IASB view this problem as two separate situations. Where there is a single item warranty, it is considered in isolation and often leads to a discloseable contingent liability unless the chances of a claim are thought to be negligible. Where there are a number of similar items, they should be considered as a whole. This may mean that whilst the chances of a claim arising on an individual item may be small, when taken as a whole, it should be possible to estimate the number of claims from past experience. Where this is the case, the estimated liability is not considered contingent and it must be provided for. (i) Bodylines 28-day refund policy is a constructive obligation. The company probably has notices in its shops informing customers of this policy. This would create an expectation that the company will honour its policy. The liability that this creates is rather tricky. The company will expect to give customers refunds of $175,000 ($1,750,000 x 10%). This is not the liability. 70% of these will be resold at the normal selling price, so the effect of the refund policy for these goods is that the profit on their sale must be deferred. The easiest way to account for this is to make a provision for the unrealised profit. This has to be calculated for two different profit margins: Goods manufactured by Header (at a mark up of 40% on cost): $24,500 ($175,000 x 70% x 20%) x 40/140 = $7,000 Goods from other manufacturers (at a mark up of 25% on cost) $98,000 ($175,000 x 70% x 80%) x 25/125 = $19,600 The sale of the remaining 30% at half the normal selling price will create a loss. Again this must be calculated for both group of sales: Goods manufactured by Header were originally sold for $10,500 (175,000 x 30% x 20%). These will be resold (at a loss) for half this amount i.e. $5,250. Thus a provision of $5,250 is required. Goods manufactured by other manufacturers were originally sold for $42,000 (175,000 x 30% x 80%). These will be resold (at a loss) for half this amount i.e. $21,000. Thus a provision of $21,000 is required. The total provision in respect of the 28 day return facility will be $52,850 (7,000 + 19,600 + 5,250 + 21,000). (ii) Goods likely to be returned because they are faulty require a different treatment. These are effectively sales returns. Normally the manufacturer will reimburse the cost of the faulty goods. The effect of this is that Bodyline will not have made the profit originally recorded on their sale. This applies to all goods other than those supplied by Header. Thus these sales returns would be $128,000 (160,000 x 80%) and the credit due from the manufacturer would be $102,400 (128,000 x 100/125 removal of profit margin). The overall effect is that Bodyline would have to remove profits of $25,600 from its financial statements. For those goods supplied by Header, Bodyline must suffer the whole loss as this is reflected in the negotiated discount. Thus the provision required for these goods is $32,000 (160,000 x 20%), giving a total provision of $57,600 (25,600 + 32,000).

(d)

The Directors proposed treatment is incorrect. The replacement of the engine is an example of what has been described as cyclic repairs or replacement. Whilst it may seem logical and prudent to accrue for the cost of a replacement engine as the old one is being worn out, such practice leads to double counting. Under the Directors proposals the cost of the engine is being depreciated as part of the cost of the asset, albeit over an incorrect time period. The solution to this problem lies in IAS 16 Property, Plant and Equipment. The plant constitutes a complex asset i.e. one that may be thought of as having separate components within a single asset. Thus part of the plant $165 million (total cost of $24 million less $75 assumed cost of the engine) should be depreciated at $165 million per annum over a 10-year life and the engine should be depreciated at $1,500 per hour of use (assuming machine hour depreciation is the most appropriate method). If a further provision of $1,500 per machine hour is made, there would be a double charge against profit for the cost of the engine. IAS 37 also refers to this type of provision and says that the future replacement of the engine is not a liability. The reasoning is that the replacement could be avoided if, for example, the company chose to sell the asset before replacement was due. If an item does not meet the definition of a liability it cannot be provided for.

18

(a)

Ratios are used to assess the financial performance of a company by comparing the calculated figures to various other sources. This may be to previous years ratios of the same company, it may be to the ratios of a similar rival company, to accepted norms (say of liquidity ratios) or, as in this example, to industry averages. The problems inherent in these processes are several. Probably the most important aspect of using ratios is to realise that they do not give the answers to the assessment of how well a company has performed, they merely raise the questions and direct the analyst into trying to determine what has caused favourable or unfavourable indicators. In many ways it can be said that ratios are only as useful as the skills of the person using them. It is also true that any assessment should also consider other information that may be available including non-financial information. More specific problem areas are: Accounting policies: if two companies have different accounting policies, it can invalidate any comparison between their ratios. For example return on capital employed is materially affected by revaluations of assets. Comparing this ratio for two companies where one has revalued its assets and the other carries them at depreciated historic cost would not be very meaningful. Similar examples may involve depreciation methods, inventory valuation policies etc. Accounting practices: this is similar to differing accounting policies in its effects. An example of this would be the use of debtor factoring. If one company collects its accounts receivable in the normal way, then the calculation of the accounts receivable collection period would be a reasonable indication of the efficiency of its credit control department. However if a company chose to factor its accounts receivable (i.e. sell them to a finance company) then the calculation of its collection period would be meaningless. A more controversial example would be the engineering of a lease such that it fell to be treated as an operating lease rather than a finance lease. Balance sheet averages: many ratios are based on comparing income statement items with balance sheet items. The ratio of accounts receivable collection period is a good example of this. For such ratios to have meaning, there is an assumption that the year-end balance sheet figures are representative of annual norms. Seasonal trading and other factors may invalidate this assumption. For example the level of accounts receivable and inventory of a toy manufacturer could vary largely due to the nature of its seasonal trading. Inflation can distort comparisons over time. The definition of an accounting ratio. If a ratio is calculated by two companies using different definitions, then there is an obvious problem. Common examples of this are gearing ratios (some use debt/equity, others may use debt/debt + equity). Also where a ratio is partly based on a profit figure, there can be differences as to what is included and what is excluded from the profit figure. Problems of this type include the treatment of extraordinary items and finance costs. The use of norms can be misleading. A desirable range for the current ratio may be say between 15 and 2 : 1, but all businesses are different. This would be a very high ratio for a supermarket (with few accounts receivable), but a low figure for a construction company (with high levels of work in progress). Looking at a single ratio in isolation is rarely useful. It is necessary to form a view when considering ratios in combination with other ratios.

A more controversial aspect of ratio analysis is that management have sometimes indulged in creative accounting techniques in order that the ratios calculated from published financial statements will show a more favourable picture than the true underlying position. Examples of this are sale and repurchase agreements, which manipulate liquidity figures, and off balance sheet finance which distorts return on capital employed. Inter firm comparisons: Of particular concern with this method of using ratios is: they are themselves averages and may incorporate large variations in their composition. Some inter firm comparison agencies produce the ratios analysed into quartiles to attempt to overcome this problem. it may be that the sector in which a company is included may not be sufficiently similar to the exact type of trade of the specific company. The type of products or markets may be different. companies of different sizes operate under different economies of scale, this may not be reflected in the industry average figures. the year end accounting dates of the companies included in the averages are not going to be all the same. This highlights issues of balance sheet averages and seasonal trading referred to above. Some companies try to minimise this by grouping companies with approximately similar year-ends together as in the example of this question, but this is not a complete solution.

19

(b)

Calculation of specified ratios: Comparator Return on capital employed (186 +34 loan interest/635) 346% Net assets turnover (2,425/635) 38 times Gross profit margin (555/2,425 x 100) 229% Net profit (excluding exceptionals) margin (306/2,425 x 100) 126% Net profit (before tax) margin (186/2,425 x 100) 77% Current ratio (595/500) 119 : 1 Quick ratio (320/500) 064 : 1 Inventory holding period (275/1,870 x 365) 54 days Accounts receivable collection period (320/2,425 x 365) 48 days Creditor payment period (350/1,870 x 365) (based on cost of sales) 68 days Debt to equity (300/335 x 100) 90% Dividend yield (see below) 25% Dividend cover (96/90) 107 times The workings are in $000 (unless otherwise stated) and are for Comparators ratios. The dividend yield is calculated from a dividend per share figure of 15c ($90,000/150,000 x 4) and a share price of $600. Thus the yield is 25% (15c/$600 x 100%). Sector average 221% 18 times 30% not available 125% 16 : 1 09 : 1 46 days 45 days 55 days 40% 6% 3 times

(c)

Analysis of Comparators financial performance compared to sector average for the year to 30 September 2003: To: From: Date: Operating performance The return on capital employed of Comparator is impressive being more than 50% higher than the sector average. The components of the return on capital employed are the asset turnover and profit margins. In these areas Comparators asset turnover is much higher (nearly double) than the average, but the net profit margin after exceptionals is considerably below the sector average. However, if the exceptionals are treated as one off costs and excluded, Comparators margins are very similar to the sector average. This short analysis seems to imply that Comparators superior return on capital employed is due entirely to an efficient asset turnover i.e. Comparator is making its assets work twice as efficiently as its competitors. A closer inspection of the underlying figures may explain why its asset turnover is so high. It can be seen from the note to the balance sheet that Comparators non-current assets appear quite old. Their net book value is only 15% of their original cost. This has at least two implications; they will need replacing in the near future and the company is already struggling for funding; and their low net book value gives a high figure for asset turnover. Unless Comparator has underestimated the life of its assets in its depreciation calculations, its non-current assets will need replacing in the near future. When this occurs its asset turnover and return on capital employed figures will be much lower. This aspect of ratio analysis often causes problems and to counter this anomaly some companies calculate the asset turnover using the cost of non-current assets rather than their net book value as this gives a more reliable trend. It is also possible that Comparator is using assets that are not on its balance sheet. It may be leasing assets that do not meet the definition of finance leases and thus the assets and corresponding obligations are not recognised on the balance sheet. A further issue is which of the two calculated margins should be compared to the sector average (i.e. including or excluding the effects of the exceptionals). The gross profit margin of Comparator is much lower than the sector average. If the exceptional losses were taken in at trading account level, which they should be as they relate to obsolete inventory, Comparators gross margin would be even worse. As Comparators net margin is similar to the sector average, it would appear that Comparator has better control over its operating costs. This is especially true as the other element of the net profit calculation is finance costs and as Comparator has much higher gearing than the sector average, one would expect Comparators interest to be higher than the sector average. Liquidity Here Comparator shows real cause for concern. Its current and quick ratios are much worse than the sector average, and indeed far below expected norms. Current liquidity problems appear due to high levels of accounts payable and a high bank overdraft. The high levels of inventory contribute to the poor quick ratio and may be indicative of further obsolete inventory (the exceptional item is due to obsolete inventory). The accounts receivable collection figure is reasonable, but at 68 days, Comparator takes longer to pay its accounts payable than do its competitors. Whilst this is a source of free finance, it can damage relations with suppliers and may lead to a curtailment of further credit. Gearing As referred to above, gearing (as measured by debt/equity) is more than twice the level of the sector average. Whilst this may be an uncomfortable level, it is currently beneficial for shareholders. The company is making an overall return of 346%, but only paying 8% interest on its loan notes. The gearing level may become a serious issue if Comparator becomes unable to maintain the finance costs. The company already has an overdraft and the ability to make further interest payments could be in doubt.

20

Investment ratios Despite reasonable profitability figures, Comparators dividend yield is poor compared to the sector average. From the extracts of the changes in equity it can be seen that total dividends are $90,000 out of available profit for the year of only $96,000 (hence the very low dividend cover). It is worthy of note that the interim dividend was $60,000 and the final dividend only $30,000. Perhaps this indicates a worsening performance during the year, as normally final dividends are higher than interim dividends. Considering these factors it is surprising the companys share price is holding up so well. Summary The company compares favourably with the sector average figures for profitability, however the companys liquidity and gearing position is quite poor and gives cause for concern. If it is to replace its old assets in the near future, it will need to raise further finance. With already high levels of borrowing and poor dividend yields, this may be a serious problem for Comparator. Yours faithfully

(a)

(i)

An explanation of the origins of why deferred tax is provided for lies in understanding that accounting profit (as reported in a companys financial statements) differs from the profit figure used by the tax authorities to calculate a companys income tax liability for a given period. If deferred tax were ignored (flow through system), then a companys tax charge for a particular period may bear very little resemblance to the reported profit. For example if a company makes a large profit in a particular period, but, perhaps because of high levels of capital expenditure, it is entitled to claim large tax allowances for that period, this would reduce the amount of tax it had to pay. The result of this would be that the company reported a large profit, but very little, if any, tax charge. This situation is usually reversed in subsequent periods such that tax charges appear to be much higher than the reported profit would suggest that they should be. Many commentators feel that such a reporting system is misleading in that the profit after tax, which is used for calculating the companys earnings per share, may bear very little resemblance to the pre tax profit. This can mean that a governments fiscal policy may distort a companys profit trends. Providing for deferred tax goes some way towards relieving this anomaly, but it can never be entirely corrected due to items that may be included in the income statement, but will never be allowed for tax purposes (referred to as permanent differences in some jurisdictions). Where tax depreciation is different from the related accounting depreciation charges this leads to the tax base of an asset being different to its carrying value on the balance sheet (these differences are called temporary differences) and a provision for deferred tax is made. This balance sheet liability approach is the general principle on which IAS 12 bases the calculation of deferred tax. The effect of this is that it usually brings the total tax charge (i.e. the provision for the current years income tax plus the deferred tax) in proportion to the profit reported to shareholders. The main area of debate when providing for deferred tax is whether the provision meets the definition of a liability. If the provision is likely to crystallise, then it is a liability, however if it will not crystallise in the foreseeable future, then arguably, it is not a liability and should not be provided for. The IASB takes a prudent approach and IAS 12 does not accept the latter argument.

(ii)

IAS 12 requires deferred tax to be calculated using the balance sheet liability method. This method requires the temporary difference to be calculated and the rate of income tax applied to this difference to give the deferred tax asset or liability. Temporary differences are the differences between the carrying amount of an asset and its tax base. Carrying value at 30 September 2003 $000 Cost of plant Accumulated depreciation at 30 September 2003 (2,000 400)/8 years for 3 years Carrying value Tax base at 30 September 2003 Initial tax base (original cost) Tax depreciation Year to 30 September 2001 (2,000 x 40%) Year to 30 September 2002 (1,200 x 20%) Year to 30 September 2003 (960 x 20%) Tax base 30 September 2003 Temporary differences at 30 September 2003 (1,400 768) Deferred tax liability at 30 September 2003 (632 x 25% tax rate) Income statement credit year to 30 September 2003 ((200 192) x 25%) $000 2,000 (600) 1,400 2,000 800 240 192

1,232 768 632 158 2

21

(b) Income statement extracts year to 30 September 2003 Depreciation of leased asset (w (i)) Lease interest expense (w (ii)) Balance sheet extracts as at 30 September 2003 Leased asset at cost Accumulated depreciation (7,800 + 10,400 (w (i))) Net book value Current liabilities Accrued lease interest (w (ii)) Obligations under finance leases (w (ii)) Non-current liabilities Obligations under finance leases (w (ii))

$ 10,400 2,672 52,000 18,200 33,800 1,872 9,504 21,696

Workings (i) Depreciation for the year ended 30 September 2002 would be $7,800 ($52,000 x 20% x 9/12) Depreciation for the year ended 30 September 2003 would be $10,400 ($52,000 x 20%) (ii) The lease obligations are calculated as follows: Cash price/fair value Rental 1 January 2002 52,000 (12,000) 40,000 2,400 800 43,200 (12,000) 31,200 1,872 624 33,696

Interest to 30 September 2002 (40,000 x 8% x 9/12) Interest to 1 January 2003 (40,000 x 8% x 3/12)

Rental 1 January 2003 Capital outstanding 1 January 2003 Interest to 30 September 2003 (31,200 x 8% x 9/12) Interest to 1 January 2004 (31,200 x 8% x 3/12)

Interest expense for the year to 30 September 2003 is $2,672 (800 + 1,872 from above), of which $1,872 is a current liability. The total capital amount outstanding at 30 September 2003 is $31,200 (the same as at 1 January 2003 as no further payments have been made). This must be split between current and non-current liabilities. Next years payment will be $12,000 of which $2,496 (1,872 + 624) is interest. Therefore capital repaid in the next year will be $9,504 (12,000 2,496). This leaves capital of $21,696 (31,200 9,504) as a non-current liability. (c) (i) Most events occurring after the balance sheet date should be properly reflected in the following years financial statements. There are two circumstances where events occurring after the balance sheet date are relevant to the current years financial statements. The first category, known as adjusting events, provides additional evidence of conditions that existed at the balance sheet date. This usually means they help to determine the value of an item that may have been uncertain at the year-end. Common examples of this are post balance sheet receipts from accounts receivable and sales of inventory. These receipts help to confirm the bad debt and inventory write down provisions. The second category is non-adjusting events. As the name suggests these do not affect the amounts contained in the financial statements, but are considered of such importance that unless they are disclosed, users of financial statements would not be properly able to assess the financial position of the company. Common examples of these would be the loss of a major asset (say due to a fire) after the balance sheet date or the sale of an investment (often a subsidiary) after the balance sheet date.

22

(ii)

Inventory Sales of goods after the balance sheet date are normally a reflection of circumstances that existed prior to the year end. They are usually interpreted as a confirmation of the value of inventory as it existed at the year end, and are thus adjusting events. In this case the sale of the goods after the year-end confirmed that the value of the inventory was correctly stated as it was sold at a profit. Goods remaining unsold at the date the new legislation was enacted are worthless. Whilst this may imply that they should be written off in preparing the financial statements to 30 September 2003, this is not the case. What it is important to realise is that the event that caused the inventory to become worthless did not exist at the year end and its consequent losses should be reflected in the following accounting period. Thus there should be no adjustment to the value of inventory in the draft financial statements, but given that it is material, it should be disclosed as a non-adjusting event. Construction contract On first appearance this new legislation appears similar to the previous example, but there is a major difference. Profits on an uncompleted long term construction contract are based on assessment of the overall eventual profit that the contract is expected to make. This new legislation will mean the overall profit is $500,000 less than originally thought. This information must be taken into account when calculating the profit at 30 September 2003. This is an adjusting event.

23

Part 2 Examination Paper 2.5 (INT) Financial Reporting (International Stream)

December 2003 Marking Scheme

This marking scheme is given as a guide in the context of the suggested answers. Scope is given to markers to award marks for alternative approaches to a question, including relevant comment, and where well-reasoned conclusions are provided. This is particularly the case for written answers where there may be more than one definitive solution. Marks 2 2 4 2 2 1/ 2 1 1/ 2 1 1 2 1 2 1/ 2 1/ 2 1 23 20 5 25

(a)

discussion of contingent consideration tangible non-current assets negative goodwill (2 for credit to realised profit) software investments inventory accounts receivable tax asset bank balance in hand ordinary shares accumulated profits provision minority interest accounts payable overdraft tax liability available maximum

(b)

1 mark per relevant point to

maximum Maximum for question

(a)

should not be treated as sales/cost of sales normally only the profit in income (may require disclosure) the substance of the transaction is a secured loan plant should be left on balance sheet sale proceeds of $50 million shown as loan rentals are partly interest and partly capital repayments available maximum

1 1 1 1 1 1 6 5

(b)

Income statement discontinuing operations figures sales cost of sales distribution expenses administration expenses finance costs (including 1 for preference dividends) loss on investment properties investment income taxation available maximum

3 2 5 1 2 2 1 1 2 19 17

(c)

Changes in equity profit for period dividends transfer to realised profits maximum Maximum for question

1 1 1 3 25

25

(a) (b) (c)

one mark per valid point to max the need for the Standard and examples to a max of 2 each 28 day refund policy constructive obligation calculation of provision for unrealised profits where goods resold at full price calculation of provision for loss on goods sold at half normal price the product warranties are treated collectively warranty cost can be estimated reliably therefore a liability, not a contingency faulty goods other than from Header not a loss, but must remove profit made on them quantified return of faulty goods manufactured by Header creates a loss quantification of loss available maximum

Marks 6 6 1 1 1 1 1 1 1 1 1 9 8 1 1 2 1 5 25

(d)

directors treatment is incorrect this is an example of a complex asset depreciation is $165m per annum plus $1,500 per machine hour replacement does not meet the definition of a liability maximum Maximum for question

(a)

up to 1 mark for each limitation Note: a good answer must refer to inter firm comparison issues
1/ 2

maximum

(b) (c)

mark for each relevant ratio

maximum

6 1 3 4 2 3 2 1 16 12 25

format discussion discussion discussion discussion discussion discussion

of: of: of: of: of: of:

profitability liquidity (and working capital ratios) gearing investment ratios other issues summary available maximum Maximum for question

(a)

(i) (ii)

one mark per valid point to carrying value and tax base at 30 September 2003 caculation of deferred tax at 30 September 2003 deferred tax credit in income statement

maximum

5 3 2 2 7 6 1 1 1 1 1 1 6 5 4 3 2 5 25

available maximum (b) depreciation of leased asset lease interest expense net book value of leased asset current liabilities: accrued lease interest current liabilities: obligations under finance leases non-current liabilities: obligations under finance leases available maximum (c) (i) (ii) one mark per valid point to discussion of value of inventory up to discussion of effect on construction contract up to maximum Maximum for question maximum

26

Financial Reporting
(International Stream)
PART 2 THURSDAY 5 JUNE 2003

QUESTION PAPER Time allowed 3 hours This paper is divided into two sections Section A This ONE question is compulsory and MUST be answered THREE questions ONLY to be answered

Section B

Paper 2.5(INT)

Section A This ONE question is compulsory and MUST be attempted 1 In recent years Hillusion has acquired a reputation for buying modestly performing businesses and selling them at a substantial profit within a period of two to three years of their acquisition. On 1 July 2002 Hillusion acquired 80% of the ordinary share capital of Skeptik at a cost of $10,280,000. On the same date it also acquired 50% of Skeptiks 10% loan notes at par. The summarised draft financial statements of both companies are: Income statements: Year to 31 March 2003 Hillusion $000 60,000 (42,000) 18,000 (6,000) 75 12,075 (3,000) 9,075 16,525 25,600

Sales revenue Cost of sales Gross profit Operating expenses Loan interest received (paid) Operating profit Taxation Profit after tax for the year Accumulated profit brought forward

Skeptik $000 24,000 (20,000) 4,000 (200) (200) 3,600 (600) 3,000 5,400 8,400 8,000 nil 8,000 8,000 16,000

Accumulated profit per balance sheet

Tangible non-current Assets Investments Current Assets Total assets Equity and liabilities Ordinary shares of $1 each Accumulated profits Non-current liabilities 10% Loan notes Current liabilities Total equity and liabilities

Balance Sheets: as at 31 March 2003 19,320 11,280 30,600 15,000 45,600

10,000 25,600 35,600 nil 10,000 45,600

2,000 8,400 10,400 2,000 3,600 16,000

The following information is relevant: (i) The fair values of Skeptiks assets were equal to their book values with the exception of its plant, which had a fair value of $32 million in excess of its book value at the date of acquisition. The remaining life of all of Skeptiks plant at the date of its acquisition was four years and this period has not changed as a result of the acquisition. Depreciation of plant is on a straight-line basis and charged to cost of sales. Skeptik has not adjusted the value of its plant as a result of the fair value exercise.

(ii) In the post acquisition period Hillusion sold goods to Skeptik at a price of $12 million. These goods had cost Hillusion $9 million. During the year Skeptik had sold $10 million (at cost to Skeptik) of these goods for $15 million. (iii) Hillusion bears almost all of the administration costs incurred on behalf of the group (invoicing, credit control etc.). It does not charge Skeptik for this service as to do so would not have a material effect on the group profit. (iv) Revenues and profits should be deemed to accrue evenly throughout the year. (v) The current accounts of the two companies were reconciled at the year-end with Skeptik owing Hillusion $750,000. (vi) Consolidated goodwill is to be written off as an operating expense over a three-year life. Time apportionment should be used in the year of acquisition. (vii) Hillusion uses the allowed alternative treatment in IAS 22 Business Combinations for recording (and depreciating) the fair values of assets and liabilities in its consolidated financial statements. Required: (a) Prepare a consolidated income statement and balance sheet for Hillusion for the year to 31 March 2003 (20 marks) (b) Explain why it is necessary to eliminate unrealised profits when preparing group financial statements; and how reliance on the entity financial statements of Skeptik may mislead a potential purchaser of the company. (5 marks) (25 marks) Note: your answer should refer to the circumstances described in the question.

[P.T.O.

Section B THREE questions ONLY to be attempted 2 The following trial balance relates to Allgone at 31 March 2003: $000 Sales revenue (note (i)) Purchases 127,850 Operating expenses 12,400 Loan interest paid 2,400 Preference dividend 1,000 Land and buildings at valuation (note (ii)) 130,000 Plant and equipment cost 84,300 Software cost 1 April 2000 10,000 Stock market investments valuation 1 April 2002 (note (iii)) 12,000 Depreciation 1 April 2002 plant and equipment Depreciation 1 April 2002 software Extraordinary item (note (iv)) 32,000 Trade receivables 23,000 Inventory 1 April 2002 19,450 Bank Trade payables Ordinary shares of 25c each 10% Preference shares 12% Loan note (issued 1 July 2002) Deferred tax Revaluation reserve (relating to land and buildings and the investments) Accumulated profits 1 April 2002 454,400 $000 236,200

24,300 6,000

350 15,200 60,000 20,000 40,000 3,000 45,000 4,350 454,400

The following notes are relevant: (i) Sales include $8 million for goods sold in March 2003 for cash to Funders, a merchant bank. The cost of these goods was $6 million. Funders has the option to require Allgone to repurchase these goods within one month of the year-end at their original selling price plus a facilitating fee of $250,000. The inventory at 31 March 2003 was counted at a cost value of $85 million. This includes $500,000 of slow moving inventory that is expected to be sold for a net $300,000. (ii) Non-current assets: On 1 April 2002 Allgone revalued its land and buildings. The details are: cost 1 April 1997 $000 20,000 80,000 valuation 1 April 2002 $000 25,000 105,000

land building

The building had an estimated life of 40 years when it was acquired and this has not changed as a result of the revaluation. Depreciation is on a straight-line basis. The surplus on the revaluation has been added to the revaluation reserve, but no other movements on the revaluation reserve have been recorded. Plant and equipment is depreciated at 20% per annum on the reducing balance basis. Software is depreciated by the sum of the digits method over a 5-year life. (iii) The investment represents 75% of the ordinary share capital of Wondaworld. Allgone has a policy of revaluing its investments at their market price at each year-end. Changes in value are taken to the revaluation reserve which at 1 April 2002 contained a surplus of $5 million for previous revaluations of the investments. The stock market price of Wondaworlds ordinary shares was $250 each on 1 April 2002 and by 31 March 2003 this had fallen to $225.

(iv) The extraordinary item is a loss incurred due to a fraud relating to the companys investments. A senior employee of the company, who left in January 2002, had diverted investment funds into his private bank account. The fraud was discovered by the employees replacement in April 2002. It is unlikely that any of the funds will be recovered. Allgone has now implemented tighter procedures to prevent such a fraud recurring. The company has been advised that this loss will not qualify for any tax relief. (v) The directors have estimated the provision for income tax for the year to 31 March 2003 at $113 million. The deferred tax provision at 31 March 2003 is to be adjusted to reflect the tax base of the companys net assets being $16 million less than their carrying values. The rate of income tax is 30%. The movement on deferred tax should be charged to the income statement. (vi) The directors declared a final ordinary dividend of 3c per share on 25 March 2003. Required: In accordance with International Accounting Standards and International Financial Reporting Standards as far as the information permits, prepare: (a) the Income Statement of Allgone for the year to 31 March 2003; and (b) the Statement of Changes in Equity for the year to 31 March 2003; and (c) a Balance Sheet as at 31 March 2003. Notes to the financial statements are not required. (25 marks) (7 marks) (5 marks) (13 marks)

[P.T.O.

Revenue recognition is the process by which companies decide when and how much income should be included in the income statement. It is a topical area of great debate in the accounting profession. The IASB looks at revenue recognition from conceptual and substance points of view. There are occasions where a more traditional approach to revenue recognition does not entirely conform to the IASB guidance; indeed neither do some International Accounting Standards. Required: (a) Explain the implications that the IASBs Framework for the Preparation and Presentation of Financial Statements (Framework) and the application of substance over form have on the recognition of income. Give examples of how this may conflict with traditional practice and some accounting standards. (6 marks) (b) Derringdo sells goods supplied by Gungho. The goods are classed as A grade (perfect quality) or B grade, having slight faults. Derringdo sells the A grade goods acting as an agent for Gungho at a fixed price calculated to yield a gross profit margin of 50%. Derringdo receives a commission of 125% of the sales it achieves for these goods. The arrangement for B grade goods is that they are sold by Gungho to Derringdo and Derringdo sells them at a gross profit margin of 25%. The following information has been obtained from Derringdos financial records: Inventory held on premises 1 April 2002 Goods from Gungho year to 31 March 2003 Inventory held on premises 31 March 2003 Required: Prepare the income statement extracts for Derringdo for the year to 31 March 2003 reflecting the above information. (5 marks) (c) Derringdo acquired an item of plant at a gross cost of $800,000 on 1 October 2002. The plant has an estimated life of 10 years with a residual value equal to 15% of its gross cost. Derringdo uses straight-line depreciation on a time apportioned basis. The company received a government grant of 30% of its cost price at the time of its purchase. The terms of the grant are that if the company retains the asset for four years or more, then no repayment liability will be incurred. If the plant is sold within four years a repayment on a sliding scale would be applicable. The repayment is 75% if sold within the first year of purchase and this amount decreases by 25% per annum. Derringdo has no intention to sell the plant within the first four years. Derringdos accounting policy for capital based government grants is to treat them as deferred credits and release them to income over the life of the asset to which they relate. Required: (i) Discuss whether the companys policy for the treatment of government grants meets the definition of a liability in the IASBs Framework; and (3 marks) A grade B grade A grade B grade A grade B grade $000 2,400 1,000 18,000 8,800 2,000 1,250

(ii) Prepare extracts of Derringdos financial statements for the year to 31 March 2003 in respect of the plant and the related grant: applying the companys policy; in compliance with the definition of a liability in the Framework. Your answer should consider whether the sliding scale repayment should be used in determining the deferred credit for the grant. (6 marks)

(d) Derringdo sells carpets from several retail outlets. In previous years the company has undertaken responsibility for fitting the carpets in customers premises. Customers pay for the carpets at the time they are ordered. The average length of time from a customer ordering a carpet to its fitting is 14 days. In previous years, Derringdo had not recognised a sale in income until the carpet had been successfully fitted as the rectification costs of any fitting error would be expensive. From 1 April 2002 Derringdo changed its method of trading by sub-contracting the fitting to approved contractors. Under this policy the sub-contractors are paid by Derringdo and they (the subcontractors) are liable for any errors made in the fitting. Because of this Derringdo is proposing to recognise sales when customers order and pay for the goods, rather than when they have been fitted. Details of the relevant sales figures are: $000 23,000 1,200 1,600

Sales made in retail outlets for the year to 31 March 2003 Sales value of carpets fitted in the 14 days to 14 April 2002 Sales value of carpets fitted in the 14 days to 14 April 2003

Note: the sales value of carpets fitted in the 14 days to 14 April 2002 are not included in the annual sales figure of $23 million, but those for the 14 days to 14 April 2003 are included. Required: Discuss whether the above represents a change of accounting policy, and, based on your discussion, calculate the amount that you would include in sales revenue for carpets in the year to 31 March 2003. (5 marks) (25 marks)

[P.T.O.

Rytetrend is a retailer of electrical goods. Extracts from the companys financial statements are set out below: Income statement for the year ended 31 March: $000 Sales revenue Cost of sales Gross profit Other operating expenses Operating profit Interest payable loan notes Interest payable overdraft Profit before taxation Taxation Profit after taxation Dividends Net profit for the period Accumulated profits brought forward Accumulated profits carried forward (260) (200) 2003 $000 31,800 (22,500) 9,300 (5,440) 3,860 (460) 3,400 (1,000) 2,400 (600) 1,800 5,880 7,680 $000 2002 $000 23,500 (16,000) 7,500 (4,600) 2,900 (500) 2,400 (800) 1,600 (400) 1,200 4,680 5,880

(500) nil

Balance Sheets as at 31 March $000 Non-current assets (note (i)) Current Assets Inventory Receivables Bank Total assets Equity and liabilities Ordinary capital ($1 shares) Share premium Accumulated profits Non-current liabilities 10% loan notes 6% loan notes Current liabilities Bank overdraft Trade payables Proposed dividends (declared before the year end) Taxation Warranty provision (note (ii)) Total equity and liabilities

2003 $000 24,500 $000

2002 $000 17,300

2,650 1,100 nil

3,750 28,250 11,500 1,500 7,680 20,680 nil 2,000

3,270 1,950 400

5,620 22,920 10,000 nil 5,880 15,880 4,000 nil

1,050 2,850 450 720 500

nil 1,980 280 630 150

5,570 28,250

3,040 22,920

Notes (i) The details of the non-current assets are: Cost $000 27,500 37,250 Accumulated depreciation $000 10,200 12,750 Net book value $000 17,300 24,500

At 31 March 2002 At 31 March 2003

During the year there was a major refurbishment of display equipment. Old equipment that had cost $6 million in September 1998 was replaced with new equipment at a gross cost of $8 million. The equipment manufacturer had allowed Rytetrend a trade in allowance of $500,000 on the old display equipment. In addition to this Rytetrend used its own staff to install the new equipment. The value of staff time spent on the installation has been costed at $300,000, but this has not been included in the cost of the asset. All staff costs have been included in operating expenses. All display equipment held at the end of the financial year is depreciated at 20% on its cost. No equipment is more than five years old. (ii) Operating expenses contain a charge of $580,000 for the cost of warranties on the goods sold by Rytetrend. The company makes a warranty provision when it sells its products and cash payments for warranty claims are deducted from the provision as they are settled. Required: (a) Prepare a cash flow statement for Rytetrend for the year ended 31 March 2003. (12 marks)

(b) Write a report briefly analysing the operating performance and financial position of Rytetrend for the years ended 31 March 2002 and 2003. (13 marks) Your report should be supported by appropriate ratios. (25 marks)

[P.T.O.

(a) Most companies prepare their financial statements under the historical cost convention. In times of rising prices it has been said that without modification such financial statements can be misleading. Required: (i) Explain the problems that can be encountered when users rely on financial statements prepared under the historical cost convention for their information needs. (6 marks) Note: your answer should consider problems with the income statement and the balance sheet. (ii) Update has been considering the effect of alternative methods of preparing their financial statements. As an example they picked an item of plant that they acquired from Suppliers on 1 April 2000 at a cost of $250,000. The following details have been obtained: the company policy is to depreciate plant at 20% per annum on the reducing balance basis. the movement in the retail price index has been: 1 April 2000 1 April 2001 1 April 2002 31 March 2003 180 202 206 216

Suppliers price catalogue at 31 March 2003 shows an item of similar plant at a cost of $320,000. On reading the specification it appears that the new model can produce 480 units per hour whereas the model owned by Update can only produce 420 units per hour.

Required: Calculate for Update the depreciation charge for the plant for the year to 31 March 2003 (based on year end values) and its balance sheet carrying value on that date using: the historical cost basis; a current purchasing power basis; and a current cost basis. (6 marks)

(b) Extracts of Niagaras consolidated income statement for the year to 31 March 2003 are: $000 36,000 (21,000) 15,000 (6,200) 8,800 1,500 (800) (4,000) 5,500 (2,800) 2,700 (115) 2,585

Sales Cost of sales Gross profit Other operating expenses Operating profit Income from associated companies Interest payable Impairment of non-current assets Profit before tax Taxation Profit after tax Minority interest Profit for financial year

10

The impairment of non-current assets attracted tax relief of $1 million which has been included in the tax charge. Niagara paid an interim ordinary dividend of 3c per share in June 2002 and declared a final dividend on 25 March 2003 of 6c per share. The issued share capital of Niagara on 1 April 2002 was: Ordinary shares of 25c each 8% Preference shares The preference shares are non-redeemable. The company also had in issue $2 million 7% convertible loan stock dated 2005. The loan stock will be redeemed at par in 2005 or converted to ordinary shares on the basis of 40 new shares for each $100 of loan stock at the option of the stockholders. Niagaras income tax rate is 30%. There are also in existence directors share warrants (issued in 2001) which entitle the directors to receive 750,000 new shares in total in 2005 at no cost to the directors. The following share issues took place during the year to 31 March 2003: 1 July 2002; a rights issue of 1 new share at $150 for every 5 shares held. The market price of Niagaras shares the day before the rights was $240. 1 October 2002; an issue of $1 million 6% non-redeemable preference shares at par. $3 million $1 million

Both issues were fully subscribed. Niagaras basic earnings per share in the year to 31 March 2002 was correctly disclosed as 24c. Required: Calculate for Niagara for the year to 31 March 2003: (i) the dividend cover and explain its significance; (3 marks) (4 marks)

(ii) the basic earnings per share including the comparative;

(iii) the fully diluted earnings per share (ignore comparative); and advise a prospective investor of the significance of the diluted earnings per share figure. (6 marks) (25 marks)

End of Question Paper

11

Answers

Part 2 Examination Paper 2.5(INT) Financial Reporting (International Stream) 1 (a) Hillusion Consolidated income statement for the year to 31 March 2003 $000 Sales revenue (60,000 + (24,000 x 9/12) 12,000 intra-group sales) Cost of sales (w (i)) Gross profit Operating expenses (6,000 + (200 x 9/12) + 300 goodwill (w (ii)) Loan interest ((200 x 9/12) 75 intra-group)) Taxation (3,000 + (600 x 9/12)) Profit after tax Minority interest (((3,000 x 9/12) 600 depreciation adjustment) x 20%) Profit after tax and minority interest Accumulated profit b/f Accumulated profit c/f Consolidated balance sheet at 31 March 2003 Non-current assets Goodwill (w (ii) 1,200 300) Tangible non-current Assets (19,320 + 8,000 + 3,200 600 depreciation adjustment (w (i))) Current Assets (w (v)) Total assets Equity and liabilities Ordinary shares of $1 each Reserves: Accumulated profits (w (iii)) Minority interest (w (iv)) Non-current liabilities 10% Loan notes (2,000 1,000 intra-group) Current liabilities (w (v)) Total equity and liabilities Workings in $000 (i) Cost of sales Hillusion Skeptik (20,000 x 9/12) Intra-group sales URP in inventory Additional depreciation 42,000 15,000 (12,000) 500 600 46,100 (6,450) (75)

June 2003 Answers

$000 66,000 (46,100) 19,900 (6,525) 13,375 (3,450) 9,925 (330) 9,595 16,525 26,120

900 29,920 30,820 21,750 52,570 10,000 26,120 36,120 2,600 1,000 12,850 52,570

The unrealised profit (URP) in inventory is calculated as: Intra-group sales are $12 million of which Skeptik has sold on $10 million leaving $2 million (1/6) still in inventory at the year-end. The cost of the sales made by Hillusion to Skeptik was $9 million giving Hillusion a profit of $3 million (12m 9m). The unrealised element of this is $500,000 ($3 million x 1/6). The fair value adjustment to the tangible non-current assets is $32 million. At the date of acquisition they have a remaining life of four years. Additional depreciation would be $800,000 per annum which requires apportioning by 9/12 = $600,000.

15

(ii)

Goodwill/Cost of control in Skeptik: Investment at cost (11,280 1,000) Less ordinary shares of Skeptik (2,000 x 80%) pre-acquisition reserves (6,150 x 80% (see below)) fair value adjustments (3,200 x 80%) Goodwill on consolidation Goodwill amortisation will be $1,200/3 x 9/12 = The pre-acquisition reserves are: Brought forward 1 April 2002 To date of acquisition (3,000 x 3/12)

10,280 (1,600) (4,920) (2,560)

(10,080) 1,200 300 5,400 750 6,150

(iii) Consolidated reserves: Hillusions reserves 25,600 Skeptiks post acquisition (((3,000 x 9/12) 600 depreciation adjustment (w (i))) x 80%) 1,320 URP in inventory (see (i)) (500) Goodwill amortisation (w (ii) above) (300) 26,120 (iv) Minority interest Ordinary shares of Skeptik (2,000 x 20%) Fair value adjustments (3,200 x 20%) Accumulated profits ((8,400 600 depreciation adjustment (w (i))) x 20%) 400 640 1,560 2,600

(v)

Current assets and liabilities Current assets: Hillusion Skeptik URP in inventory Intra-group balance

15,000 8,000 (500) (750) 21,750 10,000 3,600 (750) 12,850

Current liabilities Hillusion Skeptik Intra-group balance

16

(b)

The main reason why intra-group unrealised profits must be eliminated on consolidation is to achieve the main objective of group financial statements which is to show the position of the group as if it were a single economic entity. As such, a group cannot trade with itself, nor can it make a profit out of itself. In a similar way it cannot increase its sales or its net assets by transferring assets and liabilities between members of the group. As a simple illustrative example, but for the requirement to eliminate intra-group profits, a group could buy an item of inventory; sell it to another member of the group (at a profit), who in turn could sell it to another member of the group and so on. The result would be that each member of the group would make a profit which would then be combined to form a large group profit. This would be balanced by an over inflated inventory value in the balance sheet (in practice this effect would be limited by the application of the lower of cost and net realisable value principle of valuing inventory). Such accounting would not show a true and fair view. The main problem with using Skeptiks entity financial statements to assess its performance is that it is a related party of its parent, Hillusion. Related party transactions can distort the true economic performance and financial position of a company. In this case, the related party relationship extends to complete control of Skeptik by Hillusion. From the information in the question, it can be seen that most of Skeptiks trading is from goods it buys from Hillusion. Sales of non-group sourced goods are only $9 million (out of $24 million). It may be that these have been transferred at a favourable price allowing Skeptik to achieve a higher level of sales and make a higher than normal profit. Ultimately this course of action is no real detriment to the group as a whole as most of Skeptiks profits (and all of them if it were 100% owned) are consolidated into the group profit. In a similar manner the fact that Hillusion does not make any charge for Skeptiks administration costs acts to increase Skeptiks profit. If Skeptik was to be purchased by an external party, all these beneficial effects would cease and Skeptiks profit would then be much lower. It could be observed that Hillusion may be massaging Skeptiks financial statements with a view to obtaining a favourable price on its future sale. Hillusions past record of success in selling previous businesses at a considerable profit after only a short period of ownership supports this view.

(a)

Allgone Income Statement Year to 31 March 2003 Sales revenue (236,200 8,000 (see below)) Cost of sales (w (i)) Gross profit Operating expenses Profit from operations Financing cost (w (ii)) Profit before tax Taxation (w (iii)) Net profit from ordinary activities for the period $000 228,200 (150,000) 78,200 (12,400) 65,800 (3,850) 61,950 (13,100) 48,850

The sale of goods to Funders is an attempt to window dress the balance sheet by improving its liquidity position. It is in substance a (short term) loan with a finance cost of $250,000. (b) Allgone Statement of Changes in Equity Year to 31 March 2003 Ordinary Shares $000 Balance at 1 April 2002 60,000 Fundamental error (see below) Restated balance 60,000 Surplus on revaluation of land and buildings (w (iv)) Transfer to realised profits re building (35,000/35 years) Deficit on value of investments Net profit for the period Dividends Preference (1,000 + 1,000) Final ordinary (60,000 x 4 x 3c) Balance at 31 March 2003 60,000

Revaluation reserve $000 5,000 5,000 40,000 (1,000) (1,200)

Accumulated profits $000 4,350 (32,000) (27,650) 1,000 48,850 (2,000) (7,200) 13,000

Total $000 69,350 (32,000) 37,350 40,000 (1,200) 48,850 (2,000) (7,200) 115,800

42,800

The discovery of the major fraud is not an extraordinary item. As it occurred in previous years and is so fundamental, it should be treated as a prior period adjustment.

17

(c)

Allgone Balance Sheet as at 31 March 2003 Non-current assets Software (w (iv)) Property, plant and equipment (w (iv)) Investments (12,000 1,200 (w (iv))) Current Assets Inventory (w (i)) Trade receivables Total assets Total equity and liabilities: Ordinary shares of 25c each 10% Preference shares $1 each Reserves: Accumulated profits (see (b) above) Revaluation reserve (see (b) above) Non-current liabilities (w (v)) Current liabilities Trade payables Bank overdraft In substance loan from Funders Accrued finance costs (1,200 + 250 (w (ii))) Taxation Proposed dividends (1,000 + 7,200) Total equity and liabilities Workings (i) Cost of sales: Opening inventory Purchases Depreciation (w (iv)) software building plant Closing inventory (8,500 200 + 6,000 see below) 15,200 350 8,000 1,450 11,300 8,200 $000 $000 2,000 175,000 10,800 187,800

14,300 23,000

37,300 225,100 60,000 20,000 80,000

13,000 42,800

55,800 135,800 44,800

44,500 225,100 $000 19,450 127,850 2,000 3,000 12,000 (14,300) 150,000

The slow moving inventory requires a write down of $200,000 to its net realisable value of $300,000. The cost of the goods of the sale and repurchase agreement ($6 million) should be treated as inventory. (ii) Finance costs: Per question Accrued loan interest (see below) Accrued facilitating fee for in substance loan (treated as a finance cost) 2,400 1,200 250 3,850

The loan has been in issue for nine months, but only six months interest has been paid. Accrued interest of $1,200,000 is required. (iii) Taxation: Provision for year Deferred tax (see below) 11,300 1,800 13,100

The difference between the tax base of the assets and their carrying value of $16 million would require a balance sheet provision for deferred tax of $48 million (at 30%). The opening provision is $3 million, thus an additional charge of $18 million is required.

18

(iv) Non-current assets/depreciation/revaluation: The software was purchased on 1 April 2000 with a five-year life. The depreciation for the year to 31 March 2003 will be for the third year of its life. Using the sum of the digits method this will be 3/15 of the cost i.e. $2 million. This will give accumulated depreciation of $8 million ($6 million b/f + $2 million). Land and buildings Cost 1 April 1997 Five years depreciation (80,000 x 5/40) Net book value prior to revaluation Valuation 1 April 2002 Revaluation surplus Depreciation year to 31 March 2003 (105,000/35 years) Plant depreciation ((84,300 24,300) x 20%) Summarising: Land and building (25 + 105) Plant and equipment Property, plant and equipment Software cost/valuation $000 130,000 84,300 214,300 10,000 Buildings $000 80,000 (10,000) 70,000 105,000 35,000 3,000 12,000 accumulated depreciation $000 3,000 36,300 39,300 8,000 45,000 (1,200) (1,000) 42,800 4,800 40,000 44,800 Net book value $000 127,000 48,000 175,000 2,000 Land $000 20,000

25,000 5,000

Revaluation reserve: Per trial balance Loss of value of investments (12,000 (12,000 x 225/250)) Transfer to realised profits re building (35,000/35 years) Balance at 31 March 2003 (v) Non-current liabilities Deferred tax (3,000 + 1,800 (w (iii))) 12% Loan note

(a)

The Framework advocates that revenue recognition issues are resolved within the definition of assets (gains) and liabilities (losses). Gains include all forms of income and revenue as well as gains on non-revenue items. Gains and losses are defined as increases or decreases in net assets other than those resulting from transactions with owners. Thus in its Framework, the IASB takes a balance sheet approach to defining revenue. In effect a recognisable increase in an asset results in a gain. The more traditional view, which is largely the basis used in IAS 18 Revenue, is that (net) revenue recognition is part of a transactions based accruals or matching process with the balance sheet recording any residual assets or liabilities such as receivables and payables. The issue of revenue recognition arises out of the need to report company performance for specific periods. The Framework identifies three stages in the recognition of assets (and liabilities): initial recognition, when an item first meets the definition of an asset; subsequent remeasurement, which may involve changing the value (with a corresponding effect on income) of a recognised item; and possible derecognition, where an item no longer meets the definition of an asset. For many simple transactions both the Frameworks approach and the traditional approach (IAS 18) will result in the same profit (net income). If an item of inventory is bought for $100 and sold for $150, net assets have increased by $50 and the increase would be reported as a profit. The same figure would be reported under the traditional transactions based reporting (sales of $150 less cost of sales of $100). However, in more complex areas the two approaches can produce different results. An example of this would be deferred income. If a company received a fee for a 12 month tuition course in advance, IAS 18 would treat this as deferred income (on the balance sheet) and release it to income as the tuition is provided and matched with the cost of providing the tuition. Thus the profit would be spread (accrued) over the period of the course. If an asset/liability approach were taken, then the only liability the company would have after the receipt of the fee would be for the cost of providing the course. If only this liability is recognised in the balance sheet, the whole of the profit on the course would be recognised on receipt of the income. This is not a prudent approach and has led to criticism of the Framework for this very reason. Arguably the treatment of government grants under IAS 20 (as deferred income) does not comply with the Framework as deferred income does not meet the definition of a liability. Other standards that may be in conflict with the Framework are the use of the accretion approach in IAS 11 Construction Contracts and a deferred tax liability in IAS 12 Income Tax may not fully meet the Frameworks definition of a liability.

19

The principle of substance over form should also be applied to revenue recognition. An example of where this can impact on reporting practice is on sale and repurchase agreements. Companies sometimes sell assets to another company with the right to buy them back on predetermined terms that will almost certainly mean that they will be repurchased in the future. In substance this type of arrangement is a secured loan and the sale should not be treated as revenue. A less controversial area of the application of substance in relation to revenue recognition is with agency sales. IAS 18 says, where a company sells goods acting as an agent, those sales should not be treated as sales of the agent, instead only the commission from the sales is income of the agent. Recently several Internet companies have been accused of boosting their revenue figures by treating agency sales as their own. (b) Sales made by Derringdo of goods from Gungho must be treated under two separate categories. Sales of the A grade goods are made by Derringdo acting as an agent of Gungho. For these sales Derringdo must only record in income the amount of commission (125%) it is entitled to under the sales agreement. There may also be a receivable or payable for Gungho in the balance sheet. Sales of the B grade goods are made by Derringdo acting as a principal, not an agent. Thus they will be included in sales with their cost included in cost of sales. Sales revenue (4,600 (w (i)) + 11,400 w (ii)) Cost of sales (w (ii)) Gross profit Workings: (all figures in $000) (i) Opening inventory Transfers/purchases Closing inventory Cost of sales Selling price (to give 50% gross profit) Gross profit Commission (125% x 36,800) (ii) Opening inventory Transfers/purchases Closing inventory Cost of sales Selling price (8,550 x 4/3 see below) $000 16,000 (8,550) 7,450 A grade 2,400 18,000 20,400 (2,000) 18,400 36,800 18,400 4,600 B grade 1,000 8,800 9,800 (1,250) 8,550 11,400

A gross profit margin of 25% is equivalent to a mark up on cost of 1/3. Thus if cost of sale is multiplied by 4/3 this will give the relevant selling price. (c) (i) The IASBs Framework defines liabilities as obligations to transfer economic benefits as a result of past transactions. Such transfers of economic benefits are to third parties and normally as cash payments. Traditionally and in compliance with IAS 20 Accounting for Government Grants and Disclosure of Government Assistance, capital based government grants are treated as deferred credits and spread over the life of the related assets. This is the application of the matching concept. A strict interpretation of the Framework would not normally allow deferred credits to be treated as liabilities as there is usually no obligation to transfer economic benefits. In this particular example the only liability that may occur in respect of the grant would be if Derringdo were to sell the related asset within four years of its purchase. A possible argument would be that the grant should be treated as a reducing liability (in relation to a potential repayment) over the four-year claw back period. On closer consideration this would not be appropriate. The repayment would only occur if the asset were sold, thus it is potentially a contingent liability. As Derringdo has no intention to sell the asset there is no reason to believe that the repayment will occur, thus it is not a reportable contingent liability. The implication of this is that the companys policy for the government grant does not comply with the definition of a liability in the Framework. Applying the guidance in the Framework would require the whole of the grant to be included in income as it is earned i.e. in the year of receipt.

20

(ii)

Treatment under the companys policy Income statement extract year to 31 March 2003 Depreciation plant ((800,000 120,000 estimated residual value)/10 years x 6/12) Government grant ((800,000 x 30%)/10 years x 6/12) Balance sheet extracts as at 31 March 2003 Non-current assets: Plant at cost Accumulated depreciation

$ Dr 34,000 Cr 12,000

800,000 (34,000) 766,000 24,000 204,000

Current liabilities: Government grant (240,000/10 years) Non-current liabilities: Government grant (240,000 12,000 24,000) Treatment under the Framework Income statement extract year to 31 March 2003 Depreciation plant ((800,000 120,000 estimated residual value)/10 years x 6/12) Government grant (whole amount) Balance sheet extracts as at 31 March 2003 Non-current assets: Plant at cost Accumulated depreciation

Dr 34,000 Cr 240,000

800,000 (34,000) 766,000

(d)

On first impression, it appears that the company has changed its accounting policy from recognising carpet sales at the point of fitting to recognising them at the point when they are ordered and paid for. If this were the case then the new accounting policy should be applied as if it had always been in place and the income recognised in the year to 31 March 2003 would be $23 million. Without the change in policy, sales would have been $226 million (23m + 12m 16m). Sales made from the retail premises during the current year, but not yet fitted ($16 million) will not be recognised until the following period. A corresponding adjustment is made recognising the equivalent figure ($12 million) from the previous year. The difference between the $23 million and $226 million would be a prior year adjustment (less the cost of sales relating to this amount). This analysis assumes that the figures are material. Despite first impressions, the above is not a change of accounting policy. This is because a change of accounting policy only occurs where the same circumstances are treated differently. In this case there are different circumstances. Derringdo has changed its method of trading; it is no longer responsible for any errors that may occur during the fitting of the carpets. An accounting policy that is applied to circumstances that differ from previous circumstances is not a change of accounting policy. Thus the amount to be recognised in income for the year to 31 March 2003 would be $242 million (23m + 12m). Whilst this appears to boost the current years income it would be mitigated by the payments to the sub-contractors for the carpet fitting.

21

(a)

Rytetrend Cash Flow Statement for the year to 31 March 2003: Cash flows from operating activities Note: figures in brackets are in $000 Operating profit per question Capitalisation of installation costs less depreciation (300 60) (w (i)) Adjustments for: depreciation of non-current assets (w (i)) loss on disposal of plant (w (i)) increase in warranty provision (500 150) decrease in inventory (3,270 2,650) decrease in receivables (1,950 1,100) increase in payables (2,850 1,980) Cash generated from operations Interest paid Income taxes paid (w (ii)) Net cash from operating activities Cash flows from investing activities (w (i)) Cash flows from financing activity: Issue of ordinary shares (1,500 + 1,500) Issue of 6% loan note Repayment of 10% loan notes Ordinary dividends paid (280 + (600 450) interim) Net decrease in cash and cash equivalents Cash and cash equivalents at beginning of period Cash and cash equivalents at end of period Workings (i) Non-current assets cost Balance b/f Disposal Balance c/f (37,250 + 300 re installation) Cost of assets acquired Trade in allowance Cash flow for acquisitions Depreciation Balance b/f Disposal (6,000 x 20% x 4 years) Balance c/f (12,750 + (300 x 20%)) Difference charge for year Disposal Cost Depreciation Net book value Trade in allowance Loss on sale (ii) Income tax paid: Provision b/f Income statement tax charge Provision c/f Difference cash paid $000 27,500 (6,000) (37,550) (16,050) 500 (15,550) (10,200) 4,800 12,810 7,410 6,000 (4,800) 1,200 (500) 700 (630) (1,000) 720 (910) $000 $000 3,860 240 7,410 700

8,110 350 620 850 870 14,900 (460) (910) 13,530 (15,550) (2,020)

3,000 2,000 (4,000) (430)

570 (1,450) 400 (1,050)

22

(b)

Report on the financial performance of Rytetrend for the year ended 31 March 2003 To: From: Date: Operating performance (i) (ii) revenue up $83 million representing an increase of 35% on 2002 figures. costs of sales up by $65 million (40% increase on 2002)

Overall the increase in activity has led to an increase in gross profit of $18 million, however the gross profit margin has eased slightly from 319% in 2002 to 292% in 2003. Perhaps the slight reduction in margins gave a boost to sales. (iii) operating expenses have increased by $600,000 (($5,440,000 $240,000) $4,600,000), an increase of 13% on 2002 figures. (iv) interest costs reduced by $40,000. It is worth noting that the composition of them has changed. It appears that Rytetrend has taken advantage of a cyclic reduction in borrowing cost and redeemed its 10% loan notes and (partly) replaced these with lower cost 6% loan notes. From the interest cost figure, this appears to have taken place half way through the year. Although borrowing costs on long-term finance have decreased, other factors have led to a substantial overdraft which has led to further interest of $200,000. (v) The accumulated effect is an increase in profit before tax of $124 million (up 517% on 2002) which is reflected by an increase in dividends of $200,000.

(vi) The company has invested heavily in acquiring new non-current assets (over $15 million see cash flow statement). The refurbishment of the equipment may be responsible for the increase in the companys sales and operating performance. Analysis of financial position (vii) Inventory and receivables have both decreased markedly. Inventory is now at 43 days from 75 days, this may be due to new arrangements with suppliers or that the different range of equipment that Rytetrend now sells may offer less choice requiring lower inventory. Receivables are only 13 days (from 30 days). This low figure is probably a reflection of a retailing business and the fall from the 2002 figure may mark a reduction in sales made by credit cards. (viii) Although trade payables have increased significantly, they still represent only 46 days (based on cost of sales) which is almost the same as in 2002. (ix) A very worrying factor is that the company has gone from net current assets of $2,580,000 to net current liabilities of $1,820,000. This is mainly due to a combination of the above mentioned items: decreased inventory and receivables and increased trade payables leading to a fall in cash balances of $1,450,000. That said, traditionally acceptable norms for liquidity ratios are not really appropriate to a mainly retailing business. (x) Long-term borrowing has fallen by $2 million; this has lowered gearing from 20% (4,000,000/19,880,000) to only 9% (2,000,000/22,680,00). This is a very modest level of gearing.

The cash flow statement This indicates very healthy cash flows generated from operations of $14,900,000, more than sufficient to pay interest costs, taxation and dividends. The main reason why the overall cash balance has fallen is that new non-current assets (costing over $15 million) have largely been financed from operating cash flows (only $1 million net of new capital has been raised). If Rytetrend continues to generate operating cash flows in the order of the current year, its liquidity will soon get back to healthy levels. Note: The above analysis takes into account the net effect of capitalising the staff costs. 5 (a) (i) The most common problem associated with the use of historical cost financial statements is that they can be misleading and can distort comparisons with other companies financial statements. The problems relate to both the income statement and the balance sheet. In times of rising prices historical values in the balance sheet can quickly become irrelevant. Where current values are significantly higher than carrying values the following problems can be identified: (i) The value of the net assets is equal to the capital employed of an entity. Many important accounting ratios are based on these figures e.g. the return on capital employed, asset utilisation ratios etc. The usefulness of such ratios is limited if the net assets/capital employed are understated. Due to the above, inter company comparison of performance can be invalidated. This point is not immediately obvious as many people may consider that as long as historical cost is compared with historical cost then the comparison is being made on a consistent basis. This is not the case. Historical cost can be thought of as mixed values in that historical costs may represent out of date values (when particular assets were acquired many years ago), but in other cases they may represent current values (when assets have been acquired recently). Thus equivalent assets may be shown at considerably different values.

(ii)

(iii) Understatement of asset values tends to overstate gearing (another important ratio), and leads to a low asset per share value and can make the company vulnerable to a take over.

23

(iv) Where assets, particularly land and buildings, are being used as security to raise finance, it is the current value that lenders are interested in, not historical values. The problems historical costs cause in the income statement are mainly due to certain costs being understated in terms of their current cost. Most notable of these are depreciation charges being based on out of date values of their respective assets and the cost of sales figure may be understated as it does not include current values for bought in items or manufactured goods. The understatement of operating costs leads to an overstatement of profit. There is a possibility that this may lead to higher taxation, wage demands and dividend expectation (based on overstated earnings per share). The combination of these effects is that a company may overspend or over distribute its profits and not maintain its capital base. The above problems affect all users not just shareholders. Managers may not be allocating scarce resources in an optimal way, lenders can be misled (as above), arguably the government is not levying taxes on a fair basis, employees may not be aware of the wealth (or added value) that they are generating, and analysts and advisors encounter comparability problems. (ii) Non-current assets as at 31 March 2003 historical cost $ 250,000 (122,000) 128,000 current purchasing power $ 300,000 (146,400) 153,600 38,400 current cost $ 280,000 (136,640) 143,360 35,840

Cost/valuation Accumulated depreciation Carrying value

Income statement extract year to 31 March 2003 Depreciation of plant 32,000 Workings: Historical cost:

Depreciation at 20% on $250,000 on the reducing balance method would give depreciation of $50,000 in the year to 31 March 2001; $40,000 ((250,000 50,000) x 20%) in the year to 31 March 2002; and $32,000 ((250,000 50,000 40,000) x 20%) in the year to 31 March 2003. The accumulated depreciation at 31 March 2003 is $122,000 (50,000 + 40,000 + 32,000). Current purchasing power: The historical cost of the asset will be remeasured using the increase in the RPI. Thus the CPP cost will be: $250,000 x 216/180 = $300,000 The remaining figures are calculated as per the historical cost method only using $300,000 as the cost. Current (replacement) cost: Although the list price of the new model is $320,000, the two are not directly comparable as the new model is more efficient than the old. A sensible approach would be to reduce the cost of the new model in proportion to its increased efficiency. Equivalent cost $320,000 x 420/480 = $280,000. The remaining figures are calculated on a similar basis to the historical cost figures. (b) (i) The ordinary dividends for the year to 31 March 2003 are: Interim (12 million (3 million x $1/25c) x 3c) Final (12 million x 12 x 6c) $ 360,000 864,000 1,224,000 2,475,000 202 times

Earnings attributable to ordinary shares (see (ii)) Dividend cover (2,475,000/1,224,000)

The dividend cover is the number of times the current years ordinary dividends could have been paid out of the current years profit attributable to ordinary shareholders. It is an indication of the companys dividend policy i.e. a company having a dividend cover of three has paid out one third of its profit as dividends. In terms of maintainable dividends, the dividend cover is a basic measure of risk, the higher the dividend cover the less is the risk that dividends would be reduced if profits suffer a downturn. Conversely, a low dividend cover means that future dividends are more vulnerable to a deterioration in profit. A dividend cover of less than one means the company has used previous years retained earnings to pay the current years dividend. This is not a good sign and is not sustainable in the long term.

24

(ii)

All items in arriving at the profit for the financial year are included in the calculation of the earnings per share. Earnings attributable to the ordinary shares are after the payment of preference dividends: 8% on $1 million for full year new issue 6% on $1 million for six months $ 80,000 30,000 110,000

Earnings attributable to ordinary shares (2,585,000 110,000) 2,475,000 Weighted average number of shares in issue: Calculation of theoretical ex-rights price: 100 shares at $240 would be worth 240 rights to 20 shares at $150 each costing 30 120 shares now worth 270 This gives a theoretical ex-rights value of $225 per share ($270/120) Weighted average calculation: 12,000,000 x $24/$225 x 3/12 3,200,000 14,400,000 (12 million x 12) x 9/12 10,800,000 Weighted average number 14,000,000 Earnings per share is 177c ($2,475,000/14,000,000 x 100) Restated earnings per share for the year to 31 March 2002 is 225c (24 x 225/240) (iii) Fully diluted earnings per share On conversion the loan stock would create an extra 800,000 new shares ($2 million x 40/$100) The effect on earnings would be a saving of interest of $140,000 ($2 million x 7%) before tax and $98,000 after tax (140,000 x (100% 30%)) The directors warrants would create an additional 750,000 new shares without any effect on earnings. Fully diluted earnings per share is 165c ((2,475,000 + 98,000)/(14,000,000 + 800,000 + 750,000)) The basic earnings per share is a measure of past performance. The diluted earnings per share figure is more forward looking and is intended to act as a warning to existing and prospective shareholders. Although it is still based on past performance, it does give effect to potential ordinary shares outstanding during the period. Its disclosure is required where circumstances exist that would cause the eps to be lower if those circumstances had crystallised. It is not a prediction of the future earnings per share figures, as these will be based on the future profits and the number of shares in issue in the future. The diluted eps is more a theoretical value, as it is unlikely that the profit in the period when the circumstances crystallise will be the same as the current years profit. The convertible loan stock in the question is a good example of diluting circumstance. On conversion the share entitlement will cause the number of shares in issue in the future to be greater than the present (assuming loan stockholders opt for conversion). There will be a compensating increase in profit as a result of the non-payment of interest but overall the expected conversion will cause a dilution.

25

Part 2 Examination Paper 2.5(INT) Financial Reporting (International Stream)

June 2003 Marking Scheme

This marking scheme is given as a guide in the context of the suggested answers. Scope is given to markers to award marks for alternative approaches to a question, including relevant comment, and where well-reasoned conclusions are provided. This is particularly the case for written answers where there may be more than one acceptable solution. 1 (a) Income statement: sales revenue cost of sales operating expenses including goodwill loan interest tax minority interest accumulated profit b/f Balance sheet: goodwill tangible non-current assets current assets accumulated profits minority interest 10% loan notes current liabilities available maximum (b) I mark per relevant point to maximum Maximum for question marks 2 3 2 1 1 1 1 3 2 2 1 2 1 2 24 20 5 25

(a)

Income statement sales revenue cost of sales operating costs finance costs taxation available maximum

1 3 1 2 2 9 7

(b)

Changes in equity unrealised gains and losses 1 each prior year adjustment dividends maximum Balance sheet software property, plant and equipment investments inventory trade receivables and payables in substance loan share capital revaluation reserve accumulated profits non-current liabilities accrued finance costs income tax provision proposed dividends available maximum Maximum for question

2 1 2 5 1 3 1 1 1 1 1 2 1 2 1 1 1 17 13 25

(c)

27

(a) (b)

one mark per valid point to max A grade treated as agency sales; B grade as own sales cost of sales for A and B grade 1 mark each commission from A sales sales figure for B grade available maximum

marks 6 2 2 1 1 6 5 1 1 1 3 1 2 1 1 1 1 7 6 2 1 1 1 1 6 5 25

(c)

(i)

deferred credits not liabilities consider claw back as a contingent liability take grant to income in year of receipt maximum

(ii)

depreciation of plant government grant to income, 1 mark under each treatment non-current asset figure company policy: current liabilities non-current liabilities no liability under Framework available maximum

(d)

discussion on change of accounting policy sales figure if treated as a change of policy prior period adjustment not a change of policy sales figure should be $242 million available maximum Maximum for question

(a)

operating profit depreciation adjustment loss on sale of plant warranty provision working capital changes interest paid tax paid purchase of non-current assets share issue; issue/redemption of loan ordinary dividends decrease in cash available maximum

1 1 1 1 2 1 1 2 2 1 1 14 12 5 9 14 13 25

(b)

relevant ratios appropriate comments

available available maximum Maximum for question

28

(a)

(i) (ii)

one mark per valid point to historic cost figures CPP figures CCA figures

maximum

marks 6 1 2 3 6 1 1 2 4 3 1 1 1 1 1 5 4 1 1 1 1 3 7 6 25

maximum (b) (i) calculation of ordinary dividends calculation of dividend cover significance available maximum (ii) preference dividends deduction ex-rights price weighted number of shares calculation of eps restatement of comparative available maximum (iii) shares on conversion option shares effect on earnings calculation comments on significance up to available maximum Maximum for question

29

Financial Reporting
(International Stream)
PART 2 THURSDAY 9 DECEMBER 2004

QUESTION PAPER Time allowed 3 hours This paper is divided into two sections Section A This ONE question is compulsory and MUST be answered THREE questions ONLY to be answered

Section B

Do not open this paper until instructed by the supervisor This question paper must not be removed from the examination hall

The Association of Chartered Certified Accountants

Paper 2.5(INT)

Section A This ONE question is compulsory and MUST be attempted 1 Holdrite purchased 75% of the issued share capital of Staybrite and 40% of the issued share capital of Allbrite on 1 April 2004. Details of the purchase consideration given at the date of purchase are: Staybrite: a share exchange of 2 shares in Holdrite for every 3 shares in Staybrite plus an issue to the shareholders of Staybrite 8% loan notes redeemable at par on 30 June 2006 on the basis of $100 loan note for every 250 shares held in Staybrite. Allbrite: a share exchange of 3 shares in Holdrite for every 4 shares in Allbrite plus $1 per share acquired in cash. The market price of Holdrites shares at 1 April 2004 was $6 per share. The summarised income statements for the three companies for the year to 30 September 2004 are: Holdrite $000 75,000 (47,400) 27,600 (10,480) 17,120 (170) 16,950 (4,800) 12,150 Staybrite $000 40,700 (19,700) 21,000 (9,000) 12,000 12,000 (3,000) 9,000 Allbrite $000 31,000 (15,300) 15,700 (9,700) 6,000 6,000 (2,000) 4,000

Revenue Cost of Sales Gross Profit Operating expenses Operating Profit Interest expense Profit before tax Income tax expense Profit for period

The following information is relevant: (i) A fair value exercise was carried out for Staybrite at the date of its acquisition with the following results: Book Value Fair Value $000 $000 Land 20,000 23,000 Plant 25,000 30,000 The fair values have not been reflected in Staybrites financial statements. The increase in the fair value of the plant would create additional depreciation of $500,000 in the post acquisition period in the consolidated financial statements to 30 September 2004. Depreciation on plant is charged to cost of sales. (ii) The details of each companys share capital and reserves at 1 October 2003 are: Holdrite Staybrite $000 $000 Equity shares of $1 each 20,000 10,000 Share premium 5,000 4,000 Retained profits 18,000 7,500 Allbrite $000 5,000 2,000 6,000

(iii) In the post acquisition period Holdrite sold goods to Staybrite for $10 million. Holdrite made a profit of $4 million on these sales. One-quarter of these goods were still in the inventory of Staybrite at 30 September 2004. (iv) Impairment tests on the goodwill of Staybrite and Allbrite at 30 September 2004 resulted in the need to write down Staybrites goodwill by $750,000. Allbrites goodwill was not impaired. (v) Holdrite paid a dividend of $5 million on 20 September 2004.

Required: (a) Calculate the goodwill arising on the purchase of the shares in both Staybrite and Allbrite at 1 April 2004. (8 marks) (b) Prepare a consolidated income statement for the Holdrite Group for the year to 30 September 2004. (15 marks) (c) Show the movement on the consolidated retained profits attributable to Holdrite for the year to 30 September 2004. (2 marks) (25 marks)

Note: The additional disclosures in IFRS 3 Business Combinations relating to a newly acquired subsidary are not required.

[P.T.O.

Section B THREE questions ONLY to be attempted 2 The following trial balance relates to Chamberlain, a publicly listed company, at 30 September 2004: $000 Ordinary share capital Retained profits at 1 October 2003 6% Loan note (issued in 2002) Deferred tax (note (v)) Land and buildings at cost (land element $163 million (note (i))) Plant and equipment at cost (note (i)) Plant on lease to customer at cost (note (ii)) Accumulated depreciation 1 October 2003 buildings Accumulated of depreciation 1 October 20 plant and equipment Trade receivables Inventory 1 October 2003 Bank Trade payables Revenue Purchases Construction contract balance (note (iii)) Operating expenses Loan interest paid Interim dividend Rental income from plant (note (ii)) Research and development expenditure (note (iv)) $000 200,000 162,000 50,000 17,500

403,000 124,000 56,000 60,000 44,000 48,000 35,500 12,500 45,000 246,500 78,500 5,000 29,000 1,500 8,000 16,000 40,000 841,000 841,000

The following notes are relevant: (i) The building had an estimated life of 40 years when it was acquired and is being depreciated on a straight-line basis. Plant and equipment, other than the leased plant, is depreciated at 125% per annum using the reducing balance basis. Depreciation of buildings and plant and equipment is charged to cost of sales.

(ii) On 1 October 2003 Chamberlain purchased an item of plant for $56 million which it leased to a customer on the same date. The lease period is four years with annual rentals of $16 million in advance. The plant is expected to have a nil residual value at the end of the four years. Chamberlain has been advised that this is a finance lease with an interest rate of 10% per annum. (iii) The construction contract balance represents costs incurred to date of $35 million less progress billings received of $30 million on a two year construction contract that commenced on 1 October 2003. The total contract price has been agreed at $125 million and Chamberlain expects the total contract cost to be $75 million. The company policy is to accrue for profit on uncompleted contracts by applying the percentage of completion to the total estimated profit. The percentage of completion is determined by the proportion of the contract costs to date compared to the total estimated contract costs. At 30 September 2004, $5 million of the $35 million costs incurred to date related to unused inventory of materials on site. Other inventory at 30 September 2004 amounted to $385 million at cost (iv) The research and development expenditure is made up of $25 million of research, the remainder being development expenditure. The directors are confident of the success of this project which is likely to be completed in March 2005. (v) The directors have estimated the provision for income tax for the year to 30 September 2004 at $22 million. The deferred tax provision at 30 September 2004 is to be adjusted to a credit balance of $14 million.

Required: Prepare for Chamberlain: (a) An income statement for the year to 30 September 2004; and (11 marks)

(b) A balance sheet as at 30 September 2004 in accordance with International Financial Reporting Standards as far as the information permits. (14 marks) Note: A Statement of Changes in Equity is NOT required. Disclosure notes are ONLY required for the leased plant in item (ii) above. (25 marks)

Historically financial reporting throughout the world has differed widely. The International Accounting Standards Committee Foundation (IASCF) is committed to developing, in the public interest, a single set of high quality, understandable and enforceable global accounting standards that require transparent and comparable information in general purpose financial statements. The various pronouncements of the IASCF are sometimes collectively referred to as International Financial Reporting Standards (IFRS) GAAP. Required: (a) Describe the functions of the various internal bodies of the IASCF, and how the IASCF interrelates with other national standard setters. (10 marks) (b) Describe the IASCFs standard setting process including how standards are produced, enforced and occasionally supplemented. (10 marks) (c) Comment on whether you feel the move to date towards global accounting standards has been successful. (5 marks) (25 marks)

[P.T.O.

Bigwood, a public company, is a high street retailer that sells clothing and food. The managing director is very disappointed with the current years results. The company expanded its operations and commissioned a famous designer to restyle its clothing products. This has led to increased sales in both retail lines, yet overall profits are down. Details of the financial statements for the two years to 30 September 2004 are shown below. Income statements: Revenue clothing Sale s food Cost of sales clothing Cost of sales food Gross profit Other operating expenses Operating profit Interest expense Profit before tax Income tax expense Profit for period Summarised Changes in Equity: Retained profit b/f Profit for the period Dividends paid Retained profit c/f Balance sheets as at: Property, plant and equipment at cost Accumulated depreciation Current Assets Inventory clothing Inventory food Trade receivables Bank Total assets Equity and liabilities Issued ordinary capital ($1 shares) Share premium Retained profits Non-current liabilities Long-term loans Current liabilities Bank overdraft Trade payables Current tax payable year to 30 September 2004 $000 $000 16,000 7,000 23,000 14,500 4,750 (19,250) 3,750 (2,750) 1,000 (300) 700 (250) 450 year to 30 September 2004 $000 1,900 450 (600) 1,750 30 September 2004 $000 $000 17,000 (5,000) 12,000 2,700 200 100 nil year to 30 September 2003 $000 $000 15,600 4,000 19,600 12,700 3,000 (15,700) 3,900 (1,900) 2,000 (80) 1,920 (520) 1,400 year to 30 September 2003 $000 1,100 1,400 (600) 1,900 30 September 2003 $000 $000 9,500 (3,000) 6,500 1,360 140 50 450 -

3,000 15,000 5,000 1,000 1,750 7,750 3,000

2,000 8,500 3,000 nil 1,900 4,900 1,000

4,250 2,600 15,000 8,500 Note: the directors have signalled their intention to maintain annual dividends at $600,000 for the foreseeable future. 6

930 3,100 220

nil 2,150 450

The following information is relevant: (i) The increase in property, plant and equipment was due to the acquisition of five new stores and the refurbishment of some existing stores during the year. The carrying value of fixtures scrapped at the refurbished stores was $1.2 million; they had originally cost $3 million. Bigwood received no scrap proceeds from the fixtures, but did incur costs of $50,000 to remove and dispose of them. The losses on the refurbishment have been charged to operating expenses. Depreciation is charged to cost of sales apportioned in relation to floor area (see below). (ii) The floor sales areas (in square metres) were: Clothing Food 30 September 2004 48,000 16,000 54,000 30 September 2003 35,000 15,000 40,000

(iii) The share price of Bigwood averaged $600 during the year to 30 September 2003, but was only $300 at 30 September 2004. (iv) The following ratios have been calculated: Return on capital employed Net assets turnover Gross profit margin clothing food Net profit (after tax) margin Current ratio Inventory holding period clothing food Accounts payable period Gearing Interest cover Required: (a) Prepare, using the indirect method, a cash flow statement for Bigwood for the year to 30 September 2004 (12 marks) (b) Write a report analysing the financial performance and financial position of Bigwood for the two years ended 30 September 2004. (13 marks) Your report should utilise the above ratios and the information in your cash flow statement. It should refer to the relative performance of the clothing and food sales and be supported by any further ratios you consider appropriate. (25 marks) 2004 93% 21 times 94% 321% 20% 071:1 68 days 15 days 59 days 28% 33 times 2003 339% 33 times 186% 25% 71% 077:1 39 days 17 days 50 days 17% 25 times

[P.T.O.

(a) Derwent, a publicly listed company, made the following share capital transactions on 1 October 2003. (i) the purchase and immediate cancellation of five million of its own ordinary shares. Derwent paid $175 per share in an open market purchase. These shares had originally been issued at par. Derwent is registered in a country that permits a company to purchase and immediately cancel its own share capital. In order to protect creditors the nominal value of the redeemed share capital must be replaced by a new issue of shares or by a transfer from accumulated profits to a capital reserve. Any premium paid on the redemption of shares must be charged to retained profits. (ii) an issue of one million 5% preference shares of $1 each at par. This issue was made in order to help finance the above purchase. Derwents share capital and reserves at 30 September 2003 were: $000 Ordinary shares of 25c each fully paid 25,000 Retained profits 55,000 80,000 On 1 January 2004 Derwent paid an interim dividend of 3c per share and on 1 June 2004 it paid a further dividend of 5c per share. Derwents profit after tax for the year to 30 September 2004 was $12 million. Required: (i) Prepare extracts from Derwents balance sheet for share capital and 30 September 2004; calculate the dividends paid for the year to 30 September 2004. reserves at (8 marks) (4 marks)

(ii) State the advantages of companies being able to purchase and then cancel their own shares.

(b) Advent is a publicly listed company. Details of Advents non-current assets at 1 October 2003 were: Land and building $m 280 1(40) 240 Plant $m 150 (105) 45 Telecommunications license $m 300 1(30) 270 Total $m 730 (175) 555

Cost/valuation Accumulated depreciation/amortisation Net book value

The following information is relevant: (i) The land and building were revalued on 1 October 1998 with $80 million attributable to the land and $200 million to the building. At that date the estimated remaining life of the building was 25 years. A further revaluation was not needed until 1 October 2003 when the land and building were valued at $85 million and $180 million respectively. The remaining estimated life of the building at this date was 20 years. (ii) Plant is depreciated at 20% per annum on cost with time apportionment where appropriate. On 1 April 2004 new plant costing $45 million was acquired. In addition, this plant cost $5 million to install and commission. No plant is more than four years old. (iii) The telecommunications license was bought from the government on 1 October 2002 and has a 10 year life. It is amortised on a straight line basis. In September 2004, a review of the sales of the products related to the license showed them to be very disappointing. As a result of this review the estimated recoverable amount of the license at 30 September 2004 was estimated at only $100 million. There were no disposals of non-current assets during the year to 30 September 2004 Required: (i) Prepare balance sheet extracts of Advents non-current assets as at 30 September 2004 (including comparative figures), together with any disclosures required (other than those of the accounting policies) under current International Financial Reporting Standards. (9 marks) (4 marks) (25 marks)

(ii) Explain the usefulness of the above disclosures to the users of the financial statements.

End of Question Paper

Answers

Part 2 Examination Paper 2.5 (INT) Financial Reporting (International Stream) 1 (a) Cost of control in Staybrite: Consideration Shares (10,000 x 75% x 2/3 x $6) 8% loan notes (10,000 x 75% x $100/250) Less Equity shares Share premium Pre acq reserves (see below) Fair value adjustment (5,000 + 3,000)

December 2004 Answers

$000

$000 30,000 3,000 33,000

10,000 4,000 12,000 8,000 34,000

x 75%

Goodwill The pre acquisition reserves are: At 1 October 2003 To 1 April 2004 (9,000 x 6/12)

(25,500) 7,500 7,500 4,500 12,000

Goodwill on the purchase of Allbrite: Consideration Shares (5,000 x 40% x 3/4 x $6) Cash (5,000 x 40% x $1) Less Equity shares Share premium Pre acq reserves (6,000 + (4,000 x 6/12))

9,000 2,000 11,000 5,000 2,000 8,000 15,000

x 40%

Goodwill (b) Holdrite Group Consolidated income statement for the year ended 30 September 2004 $000 Revenue (75,000 + (40,700 x 6/12) 10,000) Cost of sales (w (i)) Gross profit Operating expenses (w (ii)) Profit from operations Income from associate (w (iii)) Interest expense Profit before tax Income tax expense Group (4,800 + (3,000 x 6/12)) Income tax expense Associate (w (iii)) Profit for the period Attributable to: Equity holders of the parent Minority interest (w (iv)) (6,300) (400)

(6,000) 5,000

$000 85,350 (48,750) 36,600 (15,730) 20,870 1,200 22,070 (170) 21,900 (6,700) 15,200 14,200 1,000 15,200

13

(c)

Net profit for period Dividend paid Retained profit b/f (Holdrite only) Retained profits c/f Workings ($000) (i) Cost of sales Holdrite Staybrite (19,700 x 6/12) Additional depreciation of plant Intra group purchases Unrealised profit in inventory (4,000 x 25%)

$000 14,200 (5,000) 9,200 18,000 27,200 $000 47,400 9,850 500 (10,000) 1,000 48,750

(ii)

Operating expenses Holdrite Staybrite (9,000 x 6/12) Impairment of Staybrites goodwill

10,480 4,500 750 15,730 1,200 400

(iii) Associated company Profit for the year (6,000 x 6/12 x 40%) Taxation (2,000 x 6/12 x 40%) (iv) Minority Interest 9,000 x 6/12 less additional depreciation

x 25%

4,500 (500) 4,000 1,000

(a)

Chamberlain Income statement Year to 30 September 2004 Revenue (246,500 + 50,000 (w (i))) Cost of sales (w (ii)) Gross profit Operating expenses Profit before interest and tax Investment income (w (iii)) Interest expense (1,500 + 1,500 accrued) Profit before tax Income tax (22,000 (17,500 14,000)) Profit for the period $000 296,500 (146,500) 150,000 (29,000) 121,000 4,000 (3,000) 122,000 (18,500) 103,500

14

(b)

Chamberlain Balance Sheet as at 30 September 2004 Non-current assets Property, plant and equipment (w (iv)) Development costs (40,000 25,000) Current assets Inventory Amounts due from construction contracts (w (i)) Trade receivables Net investment in finance lease Note 1 (w (iii)) Accrued finance lease income (w (iii)) Bank Total assets Equity and liabilities Capital and reserves: Ordinary share capital Retained profits 1 October 2003 Accumulated pr Year to 30 September 2004 Accumulated pr less dividends paid

$000

$000 407,000 15,000 422,000

38,500 25,000 48,000 40,000 4,000 12,500

168,000 590,000

200,000 162,000 103,500 (8,000)

257,500 457,500 64,000

Non-current liabilities (w (v)) Current liabilities Trade payables Accrued finance costs Taxation Total equity and liabilities Note 1 Net investment in finance lease: Amount receivable within one year Amount receivable after more than one year 45,000 1,500 22,000

68,500 590,000

12,000 28,000 40,000

15

Workings (all figures in $000) (i) Construction contract: Contract price Estimated cost Estimated total profit

$000 125,000 (75,000) 50,000 30,000 75,000 40%

Contract cost for year (35,000 5,000 inventory on site) Estimated cost Percentage complete (30,000/75,000) Year to 30 September 2004 Contract revenue included in sales (125,000 x 40%) Contract costs included in cost of sales (35,000 5,000) Amounts due from customers: Cost to date plus profit taken (35,000 + 20,000) Less progress billings received

50,000 (30,000) 55,000 (30,000) 25,000 35,500 78,500 30,000 25,000 6,000 10,000 (38,500) 146,500 56,000 (16,000) 40,000 4,000

(ii)

Cost of sales: Opening inventory Purchases Contract costs (w (i)) Research costs Depreciation (w (iv)) buildings Depreciation (w (iv)) plant Closing inventory

(iii) Finance lease: Net investment at inception of lease First rental payment 1 October 2003 Investment outstanding to 30 September 2004 Accrued interest 10% (current asset)

In the year to 30 September 2005 the second rental payment of $16,000 will be received, of this $4000 is for the accrued interest for the previous year, thus $12,000 will be a capital receipt. Therefore the analysis of the investment outstanding at 30 September 2004 of $40,000 is that $12,000 will be receivable within one year and $28,000 after more than one year. (iv) Non-current assets/depreciation: Buildings: A cost of $240,000 (403,000 163,000 for the land) over a 40 year life gives annual depreciation of $6,000 per annum. This gives accumulated depreciation at 30 September 2004 of $66,000 (60,000 + 6,000) and a net book value of $337,000 (403,000 66,000). Plant: The carrying value prior to the current years depreciation is $80,000 (124,000 44,000). Depreciation at 125% on the reducing balance basis gives an annual charge of $10,000. This gives a carrying value at 30 September 2004 of $70,000 (80,000 10,000). Therefore the carrying value of property, plant and equipment at 30 September 2004 is $407,000 (337,000 + 70,000). (v) Non-current liabilities 6% loan note Deferred tax 50,000 14,000 64,000

16

Note: The International Accounting Standards Board (IASB) has decided that their standards will be called International Financial Reporting Standards (IFRS), and that this term should be taken to encompass both Standards and Interpretations issued by the IASB (IFRS and IFRIC), and the International Accounting Standards (IAS) and Standing Interpretations Committee Interpretations (SIC) issued by its predecessor standard setter, the IASC Board. References in this answer to IFRS should be taken to have the same meaning as that used by the IASB. (a) In recognition of the increasing importance of international accounting standards, in 1999 the Board of the IASB recommended and subsequently adopted a new constitution and structure. After a two year process a new supervisory body, The International Accounting Standards Committee Foundation, was incorporated in the USA in February 2001 as an independent not-for-profit organisation. It is governed by 19 IASC Foundation Trustees who must have an understanding of international issues relevant to accounting standards for use in the worlds capital markets. The main objectives of the IASC Foundation are: to develop a single set of global accounting standards that require high quality, transparent and comparable information in financial statements to help users in making economic decisions; to promote the use and application of these standards; and to bring about convergence of national accounting standards and international accounting standards The subsidiary bodies of the IASC Foundation are the International Accounting Standards Board (IASB) (based in London UK), the Standards Advisory Council (SAC) and the International Financial Reporting Interpretations Committee (IFRIC). The International Accounting Standards Board The result of a restructuring process saw the IASB assume the responsibility for setting accounting standards from its predecessor body, the International Accounting Standards Committee. The Trustees appoint the members of all of the above bodies. They also set the agenda of and raise finance for the IASB; however the IASB has sole responsibility for setting accounting standards, International Financial Reporting Standards (IFRS), following rigorous and open due process. The Standards Advisory Council provides a forum for experts from different countries and different business sectors with an interest in international financial reporting to offer advice when drawing up new standards. Its main objectives are to give advice to the Trustees and IASB on agenda decisions and work priorities and on the major standard-setting projects. The International Financial Reporting Interpretations Committee has taken over the work of the previous Standing Interpretations Committee. It is really a compliance body whose role is to provide rapid guidance on the application and interpretation of international accounting standards where contentious or divergent interpretations of international accounting standards have arisen. It operates an open due process in accordance with its approved procedures. Its pronouncements (interpretations SICs and IFRICs) are important because financial statements cannot be described as complying with IFRSs unless they also comply with the interpretations. Other Bodies The prominence of the IASB has been enhanced even further by its relationship with the International Organisation of Securities Commissions (IOSCO). IOSCO is an influential organisation of the worlds security commissions (stock exchanges). In 1995 the IASC agreed to develop a core set of standards which, when endorsed by IOSCO, would be used as an acceptable basis for cross-border listings. In May 2000 this was achieved. Thus it can be said that international accounting standards may be the first tentative steps towards global accounting harmonisation. As part of its harmonisation process the European Union will require listed companies in all member states to prepare their financial statements using IFRSs by 2005. National standard setters such as the UKs Accounting Standards Committee and the USAs Financial Accounting Standards Board have a role to play in the formulation of international accounting standards. Seven of the leading national standard setters are members of the IASB. The IASB see this as a partnership between IASB and these national bodies as they work together to achieve the convergence of accounting standards world wide. Often the IASB will ask members of national standard setting bodies to work on particular projects in which those countries have greater experience or expertise. Many countries that are committed to closer integration with IFRSs will publish domestic standards equivalent (sometimes identical) to IFRSs on a concurrent timetable. (b) The International Accounting Standard Setting Process As referred to above the IASB is ultimately responsible for setting international accounting standards. The Board (advised by the SAC) identifies a subject and appoints an Advisory Committee to advise on the issues relevant to the given topic. Depending on the complexity and importance of the subject matter the IASB may develop and publish Discussion Documents for public comment. Following the receipt and review of comments the IASB then develops and publishes an Exposure Draft for public comment. The usual comment period for both of these is ninety days. Finally, and again after a review of any further comments, an International Financial Reporting Standard (IFRS) is issued. The IASB also publishes a Basis for Conclusions which explains how it reached its conclusions and gives information to help users to apply the Standard in practice. In addition to the above the IASB will sometimes conduct Public Hearings where proposed standards are openly discussed and occasionally Field Tests are conducted to ensure that proposals are practical and workable around the world. The authority of international accounting standards is a rather difficult area. The IASB has no power to enforce international accounting standards within those countries/enterprises that choose to adopt them. This means that the enforcement of international accounting standards is in the hands of the regulatory systems of the individual adopting countries. There is no doubt the regulatory systems in different parts of the world differ from each other considerably in their effectiveness. For example in the UK the Financial Reporting Review Panel (FRRP) is a body that investigates departures from the UKs regulatory system (which will soon include the use of international accounting standards for listed companies). The FRRP has wide and effective powers of enforcement, but not all countries have equivalent bodies, thus it can be argued that international

17

accounting standards are not enforced in a consistent manner throughout the world. Complementary to international accounting standards, there also exist international auditing standards and part of the rigour and transparency that the use of international accounting standards brings is due to the fact that those companies adopting international accounting standards should also be audited in accordance with international auditing standards. This auditing aspect is part of IOSCOs requirements for financial statements to be used for cross-border listing purposes. Where it becomes apparent (often through press reports) that there is widespread inconsistency in the interpretation of an international accounting standard, or where it is perceived that a standard is not clear enough in a particular area, the IFRIC may act to remedy/clarify the position thus supplementing the body of international standards. However where it becomes apparent (perhaps through a modified audit report) that a company has departed from IFRSs there is little that the IASB can do directly to enforce them. (c) The success of the process Any measure of success is really a matter of opinion. There is no doubt that the growing acceptance of IFRSs through IOSCOs endorsement, the European Union requirement for their use by listed companies and the ever increasing number of countries that are either adopting international accounting standards outright or basing their domestic standards very closely on IFRSs is a measure of the success of the IASB. Equally there is widespread recognition that in recent years the quality of international accounting standards has improved enormously due to the improvements project and subsequent continuing improvements. However the IASB is not without criticism. Some countries that have developed sophisticated regulatory systems feel that IFRSs are not as rigorous as the local standards and this may give cross-border listing companies an advantage over domestic companies. Some requirements of international accounting standards are regarded as quite controversial, e.g. deferred tax (part of IAS 12), financial instruments and derivatives (IAS 32 and 39) and accounting for retirement benefits (IAS 26). Many IFRSs are complex and the benefits of applying them to smaller enterprises may be outweighed by the costs. Also some securities exchanges that are part of IOSCO require non-domestic companies that are listing by filing financial statements prepared under IFRSs to produce a reconciliation to local GAAP. This involves reconciling the IFRS income statement and balance sheet assets, liabilities and equity, to what they would be if local GAAP had been used. The USA is an important example of this requirement. Critics argue that this requirement negates many of the benefits of being able to use a single set of financial statements to list on different security exchanges. This is because to produce reconciliation to local GAAP is almost as much work and expense as preparing financial statements in the local GAAP which was usually the previous requirement. Despite these criticisms there is no doubt that the work of IASB has already led, and in the future will lead, to further improvement in financial reporting throughout the world.

(a)

Bigwood Cash Flow Statement for the year to 30 September 2004: Note: figures in brackets are in $000 Net profit before tax Adjustments for: depreciation non-current assets (w (i)) loss on disposal of fixtures (w (i)) interest expense Operating profit before working capital changes increase in inventory (2,900 1,500) increase in trade receivables (100 50) increase in trade payables (3,100 2,150) Cash generated from operations Interest paid Income tax paid (w (ii)) Net cash from operating activities Cash flow from investing activities Purchase of Property, plant and equipment (w (i)) Disposal costs of fixtures (w (i)) Cash flows from financing activities Issue of ordinary shares (2,000 + 1,000) Long term loans (3,000 1,000) Equity dividend paid Net decrease in cash and cash equivalents Cash and cash equivalents at beginning of period Cash and cash equivalents at end of period $000 $000 700

3,800 1,250 300

5,350 6,050 (1,400) (50) 950 5,550 (300) (480) 4,770

(10,500) (50)

(10,550) (5,780)

3,000 2,000 (600)

4,400 (1,380) 450 (930)

18

Workings (all figures in $000) (i) Property, plant and equipment cost Balance b/f Disposal Balance c/f Difference cash purchase Depreciation Balance b/f Disposal (3,000 1,200) Balance c/f Difference charge for year Disposal Cost Depreciation Net book value Cost of disposal Total loss on disposal

9,500 (3,000) (17,000) (10,500) (3,000) 1,800 5,000 3,800 3,000 (1,800) 1,200 50 (1,250)

(ii)

Income tax paid: Provision b/f Income statement tax charge Provision c/f Difference cash paid

(450) (250) 220 (480)

(b)

Report on the financial performance of Bigwood for the year ended 30 September 2004 To: From: Date: Operating performance: Bigwoods overall performance as measured by the return on capital employed has deteriorated markedly. This ratio is effectively a composite of the companys profit margins and its asset utilisation. The expansion represented by the acquisition of the five new stores has considerably increased investment in net assets. The asset turnover (a measure of asset utilisation) has fallen from 33 times to just 21 times. This is a relatively large fall and is partly responsible for the deteriorating performance. However, it should be borne in mind that it often takes some time before new investment generates the same level of sales as existing capacity so it may be that the situation will improve in future years. Of more concern in the current year is the deteriorating gross profit margin of the companys clothing sales. This has fallen from 186% to 94%. The effect of this is all the more marked because sales of clothing (in the current year) represents nearly 70% of turnover. It should also be noted that the inventory holding period of clothing has also increased significantly from 39 days in 2003 to 68 days in the current year. This may be a reflection of a company policy to increase inventory levels in order to attract more sales, but it may also be an indication that there is some slow-moving or obsolete inventory. The clothing industry is notoriously susceptible to fashion changes, the new designs may not have gone down well with the buying public. By contrast the profit margin on food sales has increased substantially (from 25% to 321%) as indeed have the sales themselves (up 75% on last year). These improvements have helped to offset the weaker performance of clothing sales. A more detailed analysis shown by the ratios in the appendix confirms the position. The expansion has created a 35% increase in the sales floor area, but the proportionate increase in turnover is only 173%. Breaking this down between the two sectors shows that the clothing sector is responsible for this deterioration; an increase in capacity of 37% has led to an increase in sales of only 26%, whereas a more modest increase of 20% in the food floor area has led to a remarkable increase of 75% in food sales. In the current year food retailing has generated sales of $1,167,000 per square metre, whereas clothing sales per square metre has fallen from $446,000 to $333,000. When the relative profit margins of clothing and food are considered it can be seen that food retailing has been far more profitable than clothing retailing and this gap in margins has increased during the current year. This deterioration in trading margins has continued through to net profit margins (falling from 71% to only 20%). It can be observed that operating expenses have increased considerably, but this is to be expected and is probably in line with the increase in the number of stores.

19

In summary, the increase in capacity has focused on clothing rather than food retailing. On reflection this seems misguided as the performance of food retailing was superior to that of clothing (in 2003) and this has continued (even more so) during the current year. Liquidity/solvency The increase in the investment in new stores and the refurbishment of existing stores has been largely financed by increasing long term loans by $2 million and issuing $3 million of equity. The effect of this is an increase in gearing from 17% to 28%. Although the level of gearing is still modest, the interest cover has fallen from a very healthy 25 times to a worrying low 33 times. The investment has also taken its toll on the bank balance falling from $450,000 in hand to an overdraft of $930,000. This probably explains why the company has stretched its payment of accounts payable to 59 days in 2004 from 50 days in 2003. The companys current liquidity position has deteriorated slightly from 077 : 1 to 071 : 1. No quick ratios have been given, nor would they be useful. Liquidity ratios are difficult to assess for retailing companies. Most of the sales generated by such companies are for cash (thus there will be few trade receivables) and normal liquidity benchmarks are not appropriate. The cash flow statement reveals cash flows generated from operating activities of $5,550,000. This is a far more reliable indicator of the companys liquidity position. The $5,550,000 is more than adequate to service the tax and the dividend payments. Indeed the operating cash flows have contributed significantly to the financing of the expansion programme. Share price and dividends: Bigwoods share price has halved from $600 to $300 during the current year. The dilution effect of the share issue at $150 per share (2 million shares for $3million) would account for some of this fall (to approximately $420), but the further fall probably represents the markets expectations of the companys performance. It is worth noting that the company has maintained its dividends at $600,000 despite an after tax profit of only $450,000. Whilst this dividend policy cannot be maintained indefinitely (at the current level of profits), the directors may be trying to convey to the market a feeling of confidence in the future profitability of the company. It may also be a reaction designed to support the share price. It should also be noted that although the total dividend has been maintained, the dividend per share will have decreased due to the share issue during the year. Summary The above analysis of performance seems to give mixed messages, the company has invested heavily in new and upgraded stores, but operating performance has deteriorated and the expansion may have been mis-focused. This appears to have affected the share price adversely. Alternatively, it may be that the expansion will take a little time to bear fruit and the deterioration may be a reflection of the current state of the economy. Cash generation remains sound and if this continues, the poor current liquidity position will soon be reversed. Signed A N Other Appendix The following additional ratios can be calculated: Increase in sales area Increase in turnover clothing (13,000/35,000) 37% (400/15,600) 26% food (1,000/5,000) 20% (3,000/4,000) 75% sales per sq mtr 2004 $000 (23,000/54) 426 (16,000/48) 333 (7,000/6) 1,167 overall (14,000/40,000) 35% (3,400/19,600) 173% sales per sq mtr 2003 $000 (19,600/40) 490 (15,600/35) 446 (4,000/5) 800

Overall Clothing Food

20

(a)

(i)

Derwent share capital and reserves at 30 September 2004: $000 95 million ordinary shares of 25c each fully paid ((100 million 5 million) x 25c) One million 5% preference shares of $1 each Capital reserve (w (ii)) 250 Retained profits (w (ii)) 51,600 $000 23,750 1,000 51,850 76,600

Dividends year to 30 September 2004 Preference dividend ($1 million x 5%) Ordinary dividends January 2004 ((100 million 5 million) x 3c) June 2004 (95 million x 5c)

50 2,850 4,750

7,600

Workings (i) The premium on redemption is $75 million (5 million x (175c 25c)) and this must be charged to accumulated profits. (ii) Retained profits b/f profit after tax for the year (from question) preference dividend (from above) ordinary dividends (from above) premium on redemption (w (i)) transfer to capital reserve (see below) $000 12,000 (50) (7,600) (7,500) (250) $000 55,000

(3,400) 51,600

The transfer to the capital reserve is the nominal value of share capital redeemed of $125 million (5 million x 25c) less the proceeds of the new issue of $1 million. (ii) Advantages of purchasing (then cancelling) own shares: it is a method of returning excess cash/capital surpluses to shareholders (without paying dividends) if a company believes its shares are undervalued on the stock market, it may be able to improve its share price (and p/e ratio) by buying shares in the open market. The demand created by the purchase may cause an increase in price and, if these shares are cancelled (as they often are), this means the remaining shareholders own a greater proportion of the company. private companies (whose shares have no active market) can take advantage of the company being able to purchase shares. This may be used to buy out shares held by employees (say on retirement) who have received them as part of a profit sharing scheme. Companies may buy out dissenting shareholders, or buy shares from the estate of a deceased shareholder. Companies normally purchase shares in these circumstances when the other shareholders do not wish to purchase any more of the companys shares. Non-current assets Property, plant and equipment (note 1) Intangible assets (note 2) Note 1 Property, plant and equipment Cost or valuation: At 1 October 2003 Additions Revaluation (5 20) At 30 September 2004 Accumulated depreciation: At 1 October 2003 Charge for year Revaluation At 30 September 2004 Carrying value 30 September 2004 Land and building $million 280 1(15) 265 140 119 1(40) 119 256 30 September 2004 $million 316 100 Plant $million 150 150 1nil 200 105 135 1nil 140 160 30 September 2003 $million 285 270 Total $million 430 150 1(15) 465 145 144 1(40) 149 316

(b)

(i)

21

The land and buildings were revalued by an appropriately qualified valuer on an existing use basis on 1 October 2003. They are being depreciated on a straight-line basis over a 25 year life. Plant is depreciated at 20% per annum on cost. Note 2 Intangible fixed assets: Telecommunications license $million Cost at 1 October 2003 300 And at 30 September 2004 300 Accumulated amortisation 1 October 2003 130 Amortisation charge for year 30 Impairment charge for year 140 At 30 September 2004 200 Carrying value 30 September 2004 100 Total $million 300 300 130 30 140 200 100

After the impairment charge the license will be amortised over its remaining life of eight years on a straight-line basis. (ii) The usefulness of the above disclosures is: users can determine which type of non-current assets a business owns. There is a great deal of difference between owning say land and buildings compared with intangibles. The above figures give an illustration of this; the property has increased in value whereas the licence has fallen dramatically. Another factor relevant to this distinction is that it is usually easier to raise finance using property as security, whereas it can be difficult to raise finance on intangibles due to the volatility of their values. it is useful to know whether non-current assets are carried at historical cost or at revalued amount. If a company is using historical cost, it may be that balance sheet values are seriously understated with a consequential effect on depreciation charges. information on accumulated depreciation gives a broad indication of the age of the relevant assets. In the case of Advent above, other than the plant acquired during the year, plant is almost fully depreciated. The implication of this, assuming the depreciation policy is appropriate, is that further acquisitions will be required in the near future. This in turn has future cash flow implications. it can also be noted that no disposals of plant have occurred, thus the acquisition of plant represents an increase in capacity. This may be an indication of growth. the disclosure of the impairment charge as part of the accumulated depreciation disclosures is self-evident. Users can determine that the acquisition of the license appears to have been a financial disaster. Where a non-current asset is carried at historical cost, as in this case, the impairment is included as part of the depreciation rather than as a write down (revaluation) of the cost of the asset.

22

Part 2 Examination Paper 2.5 (INT) Financial Reporting (International Stream)

December 2004 Marking Scheme

This marking scheme is given as a guide in the context of the suggested answers. Scope is given to markers to award marks for alternative approaches to a question, including relevant comment, and where well-reasoned conclusions are provided. This is particularly the case for written answers where there may be more than one acceptable solution.

Marks 1 (a) Goodwill of Staybrite: value of shares exchanged 8% loan notes issued equity shares and share premium pre acquisition reserves fair value adjustments Goodwill of Allbrite: value of shares exchanged cash paid equity shares and share premium pre acquisition reserves available maximum (b) Income statement: revenue cost of sales operating expenses interest expense income from associate income tax minority interest available maximum (c) dividends retained profits b/f and c/f maximum Maximum for question 1 1 1 1 1 1 1 1 1 9 8

2 4 2 1 2 2 3 16 15 1 1 2 25

23

Marks 2 (a) Income statement revenue cost of sales operating costs investment income interest expense taxation available maximum (b) Balance sheet development costs property, plant and equipment amounts due from construction contract customers inventory and accounts receivable investment in finance leases (one for figure 1 for disclosure note) accrued finance income bank and trade creditors accrued finance costs income tax provision non-current liabilities share capital and reserves (including 1 mark for dividend paid) available maximum Maximum for question 2 6 1 1 2 2 14 11

1 2 2 1 2 1 1 1 1 2 2 16 14 25

(a) (b) (c)

1 mark per relevant point to a maximum 1 mark per relevant point to a maximum 1 mark per relevant point to a maximum Maximum for question

10 10 5 25

24

(a)

net profit before tax depreciation loss on disposal working capital items interest paid income tax paid capital expenditure disposal proceeds equity dividends financing equity shares financing loans decrease in cash available maximum

Marks 1 1 1 3 1 1 1 1 1 1 1 1 14 12 3 10 13 25

(b)

up to 3 marks for additional ratios 1 mark per relevant point including 1 mark for format maximum Maximum for question

(a)

(i)

ordinary shares preference shares capital reserve retained profits preference dividend ordinary dividends available maximum

1 1 2 2 1 2 9 8 4

(ii) (b) (i)

1 mark per relevant point to a maximum property, plant and equipment cost and accumulated depreciation at 1 October 2003 additions revaluation depreciation charges total columns license stays at cost of $300 million charge for year carrying value at 30 September 2004 disclosure note available maximum

2 1 1 1 2 1 1 1 1 11 9 4 25

(ii)

1 mark per relevant point to a maximum Maximum for question

25

Financial Reporting
(International Stream)
PART 2 THURSDAY 10 JUNE 2004

QUESTION PAPER Time allowed 3 hours This paper is divided into two sections Section A This ONE question is compulsory and MUST be answered THREE questions ONLY to be answered

Section B

The Association of Chartered Certified Accountants

Paper 2.5(INT)

4J_INTPA Paper 2.5INT

Section A This ONE question is compulsory and MUST be attempted 1 (a) Hapsburg, a public listed company, acquired the following investments: On 1 April 2003, 24 million shares in Sundial. This was by way of an immediate share exchange of two shares in Hapsburg for every three shares in Sundial plus a cash payment of $1 per Sundial share payable on 1 April 2006. The market price of Hapsburgs shares on 1 April 2003 was $2 each. On 1 October 2003, 6 million shares in Aspen paying an immediate $250 in cash for each share.

4J_INTAA Paper 2.5INT

Based on Hapsburgs cost of capital (taken as 10% per annum), $1 receivable in three years time can be taken to have a present value of $075. Hapsburg has not yet recorded the acquisition of Sundial but it has recorded the investment in Aspen. The summarised balance sheets at 31 March 2004 are: Non-current assets Property, plant and equipment Investments Hapsburg $000 $000 41,000 15,000 56,000 9,900 13,600 1,200 Sundial $000 $000 34,800 3,000 37,800 4,800 8,600 3,800 Aspen $000 $000 37,700 nil 37,700 7,900 14,400 nil

Current Assets Inventory Trade and other receivables Cash Total assets Equity and liabilities Capital and reserves Ordinary shares $1 each Reserves: Share premium Accumulated profits

24,700 80,700

17,200 55,000

22,300 60,000

20,000 8,000 10,600 2,000 8,500

30,000 nil 8,000

20,000

18,600 38,600 16,000

10,500 40,500 4,200

8,000 28,000 12,000

Non-current liabilities 10% loan note Current liabilities Trade and other payables Bank overdraft Taxation Total equity and liabilities

16,500 nil 9,600

26,100 80,700

6,900 nil 3,400

10,300 55,000

13,600 4,500 1,900

20,000 60,000

4J_INTAA Paper 2.5INT

The following information is relevant: (i) Below is a summary of the results of a fair value exercise for Sundial carried out at the date of acquisition: Asset Carrying value Fair value at acquisition at acquisition Notes $000 $000 Plant 10,000 15,000 remaining life at acquisition four years Investments 3,000 4,500 no change in value since acquisition The book values of the net assets of Aspen at the date of acquisition were considered to be a reasonable approximation to their fair values (ii) The profits of Sundial and Aspen for the year to 31 March 2004, as reported in their entity financial statements, were $45 million and $6 million respectively. No dividends have been paid by any of the companies during the year. All profits are deemed to accrue evenly throughout the year. (iii) In January 2004 Aspen sold goods to Hapsburg at a selling price of $4 million. These goods had cost Aspen $24 million. Hapsburg had $25 million (at cost to Hapsburg) of these goods still in inventory at 31 March 2004. (iv) Goodwill is to be written off over a five-year life with a proportionate charge in the year of acquisition. (v) All depreciation/amortisation is charged on a straight-line basis. (vi) Hapsburg uses the Allowed Alternative Treatment in IAS 22 Business Combinations to record the fair value of assets and liabilities. Required: Prepare the Consolidated Balance Sheet of Hapsburg as at 31 March 2004. (20 marks)

(b) Some commentators have criticised the use of equity accounting on the basis that it can be used as a form of off balance sheet financing. Required: Explain the reasoning behind the use of equity accounting and discuss the above comment. (5 marks) (25 marks)

[P.T.O.

4J_INTPB Paper 2.5INT

Section B THREE questions ONLY to be attempted 2 Reproduced below is the draft balance sheet of Tintagel, a public listed company, as at 31 March 2004. $000 $000 Non-current assets (note (i)) Freehold property 126,000 Plant 110,000 Investment property at 1 April 2003 (note (ii)) 15,000 251,000 Current Assets Inventory (note (iii)) 60,400 Trade receivables and prepayments 31,200 Bank 13,800 105,400 Total assets 356,400 Equity and liabilities Capital and Reserves: Ordinary shares of 25c each Reserves: Share premium Accumulated profits 1 April 2003 Accumulated profits Year to 31 March 2004 10,000 52,500 47,500

4J_INTBA Paper 2.5INT

150,000

110,000 260,000 18,700

Non-current liabilities Deferred tax at 1 April 2003 (note (v)) Current liabilities Trade payables (note (iii)) Provision for plant overhaul (note (iv)) Taxation Suspense account (note (vi)) Total equity and liabilities 47,400 12,000 4,200

63,600 14,100 356,400

(i)

The income statement has been charged with $32 million being the first of four equal annual rental payments for an item of excavating plant. This first payment was made on 1 April 2003. Tintagel has been advised that this is a finance lease with an implicit interest rate of 10% per annum. The plant had a fair value of $112 million at the inception of the lease. None of the non-current assets have been depreciated for the current year. The freehold property should be depreciated at 2% on its cost of $130 million, the leased plant is depreciated at 25% per annum on a straightline basis and the non-leased plant is depreciated at 20% on the reducing balance basis.

(ii) Tintagel adopts the fair value model for its investment property. Its value at 31 March 2004 has been assessed by a qualified surveyor at $124 million. (iii) During an inventory count on 31 March 2004 items that had cost $6 million were identified as being either damaged or slow moving. It is estimated that they will only realise $4 million in total, on which sales commission of 10% will be payable. An invoice for materials delivered on 12 March 2004 for $500,000 has been discovered. It has not been recorded in Tintagels bookkeeping system, although the materials were included in the inventory count. 4

4J_INTBA Paper 2.5INT

(iv) Tintagel operates some heavy excavating plant which requires a major overhaul every three years. The overhaul is estimated to cost $18 million and is due to be carried out in April 2005. The provision of $12 million represents two annual amounts of $6 million made in the years to 31 March 2003 and 2004. (v) The deferred tax provision required at 31 March 2004 has been calculated at $225 million. (vi) The suspense account contains the credit entry relating to the issue on 1 October 2003 of a $15 million 8% loan note. It was issued at a discount of 5% and incurred direct issue costs of $150,000. It is redeemable after four years at a premium of 10%. Interest is payable six months in arrears. The first payment of interest has not been accrued and is due on 1 April 2004. Apportionment of issue costs, discounts and premiums can be made on a straight-line basis. Required: (a) Commencing with the accumulated profit figures in the above balance sheet ($525 million and $475 million), prepare a schedule of adjustments required to these figures taking into account any adjustments required by notes (i) to (vi) above. (11 marks) (b) Redraft the balance sheet of Tintagel as at 31 March 2004 taking into account the adjustments required in notes (i) to (vi) above. (14 marks) (25 marks)

[P.T.O.

4J_INTBB Paper 2.5INT

(a) During the last decade it has not been unusual for the premium paid to acquire control of a business to be greater than the fair value of its tangible net assets. This increase in the relative balance sheet proportions of intangible assets has made the accounting practices for them all the more important. During the same period many companies have spent a great deal of money internally developing new intangible assets such as software and brands. IAS 38 Intangible Assets was issued in September 1998 and prescribes the accounting treatment for intangible assets. Required: In accordance with IAS 38, discuss whether intangible assets should be recognised, and if so how they should be initially recorded and subsequently amortised in the following circumstances: when they are purchased separately from other assets; when they are obtained as part of acquiring the whole of a business; and when they are developed internally. Note: your answer should consider goodwill separately from other intangibles. (10 marks)

(b) Dexterity is a public listed company. It has been considering the accounting treatment of its intangible assets and has asked for your opinion on how the matters below should be treated in its financial statements for the year to 31 March 2004. (i) On 1 October 2003 Dexterity acquired Temerity, a small company that specialises in pharmaceutical drug research and development. The purchase consideration was by way of a share exchange and valued at $35 million. The fair value of Temeritys net assets was $15 million (excluding any items referred to below). Temerity owns a patent for an established successful drug that has a remaining life of 8 years. A firm of specialist advisors, Leadbrand, has estimated the current value of this patent to be $10 million, however the company is awaiting the outcome of clinical trials where the drug has been tested to treat a different illness. If the trials are successful, the value of the drug is then estimated to be $15 million. Also included in the companys balance sheet is $2 million for medical research that has been conducted on behalf of a client. (4 marks)

(ii) Dexterity has developed and patented a new drug which has been approved for clinical use. The costs of developing the drug were $12 million. Based on early assessments of its sales success, Leadbrand have estimated its market value at $20 million. (3 marks) (iii) Dexteritys manufacturing facilities have recently received a favourable inspection by government medical scientists. As a result of this the company has been granted an exclusive five-year licence to manufacture and distribute a new vaccine. Although the licence had no direct cost to Dexterity, its directors feel its granting is a reflection of the companys standing and have asked Leadbrand to value the licence. Accordingly they have placed a value of $10 million on it. (3 marks) (iv) In the current accounting period, Dexterity has spent $3 million sending its staff on specialist training courses. Whilst these courses have been expensive, they have led to a marked improvement in production quality and staff now need less supervision. This in turn has led to an increase in revenue and cost reductions. The directors of Dexterity believe these benefits will continue for at least three years and wish to treat the training costs as an asset. (2 marks) (v) In December 2003, Dexterity paid $5 million for a television advertising campaign for its products that will run for 6 months from 1 January 2004 to 30 June 2004. The directors believe that increased sales as a result of the publicity will continue for two years from the start of the advertisements. (3 marks) Required: Explain how the directors of Dexterity should treat the above items in the financial statements for the year to 31 March 2004. (15 marks as indicated) Note: The values given by Leadbrand can be taken as being reliable measurements. You are not required to consider depreciation aspects. (25 marks) 6

This is a blank page. Question 4 begins on page 8.

[P.T.O.

4J_INTBC Paper 2.5INT

The following information relates to Planter, a small private company. It consists of an opening balance sheet as at 1 April 2003 and a listing of the companys ledger accounts at 31 March 2004 after the draft operating profit before interest and taxation (of $17,900) had been calculated. Planter Balance sheet as at Non-current assets Land and buildings (at valuation $49,200 less accumulated depreciation of $5,000) Plant (at cost of $70,000 less accumulated depreciation of $22,500) Investments at cost 1 April 2003 $ $ 44,200 47,500 16,900 108,600

Current Assets Inventory Trade receivables Bank Total assets Equity and liabilities Capital and Reserves: Ordinary shares of $1 each Reserves: Share premium Revaluation reserve Accumulated profits

57,400 28,600 1,200

87,200 195,800

25,000 5,000 12,000 70,300

87,300 112,300 43,200

Non-current liabilities 8% Loan notes Current liabilities Trade payables Taxation 31,400 8,900

40,300 195,800

4J_INTBC Paper 2.5INT

Ledger account listings at 31 March 2004 Ordinary shares of $1 each Share premium Accumulated profits 1 April 2003 Profit before interest and tax year to 31 March 2004 Revaluation reserve 8% Loan notes Trade payables Accrued loan interest Taxation Land and buildings at valuation Plant at cost Buildings accumulated depreciation 31 March 2004 Plant accumulated depreciation 31 March 2004 Investments at cost Trade receivables Inventory 31 March 2004 Bank Investment income Loan interest Ordinary dividend

Dr $

Cr $ 50,000 8,000 70,300 17,900 18,000 39,800 26,700 300

1,100 62,300 84,600 6,800 37,600 8,200 50,400 43,300 1,900 400 1,700 26,100 277,700

277,700

Notes (i) There were no disposals of land and buildings during the year. The increase in the revaluation reserve was entirely due to the revaluation of the companys land. (ii) Plant with a net book value of $12,000 (cost $23,500) was sold during the year for $7,800. The loss on sale has been included in the profit before interest and tax. (iii) Investments with a cost of $8,700 were sold during the year for $11,000. The profit has been included in the profit before interest and tax. There were no further purchases of investments. (iv) On 10 October 2003 a bonus issue of 1 for 10 ordinary shares was made utilising the share premium account. The remainder of the increase in ordinary shares was due to an issue for cash on 30 October 2003. (v) The balance on the taxation account is after settlement of the provision made for the year to 31 March 2003. A provision for the current year has not yet been made. Required: From the above information, prepare a cash flow statement using the indirect method for Planter in accordance with IAS 7 Cash Flow Statements for the year to 31 March 2004. (25 marks)

[P.T.O.

5
4J_INTBD Paper 2.5INT

(a) (i)

Linnet is a large public listed company involved in the construction industry. Accounting standards normally require construction contracts to be accounted for using the percentage (stage) of completion basis. However under certain circumstances they should be accounted for using the completed contracts basis.

Required: Discuss the principles that underlie each of the two methods and describe the circumstances in which their use is appropriate. (5 marks) (ii) Linnet is part way through a contract to build a new football stadium at a contracted price of $300 million. Details of the progress of this contract at 1 April 2003 are shown below: Cumulative sales revenue invoiced Cumulative cost of sales to date Profit to date $ million 150 112 38

The following information has been extracted from the accounting records at 31 March 2004: Total progress payment received for work certified at 29 February 2004 Total costs incurred to date (excluding rectification costs below) Rectification costs $ million 180 195 17

Linnet has received progress payments of 90% of the work certified at 29 February 2004. Linnets surveyor has estimated the sales value of the further work completed during March 2004 was $20 million. At 31 March 2004 the estimated remaining costs to complete the contract were $45 million. The rectification costs are the costs incurred in widening access roads to the stadium. This was the result of an error by Linnets architect when he made his initial drawings. Linnet calculates the percentage of completion of its contracts as the proportion of sales value earned to date compared to the contract price. All estimates can be taken as being reliable. Required: Prepare extracts of the financial statements for Linnet for the above contract for the year to 31 March 2004. (8 marks)

10

(b) (i)
4J_INTBD Paper 2.5INT

Large companies often conduct their operations across many different industrial and geographical areas. IAS 14 Segment Reporting is based on the principle that, without supplementary information, the aggregated financial statements of such companies are of little use to analysts and other users of financial statements.

Required: Describe how the provision of segment information is intended to assist users of financial statements, and identify the main problems of providing segment information. (6 marks) (ii) The Bilberry group is a large public company that operates in a single geographical market. Its directors have identified three distinguishable business segments; food processing, paint manufacturing and retailing of motor vehicles. The following information is available regarding its operations for the year to 31 March 2004: Segment sales revenue of the group (including inter-segment sales of $25 million) is $825 million made up of $270 million from food processing; $315 million from paint manufacturing and the remainder from motor vehicle retailing. The motor vehicle division sold cars at a gross profit of 20% to the other two divisions. The segment operating profit (before interest, tax and associated company income) is $187 million (including the profit from inter-segment sales). Food processing made a profit of $70 million and paint manufacturing made a profit of $65 million. The segment operating profit does not include $50 million of administration overheads which cannot be apportioned on a reasonable basis, nor $3 million interest received and $12 million finance costs. The groups share of the results of its equity accounted associated company (see below) is $30 million. The groups total assets are $890 million. These are attributable to $250 million to food processing; $270 million to paint manufacturing; $200 million to motor vehicle retailing and $140 million carrying value of the associated company investment. The associate operates entirely within the food processing sector. The remainder of the total assets are long-term investments in corporate bonds. Note: you may assume that the assets and related depreciation charges have been adjusted for the unrealised profits in the motor vehicles. The groups total liabilities are $120 million. These are attributable to $50 million to food processing; $45 million to paint manufacturing; and $25 million to motor vehicle retailing. Required: Prepare a segment report for Bilberry for the year ended 31 March 2004 incorporating the above information in accordance with IAS 14. (6 marks) (25 marks)

End of Question Paper

11

Answers

4JINTIX Paper 2.5INT

Part 2 Examination Paper 2.5 (INT) Financial Reporting (International Stream) 1 (a) Consolidated Balance Sheet of Hapsburg as at 31 March 2004: $000 Non current assets Goodwill (16,000 3,200 (w (i))) Property, plant and equipment (41,000 + 34,800 + 3,750 (w (i))) Investments: in associate (w (iv)) ordinary (3,000 + 1,500 (fair value increase) $000 12,800 79,550 15,150 4,500

June 2004 Answers

4JINTAA Paper 2.5INT

19,650 112,000

Current Assets Inventory (9,900 + 4,800 300 (w (v))) Trade receivables (13,600 + 8,600) Cash (1,200 + 3,800) Total assets Equity and liabilities Ordinary share capital (20,000 + 16,000 (w (i))) Reserves: Share premium (8,000 + 16,000 (w (i))) Accumulated profits (w (ii))

14,400 22,200 5,000

41,600 153,600 36,000

24,000 8,050

Minority interests (w (iii)) Non-current liabilities 10% Loan note (16,000 + 4,200) Deferred consideration (18,000 + 1,800 (w (vi))) Current liabilities: Trade payables (16,500 + 6,900) Taxation (9,600 + 3,400) Total equity and liabilities Workings Note: all working figures in $000. 20,200 19,800 23,400 13,000

32,050 68,050 9,150

40,000

36,400 153,600

The 80% (24m/30m shares) holding in Sundial is likely to give Hapsburg control and means it is a subsidiary and should be consolidated. The 30% (6m/20m shares) holding in Aspen is likely to give Hapsburg influence rather than control and thus it should be equity accounted. (i) Investments at cost (see below) 50,000 Cost of control Ordinary shares (30,000 80%) Share premium (2,000 80%) Pre acq profit (w (ii)) Fair value adjustments (see below) Goodwill 24,000 1,600 3,200 5,200 16,000 50,000

50,000

The purchase consideration for Sundial is $50 million. This is made up of an issue of 16 million shares (24/3 2) at $2 each totalling $32 million and deferred consideration of $24 million ($1 per share) which should be discounted to $18 million (24 million $075). The share issue should be recorded as $16 million share capital and $16 million share premium. The goodwill will be depreciated over five years at $32 million per annum.

15

4JINTAA Paper 2.5INT

Fair value adjustments: The Allowed Alternative treatment in IAS 22 requires the full fair value adjustment to be recorded with the minority being allocated their share. Fair value adjustment: Property, plant and equipment Investments Total 5,000 1,500 6,500 group share (80%) 4,000 1,200 5,200 minority (20%) 1,000 300 1,300

The fair value adjustment of $5 million to plant will be realised evenly over the next four years in the form of additional depreciation at $125 million per annum. In the year to 31 March 2004 the effect of this is $125 million charged to Sundials profits (as additional depreciation); and a net of $375 million added to the carrying value of the plant. Goodwill on acquisition of Aspen: Purchase consideration (6 million $250) Share capital Profits up to acquisition (8,000 (6,000 6/12)) Net assets at date of acquisition Difference goodwill 15,000 20,000 5,000 25,000 30%

(7,500) 7,500

Goodwill of $75 million depreciated over a five-year life would give a charge of $750,000 (i.e. six months only) for the year to 31 March 2004. (ii) Hapsburg Additional depreciation (w (i)) URP in inventory (w (v)) 300 Unwinding of interest (w (vi)) 1,800 Minority interest ((8,500 1,250) 20%) Pre-acq profit ((8,500 4,500) 80%) Post acq profit ((4,500 1,250) 80%) Amortisation of goodwill: Sundial (w (i)) Aspen (w (i)) Balance c/f 3,200 750 8,050 14,100 Accumulated profits Sundial 1,250 Per question Post acq profit Share of Aspens profit 1,450 (6,000 6/12 30%) 3,200 2,600

Hapsburg 10,600 2,600 900

Sundial 8,500

8,500 Minority interest

14,100

8,500

(iii)

Balance c/f

9,150 9,150

Ordinary shares (30,000 20%) Share premium (2,000 20%) Accumulated profits (w (ii)) Fair value adjustments (w (i))

6,000 400 1,450 1,300 9,150

(iv) Investment in associate: Investment at cost Share of post acquisition profit (w (ii)) Less goodwill amortisation (w (i))

15,000 900 (750) 15,150 8,400 6,750 15,150

Alternative calculation: Share of net assets at 31 March 2004 (28,000 30%) Goodwill (7,500 750)

16

4JINTAA Paper 2.5INT

(v)

Unrealised profit in inventory As the transaction is with an associate, only the group share of unrealised profits must be eliminated: $16 million 25 million/4 million 30% = $300,000

(vi) The deferred consideration of $24 million has been discounted to $18 million. The discounted amount will be unwound at 10% per annum. In the year to 31 March 2004 this will give an accrued finance cost of 10% (added to the carrying value of the deferred consideration). (b) In recent years many companies have increasingly conducted large parts of their business by acquiring substantial minority interests in other companies. There are broadly three levels of investment. Below 20% of the equity shares of an investee would normally be classed as an ordinary investment and shown at cost (it is permissible to revalue them to market value) with only the dividends paid by the investee being included in the income of the investor. A holding of above 50% normally gives control and would create subsidiary company status and consolidation is required. Between these two, in the range of over 20% up to 50%, the investment would normally be deemed to be an associate (note, the level of shareholding is not the only determining criterion). The relevance of this level of shareholding is that it is presumed to give significant influence over the operating and financial policies of the investee (but this presumption can be rebutted). If such an investment were treated as an ordinary investment, the investing company would have the opportunity to manipulate its profit. The most obvious example of this would be by exercising influence over the size of the dividend the associated company paid. This would directly affect the reported profit of the investing company. Also, as companies tend not to distribute all of their earnings as dividends, over time the cost of the investment in the balance sheet may give very little indication of its underlying value. Equity accounting for associated companies is an attempt to remedy these problems. In the income statement any dividends received from an associate are replaced by the investors share of the associates results. In the balance sheet the investment is initially recorded at cost and subsequently increased by the investors share of the retained profits of the associate (any other gains such as the revaluation of the associates assets would also be included in this process). This treatment means that the investor would show the same profit irrespective of the size of the dividend paid by the associate and the balance sheet more closely reflects the worth of the investment. The problem of off balance sheet finance relates to the fact that it is the net assets that are shown in the investors balance sheet. Any share of the associates liabilities is effectively hidden because they have been offset against the associates assets. As a simple example, say a holding company owned 100% of another company that had assets of $100 million and debt of $80 million, both the assets and the debt would appear on the consolidated balance sheet. Whereas if this single investment was replaced by owning 50% each of two companies that had the same balance sheets (i.e. $100 million assets and $80 million debt), then under equity accounting only $20 million ((100 80) 50% 2) of net assets would appear on the balance sheet thus hiding the $80 million of debt. Because of this problem, it has been suggested that proportionate consolidation is a better method of accounting for associated companies, as both assets and debts would be included in the investors balance sheet. IAS 28 Accounting for Investments in Associates does not permit the use of proportionate consolidation of associates, however IAS 31 Financial Reporting of Interests in Joint Ventures sets as its benchmark proportionate consolidation for jointly controlled entities (equity accounting is the allowed alternative).

4JINTBA Paper 2.5INT

(a)

Tintagel: Accumulated profits at 1 April 2003 Reversal of provision plant overhaul (w (iv))

$000

$000 52,500 6,000 58,500

Profit for the year to 31 March 2004 Lease rental charge added back (w (i)) Lease interest (w (i)) Depreciation (w (ii)) building Depreciation (w (ii)) owned plant Depreciation (w (ii)) leased plant Loss on investment property (15,000 12,400) Write down of inventory (w (iii)) Unrecorded trade payable Reversal of provision for plant overhaul (w (iv)) Increase in deferred tax (225 187) Loan note interest (06 + 03 (w (v))) Accumulated profits at 31 March 2004

47,500 3,200 (800) 2,600 22,000 2,800

(27,400) (2,600) (2,400) (500) 6,000 (3,800) (900)

18,300 76,800

17

4JINTBA Paper 2.5INT

(b)

Tintagel Balance sheet as at 31 March 2004: Tangible Non-current assets Freehold property (126,000 2,600 (w (ii))) Plant owned (110,000 22,000 (w (ii))) leased (11,200 2,800 (w (ii)) Investment property

$000

$000 123,400 88,000 8,400 12,400 232,200

Current Assets Inventory (60,400 2,400 (w (iii))) Trade receivables and prepayments Bank Total assets Equity and liabilities Capital and Reserves: Ordinary shares of 25c each Reserves: Share premium Accumulated profits 31 March 2004 (part (a))

58,000 31,200 13,800

103,000 335,200

150,000

10,000 76,800

86,800 236,800

Non-current liabilities Deferred tax Finance lease obligations (w (i)) 8% Loan note (14,100 + 300 (w (v)))

22,500 5,600 14,400

42,500

Current liabilities Trade payables (47,400 + 500) Accrued lease finance interest (w (i)) Accrued loan interest (w (v)) Finance lease obligation (w (i)) Taxation Total equity and liabilities

47,900 800 600 2,400 4,200

55,900 335,200

Workings (i) Finance lease: The lease has been incorrectly treated as an operating lease. Treating it as a finance lease gives the following figures: $000 Cash price/recorded cost 11,200 First instalment (reversed in income statement) (3,200) Capital outstanding at 1 April 2003 8,000 Interest at 10% p.a. to 31 March 2004 (current liability) 800 The capital outstanding of $8 million must be split between current and non-current liabilities. The second instalment payable on 1 April 2004 will contain $800,000 of interest (8,000 10%), therefore the capital element in this payment will be $24 million and this is a current liability. This leaves $56 million (8,000 2,400) as a non-current liability. (ii) Non-current assets depreciation: Buildings (130 2%) Non-leased plant (110 20%) Leased plant (11,200 25%) $000 2,600 22,000 2,800

(iii) The damaged and slow moving inventory should be written down to its estimated realisable value. This is $36 million ($4 million less sales commission at 10%). Therefore the required write down is $24 million ($6 million $36 million). (iv) A provision for a future major overhaul does not meet the definition of a liability in IAS 37 Provisions, Contingent Liabilities and Contingent Assets and must be reversed; this will increase the current years profit and the previous years profit by $6 million each.

18

4JINTBA Paper 2.5INT

(v)

International accounting standards require issue costs, discounts on issue and premiums on redemptions of loan instruments to be included as part of the finance costs: $000 $000 Issue proceeds (15,000 95%) 14,250 Issue costs (150) Initial carrying value (as per suspense account) 14,100 Less Payable on redemption (15,000 110%) 16,500 Total interest payments (15,000 8% 4 years) 4,800 (21,300) Total finance costs (7,200) The question says these may be apportioned on a straight-line basis at $18 million pa. The loan stock was issued on 1 October 2003 therefore an interest charge of $900,000 is required for the current year. Of this $600,000 is represented by the accrual to be paid on 1 April 2004 and the remainder is also accrued, but added to the carrying value of the loan stock on the balance sheet.

(a)

Goodwill: International Accounting Standards state that goodwill is the difference between the purchase consideration and the fair value of the acquired businesss identifiable (separable) net assets. Identifiable assets and liabilities are those that are capable of being sold or settled separately, i.e. without selling the business as a whole. Purchased goodwill should be recognised on the balance sheet at this value and amortised over its estimated useful economic life. The useful economic life is the period of time over which an asset is expected to be used by the enterprise. There is a rebuttable presumption that the life should not exceed 20 years. However, in the rare cases where there is persuasive evidence where it can be demonstrated that the life is more than 20 years, then it should be amortised over the best estimate of its useful life. Where this occurs the estimated recoverable amount must be estimated at least annually and written down if it is impaired. IAS 38 specifically states that internally generated goodwill (but not necessarily other intangibles) cannot be capitalised. Other intangibles: Where an intangible asset other than goodwill is acquired as a separate transaction, the treatment is relatively straightforward. It should be capitalised at cost and amortised over its estimated useful economic life (similar rules apply to the lives of other intangible assets as apply to goodwill as referred to above). The fair value of the purchase consideration paid to acquire an intangible is deemed to be its cost. Intangibles purchased as part of the acquisition of a business should be recognised separately to goodwill if they can be measured reliably. Reliable measurement does not have to be at market value, techniques such as valuations based on multiples of turnover or notional royalties are acceptable. This test is not meant to be overly restrictive and is likely to be met in valuing intangibles such as brands, publishing titles, patents etc. The amount of intangibles that may be recognised is restricted such that their recognition cannot create negative goodwill (as a balancing figure). Any intangible not capable of reliable measurement will be subsumed within goodwill. Recognition of internally developed intangibles is much more restrictive. IAS 38 states that internally generated brands, mastheads, publishing titles, customer lists and similar items should not be recognised as intangible assets as these items cannot be distinguished from the cost of developing the business as a whole. The Standard does require development costs to be capitalised if they meet detailed recognition criteria.

4JINTBB Paper 2.5INT

19

4JINTBB Paper 2.5INT

(b)

(i)

The purchase consideration of $35 million should be allocated as: Net tangible assets Work in progress Patent Goodwill

$m 15 2 10 8 35

The difficulty here is the potential value of the patent if the trials are successful. In effect this is a contingent asset and on an acquisition contingencies have to be valued at their fair value. There is insufficient information to make a judgment of the fair value of the contingent asset and in these circumstances it would be prudent to value the patent at $10 million. The additional $5 million is an example of where an intangible cannot be measured reliably and thus it should be subsumed within goodwill. The other issue is that although research cannot normally be treated as an asset, in this case the research is being done for another company and is in fact work in progress and should be recognised as such. (ii) This is an example of an internally developed intangible asset and although the circumstances of its valuation are similar to the patent acquired above it cannot be recognised at Leadbrands valuation. Internally generated intangibles can only be recognised if they meet the definition of development costs in IAS 38. Internally generated intangibles are permitted to be carried at a revalued amount (under the allowed alternative treatment) but only where there is an active market of homogeneous assets with prices that are available to the public. By their very nature drug patents are unique (even for similar types of drugs) therefore they cannot be part of a homogeneous population. Therefore the drug would be recorded at its development cost of $12 million.

(iii) This is an example of a granted asset. It is neither an internally developed asset nor a purchased asset. In one sense it is recognition of the standing of the company that is part of the companys goodwill. IAS 38s general requirement requires intangible assets to be initially recorded at cost and specifically mentions granted assets. IAS 38 also refers to IAS 20 Accounting for Government Grants and Disclosure of Government Assistance in this situation. This standard says that both the asset and the grant can be recognised at fair value initially (in this case they would be at the same amount). If fair values are not used for the asset it should be valued at the amount of any directly attributable expenditure (in this case this is zero). It is unclear whether IAS 38s general restrictive requirements on the revaluation of intangibles as referred to in (a) above (i.e. the allowed alternative treatment) are intended to cover granted assets under IAS 20. (iv) There is no doubt that a skilled workforce is of great benefit to a company. In this case there is an enhancement of revenues and a reduction in costs and if resources had been spent on a tangible non-current asset that resulted in similar benefits they would be eligible for capitalisation. However the Standard specifically excludes this type of expenditure from being recognised as an intangible asset and it describes highly trained staff as pseudo-assets. The main reason is the issue of control (through custody or legal rights). Part of the definition of any asset is the ability to control it. In the case of employees (or, as in this case, training costs of employees) the company cannot claim to control them, as it is quite possible that employees may leave the company and work elsewhere. (v) The benefits of effective advertising are often given as an example of goodwill (or an enhancement of it). If this view is accepted then such expenditures are really internally generated goodwill which cannot be recognised. In this particular case it would be reasonable to treat the unexpired element of the expenditure as a prepayment (in current assets) this would amount to 3/6 of $5 million i.e. $25 million. This represents the cost of the advertising that has been paid for, but not yet taken place. In the past some companies have treated anticipated continued benefits as deferred revenue expenditure, but this is no longer permitted as it does not meet the Standards recognition criteria for an asset.

20

4JINTBC Paper 2.5INT

Cash Flows From Operating Activities Planter for the Year to 31 March 2004: Reconciliation of operating profit to net cash inflow from operating activities $ Net profit before interest and tax (per question) Adjustments for: depreciation buildings (w (i)) 1,800 depreciation plant (w (i)) 26,600 loss on sale of plant (w (i)) profit on sale of investments (11,000 8,700) decrease in inventory (57,400 43,300) increase in receivables (50,400 28,600) decrease in payables (31,400 26,700) Cash generated from operations Interest paid (1,700 300 accrued) Income tax paid (8,900 + 1,100) Net cash flow from operating activities Cash Flows from Investing Activities Purchase of plant (w (i)) Purchase of land and buildings (w (i)) Investment income Sale of plant (w (i)) Sale of investments Cash flows from financing activities Issue of ordinary shares (w (ii)) Redemption of 8% loan notes Ordinary dividend paid Net decrease in cash and cash equivalents Cash and cash equivalents at 1 April 2003 Cash and cash equivalents at 31 March 2004 (38,100) (7,100) 400 7,800 11,000 28,000 (3,400) (26,100) $ 17,900

28,400 4,200 (2,300) 14,100 (21,800) (4,700) 35,800 (1,400) (10,000) 24,400

(26,000)

(1,500) (3,100) 1,200 (1,900)

21

Workings (i) Non-current assets: Land and buildings Valuation b/f Revaluation surplus (18,000 12,000) Acquisitions balancing figure Valuation c/f Depreciation b/f Charge for year balancing figure Depreciation c/f Plant Cost b/f Disposals at cost Acquisitions balancing figure Cost c/f Depreciation b/f Disposals Charge for year balancing figure Depreciation c/f Disposal of plant: Net book value Proceeds from question Loss on sale (ii) Share capital and share premium: Ordinary shares b/f Bonus issue 1 for 10 (from share premium) Ordinary shares c/f Difference issue for cash Share premium b/f Bonus issue Share premium c/f Increase is premium on cash issue Total proceeds of issue is (22,500 + 5,500) (iii) Reconciliation of revaluation reserve Balance b/f Difference revaluation of land Balance c/f

4JINTBC Paper 2.5INT

49,200 6,000 7,100 62,300 5,000 1,800 6,800 70,000 (23,500) 38,100 84,600 22,500 (11,500) 26,600 37,600 12,000 (7,800) 4,200

25,000 2,500 (50,000) 22,500 5,000 (2,500) (8,000) 5,500 28,000 12,000 6,000 18,000

22

4JINTBD Paper 2.5INT

(a)

(i)

Long-term construction contracts span more than one accounting year-end. This leads to the problem of determining how the uncompleted transactions should be dealt with over the life of the contract. Normal sales are not recognised until the production and sales cycle is complete. Prudence is the most obvious concept that is being applied in these circumstances, and this is the principle that underlies the completed contract basis. Where the outcome of a long-term contract cannot be reasonably foreseen due to inherent uncertainty, the completed contracts basis should be applied. The effect of this is that sales revenue earned to date is matched to the cost of sales and no profit is taken. The problem with the above is that for say a three year contract it can lead to a situation where no profits are recognised, possibly for two years, and in the year of completion the whole of the profit is recognised (assuming the contract is profitable). This seems consistent with the principle that only realised profits should be recognised in the income statement. The problem is that the overriding requirement is for financial statements to show a true and fair view which implies that financial statements should reflect economic reality. In the above case it can be argued that the company has been involved in a profitable contract for a three-year period, but its financial statements over the three years show a profit in only one period. This also leads to volatility of profits which many companies feel is undesirable and not favoured by analysts. An alternative approach is to apply the matching/accruals concept which underlies the percentage of completion method. This approach requires the percentage of completion of a contract to be assessed (there are several methods of doing this) and then recognising in the income statement that percentage of the total estimated profit on the contract. This method has the advantage of more stable profit recognition and can be argued shows a more true and fair view than the completed contract method. A contrary view is that this method can be criticised as being a form of profit smoothing which, in other circumstances, is considered to be an (undesirable) example of creative accounting. Accounting standards require the use of the percentage of completion method where the outcome of the contract is reasonably foreseeable. It should also be noted that where a contract is expected to produce a loss, the whole of the loss must be recognised as soon as it is anticipated.

(ii)

Linnet income statement extract year to 31 March 2004 (see working below): Sales revenue Cost of sales (64 +17) Loss for period $ million 70 (81) (11)

Linnet balance sheet extracts as at 31 March 2004 Current assets Gross amounts due from customers for contract work (w (iii)) Workings: cumulative 1 April 2003 $ million 150 (112) nil 38

59 amounts for year $ million 70 (64) (17) (11)

Sales Cost of sales Rectification costs Profit(loss) (i)

cumulative 31 March 2004 $ million (w (i)) 220 (w (ii)) (176) (17) (w (ii)) 27

progress payments received are $180 million. This is 90% of the work certified (at 29 February 2004), therefore the work certified at that date was $200 million. The value of the further work completed in March 2004 is given as $20 million, giving a total value of contract sales at 31 March 2004 of $220 million. the total estimated profit (excluding rectification costs) is $60 million: $ million contract price 300 cost to date (195) estimated cost to complete (45) estimated total profit 60 The degree of completion (by the method given in the question) is 220/300 Therefore the profit to date (before rectification costs) is $44 million ($60 million 220/300). Rectification costs must be charged to the period they were incurred and not spread over the remainder of the contract life. Therefore after rectification costs of $17 million the total reported contract profit to 31 March 2004 would be $27 million. With contract revenue of $220 million and profit to date of $44 million, this means contract costs (excluding rectification costs) would be $176 million. The difference between this figure and total cost incurred of $195 million is part of the $59 million of the amount due from customers shown in the balance sheet.

(ii)

(iii) The gross amounts due from customers is cost to date ($195 million + $17 million) plus cumulative profit ($27 million) less progress billings ($180 million) = $59 million.

23

4JGBRBD Paper 2.5INT

(b)

(i)

The financial statements of large diversified companies are an aggregation of all their separate activities. Their results are a composite of their individual segments. Each of the separate segments may have different results. The segments may have wide ranges of profitability, cash flows, growth, future prospects and risks. Without information on these separate segments, these differences would be concealed and it would be impossible for users of financial statements to properly assess past performance and future prospects. IAS 14 Segment Reporting requires primary and secondary reporting formats which are based on business and geographical segments. Providing information on sales revenues, results (profit) and net assets of each segment goes some way to resolving the difficulties outlined above. The main problems with the provision of segmental information are: defining what is a reportable segment. Although accounting standards give guidance on this, ultimately it is a matter for the directors to determine. This means that comparability between different companies can be impaired as they may be using different interpretations of what constitute reportable segments. IAS 14 has both primary and secondary reporting formats (by product group and geographically or vice versa). Similar companies may choose different primary formats to each other. apportionment between the segments of some costs on a reasonable basis can be difficult. This could be true for many common costs such as central administration. The Standard says where a reasonable basis of apportionment cannot be found they should be shown as a deduction from the aggregated profit and not apportioned. However it is possible that companies may use the apportionment of such costs to manipulate the relative profitabilities of their different segments. similar to the above there are some assets and liabilities that cannot be attributed to individual segments. Interest bearing assets are an example of this, as too are some forms of borrowing. Again the Standard says that such assets and liabilities should not be allocated to segments. Related interest receivable and finance costs are also examples of common items as referred to above.

(ii)

Bilberry Segment report Year to 31 March 2004 Food $ million Revenue External sales Inter-segment sales Total revenue Results Segment profit Unallocated corporate expenses Operating profit Interest expense Interest income Group share of associate Group profit before tax Other information Segment assets Investment in associate Unallocated corporate assets Consolidated total assets Segment liabilities Total net assets 270 nil 270 70 Paint $ million 315 nil 315 65 Motors $ million 215 25 240 47 Total $ million 800 25 825 (w (i)) 182 (50) 132 (12) 3 30 153 720 140 30 890 (120) 770

30

250 140

270

200

(50)

(45)

(25)

Working (i) The profit before tax of $187 million given in the question needs to be adjusted for $5 million profit (deducted) made on inter-segment sales ($25 million 20%).

24

4JGBRMS Paper 2.5INT

Part 2 Examination Paper 2.5 (INT) Financial Reporting (International Stream)

June 2004 Marking Scheme

This marking scheme is given as a guide in the context of the suggested answers. Scope is given to markers to award marks for alternative approaches to a question, including relevant comment, and where well-reasoned conclusions are provided. This is particularly the case for written answers where there may be more than one acceptable solution. 1 (a) Balance sheet: goodwill (four for total figure, one for depreciation) land and buildings plant investment in associate other investments inventory trade receivables and cash current liabilities 10% loan notes deferred consideration minority interest share capital and share premium accumulated profits available maximum (b) reasoning behind equity accounting discussion of off balance sheet financing maximum Maximum for question Marks 5 1 2 3 1 2 1 1 1 2 2 1 3 25 20 3 2 5 25

(a)

accumulated profits reversal of cyclic repair provision depreciation charges add back lease rental lease interest loan interest loss on investment property inventory write down unrecorded creditor deferred tax available maximum

2 3 1 1 1 1 1 1 1 12 11

(b)

Balance sheet freehold plant investment property inventory receivables and prepayments and bank trade receivables accrued lease interest accrued loan interest lease obligation (current liability) taxation lease obligation (non-current liability) 8% loan note deferred tax ordinary shares and share premium available maximum Maximum for question

1 2 1 2 1 1 1 1 1 1 1 2 1 1 17 14 25

25

(a)

discussion of goodwill other intangibles separate transactions other intangibles part of an acquisition other intangibles internally developed available maximum

Marks 3 2 3 3 11 10 4 1 1 1 1 1 1 1 1 1 1 1 1 1 1 18 15 25

4JGBRMS Paper 2.5INT

(b)

(i) (ii)

One mark for each item in balance sheet does it qualify as development expenditure the need for an active market drugs are unique, not homogeneous

(iii) neither an acquired asset nor internally generated really recognition of goodwill can recognise both the asset and the grant at fair value or at cost granted asset has zero cost (iv) in reality a valuable asset, in accounting a pseudo-asset cannot control workforce does not meet recognition criteria (v) effective advertising really part of goodwill cannot be recognised as a non-current asset prepayment of $25 million cannot spread over two years available maximum Maximum for question

reconciliation of operating profit to operating cash flows, one per item interest paid investment income taxation purchase of plant purchase of land and buildings sale of plant sale of investments ordinary dividend issue of ordinary shares redemption of loan notes decrease in cash available Maximum for question

8 2 1 2 2 2 1 1 2 2 2 1 26 25

26

4JGBRMS Paper 2.5INT

(a)

(i) (ii)

one mark per valid point to sales revenues cost of sales gross amounts due from customers

maximum

Marks 5 3 4 2 9 8 6 1 1 1 1 1 1 1 1 1 9 6 25

available maximum (b) (i) (ii) one mark per valid point to total sales less inter-segment profit before tax common costs and net interest share of associate segment assets share of associates net assets unallocated assets segment liabilities available maximum Maximum for question maximum

27

Financial Reporting
(International Stream)
PART 2 THURSDAY 8 DECEMBER 2005

QUESTION PAPER Time allowed 3 hours This paper is divided into two sections Section A This ONE question is compulsory and MUST be answered THREE questions ONLY to be answered

Section B

Do not open this paper until instructed by the supervisor This question paper must not be removed from the examination hall

The Association of Chartered Certified Accountants

Paper 2.5(INT)

Section A This ONE question is compulsory and MUST be attempted 1 Hedra, a public listed company, acquired the following investments: (i) On 1 October 2004, 72 million shares in Salvador for an immediate cash payment of $195 million. Hedra agreed to pay further consideration on 30 September 2005 of $49 million if the post acquisition profits of Salvador exceeded an agreed figure at that date. Hedra has not accounted for this deferred payment as it did not believe it would be payable, however Salvadors profits have now exceeded the agreed amount (ignore discounting). Salvador also accepted a $50 million 8% loan from Hedra at the date of its acquisition.

(ii) On 1 April 2005, 40 million shares in Aragon by way of a share exchange of two shares in Hedra for each acquired share in Aragon. The stock market value of Hedras shares at the date of this share exchange was $250. Hedra has not yet recorded the acquisition of the investment in Aragon. The summarised balance sheets of the three companies as at 30 September 2005 are: Hedra Salvador Aragon Non-current Assets $m $m $m $m $m $m Property, plant and equipment 358 240 270 Investments in Salvador 245 nil nil other 45 nil nil 648 240 270 Current Assets Inventories Trade receivables Cash and bank Total assets Equity and liabilities Ordinary share capital ($1 each) Reserves: Share premium Revaluation Retained earnings Non-current liabilities 8% loan note Deferred tax Current liabilities Trade payables Bank overdraft Current tax payable Total equity and liabilities The following information is relevant: (a) Fair value adjustments and revaluations: Hedras accounting policy for land and buildings is that they should be carried at their fair values. The fair value of Salvadors land at the date of acquisition was $20 million in excess of its carrying value. By 30 September 2005 this excess had increased by a further $5 million. Salvadors buildings did not require any fair value adjustments. The fair value of Hedras own land and buildings at 30 September 2005 was $12 million in excess of its carrying value in the above balance sheet. (ii) The fair value of some of Salvadors plant at the date of acquisition was $20 million in excess of its carrying value and had a remaining life of four years (straight-line depreciation is used). (i) 130 142 nil 80 97 4 110 70 20

272 920 400

181 421 120

200 470 100

40 15 240

295 695

50 nil 60

110 230

nil nil 300

300 400

nil 45 118 12 50

45

50 nil 141 nil nil

50

nil nil 40 nil 30

nil

180 920

141 421

70 470

(iii) At the date of acquisition Salvador had unrelieved tax losses of $40 million from previous years. Salvador had not accounted for these as a deferred tax asset as its directors did not believe the company would be sufficiently profitable in the near future. However, the directors of Hedra were confident that these losses would be utilised and accordingly they should be recognised as a deferred tax asset. By 30 September 2005 the group had not yet utilised any of these losses. The income tax rate is 25%. (b) The retained earnings of Salvador and Aragon at 1 October 2004, as reported in their separate financial statements, were $20 million and $200 million respectively. All profits are deemed to accrue evenly throughout the year. (c) An impairment test on 30 September 2005 showed that consolidated goodwill should be written down by $20 million. Hedra has applied IFRS 3 Business combinations since the acquisition of Salvador. (d) The investment in Aragon has not suffered any impairment. Required: Prepare the consolidated balance sheet of Hedra as at 30 September 2005. (25 marks)

[P.T.O.

Section B THREE questions ONLY to be attempted 2 The following trial balance relates to Petra, a public listed company, at 30 September 2005: $000 Revenue (note (i)) Cost of sales (note (i)) 114,000 Distribution costs 17,000 Administration expenses 18,000 Loan interest paid 1,500 Ordinary shares of 25 cents each fully paid Share premium Retained earnings 1 October 2004 6% Redeemable loan note (issued in 2003) Land and buildings at cost ((land element $40 million) note (ii)) 100,000 Plant and equipment at cost (note (iii)) 66,000 Deferred development expenditure (note (iv)) 40,000 Accumulated depreciation at 1 October 2004 buildings plant and equipment Accumulated amortisation of development expenditure at 1 October 2004 Income tax (note (v)) 1,000 Deferred tax (note (v)) Trade receivables 24,000 Inventories 30 September 2005 21,300 Cash and bank 11,000 Trade payables 413,800 The following notes are relevant: (i) Included in revenue is $12 million for receipts that the companys auditors have advised are commission sales. The costs of these sales, paid for by Petra, were $8 million. $3 million of the profit of $4 million was attributable to and remitted to Sharma (the auditors have advised that Sharma is the principal for these transactions). Both the $8 million cost of sales and the $3 million paid to Sharma have been included in cost of sales. $000 197,800

40,000 12,000 34,000 50,000

16,000 26,000 8,000 15,000

15,000 413,800

(ii) The buildings had an estimated life of 30 years when they were acquired and are being depreciated on the straight-line basis. (iii) Included in the trial balance figures for plant and equipment is plant that had cost $16 million and had accumulated depreciation of $6 million. Following a review of the companys operations this plant was made available for sale during the year. Negotiations with a broker have concluded that a realistic selling price of this plant will be $75 million and the broker will charge a commission of 8% of the selling price. The plant had not been sold by the year end. Plant is depreciated at 20% per annum using the reducing balance method. Depreciation of buildings and plant is charged to cost of sales. (iv) The development expenditure relates to the capitalised cost of developing a product called the Topaz. It had an original estimated life of five years. Production and sales of the Topaz started in October 2003. A review of the sales of the Topaz in late September 2005, showed them to be below forecast and an impairment test concluded that the fair value of the development costs at 30 September 2005 was only $18 million and the expected period of future sales (from this date) was only a further two years. (v) The balance on the income tax account in the trial balance is the under-provision in respect of the income tax liability for the year ended 30 September 2004. The directors have estimated the provision for income tax for the year ended 30 September 2005 to be $4 million and the required balance sheet provision for deferred tax at 30 September 2005 is $176 million.

Required: Prepare for Petra: (a) An income statement for the year ended 30 September 2005; and (b) A balance sheet as at 30 September 2005. Note: A statement of changes in equity is NOT required. Disclosure notes are NOT required. (c) The directors hold options to purchase 24 million shares for a total of $72 million. The options were granted two years ago and have been correctly accounted for. The options do not affect your answer to (a) and (b) above. The average stock market value of Petras shares for the year ended 30 September 2005 can be taken as 90 cents per share. Required: A calculation of the basic and diluted earnings per share for the year ended 30 September 2005 (comparatives are not required). (5 marks) (25 marks) (10 marks) (10 marks)

[P.T.O.

(a) IAS 36 Impairment of assets was issued in June 1998 and subsequently amended in March 2004. Its main objective is to prescribe the procedures that should ensure that an entitys assets are included in its balance sheet at no more than their recoverable amounts. Where an asset is carried at an amount in excess of its recoverable amount, it is said to be impaired and IAS 36 requires an impairment loss to be recognised. Required: (i) Define an impairment loss explaining the relevance of fair value less costs to sell and value in use; and state how frequently assets should be tested for impairment; (6 marks) Note: your answer should NOT describe the possible indicators of an impairment. (ii) Explain how an impairment loss is accounted for after it has been calculated. (5 marks)

(b) The assistant financial controller of the Wilderness group, a public listed company, has identified the matters below which she believes may indicate an impairment to one or more assets: (i) Wilderness owns and operates an item of plant that cost $640,000 and had accumulated depreciation of $400,000 at 1 October 2004. It is being depreciated at 121/2% on cost. On 1 April 2005 (exactly half way through the year) the plant was damaged when a factory vehicle collided into it. Due to the unavailability of replacement parts, it is not possible to repair the plant, but it still operates, albeit at a reduced capacity. Also it is expected that as a result of the damage the remaining life of the plant from the date of the damage will be only two years. Based on its reduced capacity, the estimated present value of the plant in use is $150,000. The plant has a current disposal value of $20,000 (which will be nil in two years time), but Wilderness has been offered a trade-in value of $180,000 against a replacement machine which has a cost of $1 million (there would be no disposal costs for the replaced plant). Wilderness is reluctant to replace the plant as it is worried about the long-term demand for the product produced by the plant. The trade-in value is only available if the plant is replaced. Required: Prepare extracts from the balance sheet and income statement of Wilderness in respect of the plant for the year ended 30 September 2005. Your answer should explain how you arrived at your figures. (7 marks) (ii) On 1 April 2004 Wilderness acquired 100% of the share capital of Mossel, whose only activity is the extraction and sale of spa water. Mossel had been profitable since its acquisition, but bad publicity resulting from several consumers becoming ill due to a contamination of the spa water supply in April 2005 has led to unexpected losses in the last six months. The carrying amounts of Mossels assets at 30 September 2005 are: Brand (Quencher see below) Land containing spa Purifying and bottling plant Inventories $000 7,000 12,000 8,000 5,000 32,000

The source of the contamination was found and it has now ceased. The company originally sold the bottled water under the brand name of Quencher, but because of the contamination it has rebranded its bottled water as Phoenix. After a large advertising campaign, sales are now starting to recover and are approaching previous levels. The value of the brand in the balance sheet is the depreciated amount of the original brand name of Quencher. The directors have acknowledged that $15 million will have to be spent in the first three months of the next accounting period to upgrade the purifying and bottling plant. Inventories contain some old Quencher bottled water at a cost of $2 million; the remaining inventories are labelled with the new brand Phoenix. Samples of all the bottled water have been tested by the health authority and have been passed as fit to sell. The old bottled water will have to be relabelled at a cost of $250,000, but is then expected to be sold at the normal selling price of (normal) cost plus 50%. 6

Based on the estimated future cash flows, the directors have estimated that the value in use of Mossel at 30 September 2005, calculated according to the guidance in IAS 36, is $20 million. There is no reliable estimate of the fair value less costs to sell of Mossel. Required: Calculate the amounts at which the assets of Mossel should appear in the consolidated balance sheet of Wilderness at 30 September 2005. Your answer should explain how you arrived at your figures. (7 marks) (25 marks)

[P.T.O.

The following draft financial statements relate to Tabba, a private company. Balance sheets as at: Tangible non-current assets (note (ii)) Current assets Inventories Trade receivables Insurance claim (note (iii)) Cash and bank Total assets Equity and liabilities Share capital ($1 each) Reserves: Revaluation (note (ii)) Retained earnings 30 September 2005 $000 $000 10,600 2,550 3,100 1,500 850 30 September 2004 $000 $000 15,800 1,850 2,600 1,200 nil

8,000 18,600 6,000

5,650 21,450 6,000

nil 2,550

2,550 8,550

1,600 850

2,450 8,450

Non-current liabilities Finance lease obligations (note (ii)) 6% loan notes 10% loan notes Deferred tax Government grants (note (ii)) Current liabilities Bank overdraft Trade payables Government grants (note (ii)) Finance lease obligations (note (ii)) Current tax payable Total equity and liabilities The following information is relevant: (i)

2,000 800 nil 200 1,400 nil 4,050 600 900 100

4,400

1,700 nil 4,000 500 900 550 2,950 400 800 1,200

7,100

5,650 18,600

5,900 21,450

Income statement extract for the year ended 30 September 2005: $000 Operating profit before interest and tax 270 Interest expense (260) Interest receivable 40 Profit before tax 50 Net income tax credit 50 Profit for the period 100 Note: the interest expense includes finance lease interest.

(ii) The details of the tangible non-current assets are: Cost Accumulated depreciation Carrying value $000 $000 $000 At 30 September 2004 20,200 4,400 15,800 At 30 September 2005 16,000 5,400 10,600

During the year Tabba sold its factory for its fair value $12 million and agreed to rent it back, under an operating lease, for a period of five years at $1 million per annum. At the date of sale it had a carrying value of $74 million based on a previous revaluation of $86 million less depreciation of $12 million since the revaluation. The profit on the sale of the factory has been included in operating profit. The surplus on the revaluation reserve related entirely to the factory. No other disposals of non-current assets were made during the year. Plant acquired under finance leases during the year was $15 million. Other purchases of plant during the year qualified for government grants of $950,000. Amortisation of government grants has been credited to cost of sales. (iii) The insurance claim relates to flood damage to the companys inventories which occurred in September 2004. The original estimate has been revised during the year after negotiations with the insurance company. The claim is expected to be settled in the near future. Required: (a) Prepare a cash flow statement using the indirect method for Tabba in accordance with IAS 7 Cash flow statements for the year ended 30 September 2005. (17 marks) (b) Using the information in the question and your cash flow statement, comment on the change in the financial position of Tabba during the year ended 30 September 2005. (8 marks) Note: you are not required to calculate any ratios. (25 marks)

[P.T.O.

(a) Elite Leisure is a private limited liability company that operates a single cruise ship. The ship was acquired on 1 October 1996. Details of the cost of the ships components and their estimated useful lives are: component original cost ($million) ships fabric (hull, decks etc) 300 cabins and entertainment area fittings 150 propulsion system 100 depreciation basis 25 years straight-line 12 years straight-line useful life of 40,000 hours

At 30 September 2004 no further capital expenditure had been incurred on the ship. In the year ended 30 September 2004 the ship had experienced a high level of engine trouble which had cost the company considerable lost revenue and compensation costs. The measured expired life of the propulsion system at 30 September 2004 was 30,000 hours. Due to the unreliability of the engines, a decision was taken in early October 2004 to replace the whole of the propulsion system at a cost of $140 million. The expected life of the new propulsion system was 50,000 hours and in the year ended 30 September 2005 the ship had used its engines for 5,000 hours. At the same time as the propulsion system replacement, the company took the opportunity to do a limited upgrade to the cabin and entertainment facilities at a cost of $60 million and repaint the ships fabric at a cost of $20 million. After the upgrade of the cabin and entertainment area fittings it was estimated that their remaining life was five years (from the date of the upgrade). For the purpose of calculating depreciation, all the work on the ship can be assumed to have been completed on 1 October 2004. All residual values can be taken as nil. Required: Calculate the carrying amount of Elite Leisures cruise ship at 30 September 2005 and its related expenditure in the income statement for the year ended 30 September 2005. Your answer should explain the treatment of each item. (12 marks) (b) Related party relationships are a common feature of commercial life. The objective of IAS 24 Related party disclosures is to ensure that financial statements contain the necessary disclosures to make users aware of the possibility that financial statements may have been affected by the existence of related parties. Required: (i) Describe the main circumstances that give rise to related parties. (4 marks) (3 marks) (iii) Hideaway is a public listed company that owns two subsidiary company investments. It owns 100% of the equity shares of Benedict and 55% of the equity shares of Depret. During the year ended 30 September 2005 Depret made several sales of goods to Benedict. These sales totalled $15 million and had cost Depret $14 million to manufacture. Depret made these sales on the instruction of the Board of Hideaway. It is known that one of the directors of Depret, who is not a director of Hideaway, is unhappy with the parent companys instruction as he believes the goods could have been sold to other companies outside the group at the far higher price of $20 million. All directors within the group benefit from a profit sharing scheme. Required: Describe the financial effect that Hideaways instruction may have on the financial statements of the companies within the group and the implications this may have for other interested parties. (6 marks) (25 marks)

(ii) Explain why the disclosure of related party relationships and transactions may be important.

End of Question Paper

10

Answers

Part 2 Examination Paper 2.5(INT) Financial Reporting (International Stream) 1 Consolidated balance sheet of Hedra as at 30 September 2005: $m Non-current assets Property, plant and equipment (358 + 240 + 12 + 20 + 5 +15 (w (iv))) Goodwill (100 20 (w (i))) Investment in associate (w (v)) Other investments Current Assets Inventories (130 + 80) Trade receivables (142 + 97) Cash and bank Total assets Equity and liabilities Equity attributable to the parent Ordinary share capital (400 + 80 (w (v))) Reserves: Share premium (40 + 120 (w (v))) Revaluation (15 + 12 + (5 x 60%) (w (iv))) Retained earnings (w (ii)) Minority interest (w (iii)) Non-current liabilities Deferred tax (45 10) Current liabilities Bank overdraft Trade payables (118 + 141) Deferred consideration (w (i)) Current tax payable Total equity and liabilities 12 259 49 50 $m 650 80 220 45 995

December 2005 Answers

210 239 4

453 1,448

480 160 30 261

451 931 112 1,043 35

370 1,448

Workings Note: all working figures in $million. The investment in Salvador represents 60% (72/120) of its equity and is likely to give Hedra control thus Salvador should be consolidated as a subsidiary. The investment in Aragon represents 40% (40/100) of its equity. Normally this would give Hedra significant influence and Aragon would be classed as an associate that should be equity accounted. (i) Investment at cost immediate deferred 195 49 Cost of control Ordinary shares Share premium (50 x 60%) Pre acq profit (w (ii)) Fair value adjustments (w (iv)) Goodwill 72 30 12 30 100 244

244 (ii) Additional depreciation (w (iv)) Minority interest ((60 5) x 40%) Pre-acq profit (20 x 60%) Post acq profit ((55 20) x 60%) Impairment of goodwill Balance c/f Retained earnings Hedra Salvador 5 22 12 21 20 261 281 60

The deferred contingent consideration has now become payable and has to be accounted for. Hedra 240 21 20 281 60 Salvador 60

Per balance sheet Post acq profit Share of Aragons profit ((300 200) x 6/12 x 40%)

13

(iii)

Minority interest Ordinary shares (120 x 40%) Share premium (50 x 40%) Retained earnings (w (ii)) Fair value adjustments (w (iv)) Post acq revaluation (5 x 40% (w (iv))) 48 20 22 20 2 112

Balance c/f

112 112 20 20 10 50 30 20

(iv) Fair value adjustments/revaluation Land and buildings Plant Deferred tax asset (40 x 25%)

Group share (60%) Minority share (40%)

The increase in the fair value of the land at the date of acquisition is accounted for as a fair value adjustment. The increase of a further $5 million in the year ended 30 September 2005 is a revaluation increase (accounted for as 60% to the group revaluation reserve and 40% to minority interest). The fair value adjustment of $20 million to plant will be realised evenly over the next four years in the form of additional depreciation at $5 million per annum. In the year ended 30 September 2005 the effect on the consolidated financial statements is that $5 million will be charged to Salvadors profit (as additional depreciation); and a net $15 million added to the carrying value of the plant. (v) Investment in associate: Investment at cost (100 x 40% x 2 x $250) Share of post acquisition profit (100 x 6/12 x 40%) 200 20 220

The purchase consideration by way of a share exchange (80 million in Hedra for 40 million in Aragon) would be recorded as an increase in share capital of $80 million ($1 nominal value) and an increase in share premium of $120 million (80 x $150).

(a)

Petra Income statement for the year ended 30 September 2005 Revenue (197,800 12,000 (w (i))) Cost of sales (w (ii)) Gross profit Other income commission received (w (i)) Distribution costs Administration expenses Interest expense (1,500 + 1,500) Profit before tax Income tax expense (4,000 +1,000 + (17,600 15,000)) Profit for the period $000 185,800 (128,100) 57,700 1,000 58,700 (17,000) (18,000) (3,000) 20,700 (7,600) 13,100

14

(b)

Petra Balance sheet as at 30 September 2005 Non-current assets (w (iii)) Property, plant and equipment Development costs cost $000 150,000 40,000 190,000 acc depn $000 44,000 22,000 66,000 21,300 24,000 11,000 6,900 carrying amount $000 106,000 18,000 124,000

Current assets Inventories Trade receivables Bank Held for sale assets plant (w (iii)) Total assets Equity and liabilities: Ordinary shares of 25c each Reserves: Share premium Retained earnings (34,000+ 13,100) Non-current liabilities 6% loan note Deferred tax Current liabilities Trade payables Accrued interest Current tax payable Total equity and liabilities (c)

63,200 187,200 40,000

12,000 47,100

59,100 99,100

50,000 17,600 15,000 1,500 4,000

67,600

20,500 187,200

Basic EPS: A nominal value of 25c per share would mean that the $40 million share capital represented 160 million shares. The basic EPS would thus be 82 cents ($131 million /160 million shares) Diluted EPS: The existence of the directors share options requires the disclosure of a diluted EPS. The dilution effect of the options is: Proceeds from options when exercised $72 million. This is equivalent to buying 8 million shares at full market value (72 million/90c). Thus the dilutive number of shares is 16 million (24 million 8 million). Diluted EPS is 74 cents ($131 million /(160 + 16 million shares)) Workings (figures in brackets are $000) (i) Agency sales: Petra has treated the sales it made on behalf of Sharma as its own sales. The advice from the auditors is that these are agency sales. Thus $12 million should be removed from revenue and the cost of the sales of $8 million and the $3 million share of profit to Sharma should also be removed from cost of sales. Petra should only recognise the commission of $1 million as income. The answer has included this as other income, but it would also be acceptable to include the commission in revenue. (ii) Cost of sales: Cost of sales (114,000 (8,000 3,000) (w (i))) Depreciation (w (iii)) buildings plant Amortisation (w (iii)) development expenditure Impairment of development expenditure (w (iii)) Impairment of plant held for sale (w (iii)) $000 103,000 2,000 6,000 8,000 6,000 3,100 128,100

15

(iii) Non-current assets/depreciation: The buildings will have a depreciation charge of $2 million (100,000 40,000)/30 years) giving accumulated depreciation at 30 September 2005 of $18 million (16,000 + 2,000). IFRS 5 Non-current assets held for sale and discontinued operations requires plant whose carrying amount will be recovered principally through sale (rather than use) to be classified as held for sale. It must be shown separately in the balance sheet and carried at the lower of its carrying amount (when classified as for continuing use) and its fair value less estimated costs to sell. Assets classified as held for sale should not be depreciated. Applying this: cost depn at 1 Oct 2004 carrying value $000 $000 $000 Plant and equipment per trial balance 66,000 26,000 40,000 Plant held for sale (16,000) (6,000) (10,000) Plant held for continuing use 50,000 20,000 30,000 Land and buildings 100,000 16,000 84,000 Property, plant and equipment 150,000 36,000 114,000 The continuing use plant will have a depreciation charge of $6 million ((50,000 20,000) x 20%) giving accumulated depreciation at 30 September 2005 of $26 million. The total accumulated depreciation for property, plant and equipment at 30 September 2005 will be $44 million (18,000 + 26,000). Plant held for sale must be valued at $69 million (7,500 selling price less commission of 600 (7,500 x 8%)) as this is lower than its carrying amount of $10 million. Thus an impairment charge of $31 million is required for the plant held for sale. Development expenditure: This has suffered an impairment as a result of disappointing sales. The impairment loss should be calculated after charging amortisation of $8 million (40,000/5 years) for the current year. Thus the impairment charge will be $6 million ((40,000 16,000) 18,000). The carrying amount of $18 million will then be written off over the next two years.

(a)

(i)

An impairment loss arises where the carrying amount of an asset is higher than its recoverable amount. The recoverable amount of an asset is defined in IAS 36 Impairment of assets as the higher of its fair value less costs to sell and its value in use (fair value less cost to sell was previously referred to as net selling price). Thus an impairment loss is simply the difference between the carrying amount of an asset and the higher of its fair value less costs to sell and its value in use. Fair value: The fair value could be based on the amount of a binding sale agreement or the market price where there is an active market. However many (used) assets do not have active markets and in these circumstances the fair value is based on a best estimate approach to an arms length transaction. It would not normally be based on the value of a forced sale. In each case the costs to sell would be the incremental costs directly attributable to the disposal of the asset. Value in use: The value in use of an asset is the estimated future net cash flows expected to be derived from the asset discounted to a present value. The estimates should allow for variations in the amount, timing and inherent risk of the cash flows. A major problem with this approach in practice is that most assets do not produce independent cash flows i.e. cash flows are usually produced in conjunction with other assets. For this reason IAS 36 introduces the concept of a cashgenerating unit (CGU) which is the smallest identifiable group of assets, which may include goodwill, that generates (largely) independent cash flows. Frequency of testing for impairment: Goodwill and any intangible asset that is deemed to have an indefinite useful life should be tested for impairment at least annually, as too should any intangible asset that has not yet been brought into use. In addition, at each balance sheet date an entity must consider if there has been any indication that other assets may have become impaired and, if so, an impairment test should be done. If there are no indications of impairment, testing is not required.

(ii)

Once an impairment loss for an individual asset has been identified and calculated it is applied to reduce the carrying amount of the asset, which will then be the base for future depreciation charges. The impairment loss should be charged to income immediately. However, if the asset has previously been revalued upwards, the impairment loss should first be charged to the revaluation surplus. The application of impairment losses to a CGU is more complex. They should first be applied to eliminate any goodwill and then to the other assets on a pro rata basis to their carrying amounts. However, an entity should not reduce the carrying amount of an asset (other than goodwill) to below the higher of its fair value less costs to sell and its value in use if these are determinable. The plant had a carrying amount of $240,000 on 1 October 2004. The accident that may have caused an impairment occurred on 1 April 2005 and an impairment test would be done at this date. The depreciation on the plant from 1 October 2004 to 1 April 2005 would be $40,000 (640,000 x 121/2% x 6/12) giving a carrying amount of $200,000 at the date of impairment. An impairment test requires the plants carrying amount to be compared with its recoverable amount. The recoverable amount of the plant is the higher of its value in use of $150,000 or its fair value less costs to sell. If Wilderness trades in the plant it would receive $180,000 by way of a part exchange, but this is conditional on buying new plant which Wilderness is reluctant to do. A more realistic amount of the fair value of the plant is its current

(b)

(i)

16

disposal value of only $20,000. Thus the recoverable amount would be its value in use of $150,000 giving an impairment loss of $50,000 ($200,000 $150,000). The remaining effect on income would be that a depreciation charge for the last six months of the year would be required. As the damage has reduced the remaining life to only two years (from the date of the impairment) the remaining depreciation would be $37,500 ($150,000/ 2 years x 6/12).Thus extracts from the financial statements for the year ended 30 September 2005 would be: Balance sheet Non-current assets Plant (150,000 37,500) Income statement Plant depreciation (40,000 + 37,500) Plant impairment loss (ii) $ 112,500 77,500 50,000

There are a number of issues relating to the carrying amount of the assets of Mossel that have to be considered. It appears the value of the brand is based on the original purchase of the Quencher brand. The company no longer uses this brand name; it has been renamed Phoenix. Thus it would appear the purchased brand of Quencher is now worthless. Mossel cannot transfer the value of the old brand to the new brand, because this would be the recognition of an internally developed intangible asset and the brand of Phoenix does not appear to meet the recognition criteria in IAS 38. Thus prior to the allocation of the impairment loss the value of the brand should be written off as it no longer exists. The inventories are valued at cost and contain $2 million worth of old bottled water (Quencher) that can be sold, but will have to be relabelled at a cost of $250,000. However, as the expected selling price of these bottles will be $3 million ($2 million x 150%), their net realisable value is $2,750,000. Thus it is correct to carry them at cost i.e. they are not impaired. The future expenditure on the plant is a matter for the following years financial statements. Applying this, the revised carrying amount of the net assets of Mossels cash-generating unit (CGU) would be $25 million ($32 million $7 million re the brand). The CGU has a recoverable amount of $20 million, thus there is an impairment loss of $5 million. This would be applied first to goodwill (of which there is none) then to the remaining assets pro rata. However under IAS2 the inventories should not be reduced as their net realisable value is in excess of their cost. This would give revised carrying amounts at 30 September 2005 of: Brand Land containing spa (12,000 (12,000/20,000 x 5,000)) Purifying and bottling plant (8,000 (8,000/20,000 x 5,000)) Inventories $000 nil 9,000 6,000 5,000 20,000

17

(a)

Cash flow statement of Tabba for the year ended 30 September 2005: Cash flows from operating activities Profit before tax Adjustments for: Depreciation (w (i)) Amortisation of government grant (w (iii)) Profit on sale of factory (w (i)) Increase in insurance claim provision (1,500 1,200) Interest receivable Interest expense Working capital adjustments: Increase in inventories (2,550 1,850) Increase in trade receivables (3,100 2,600) Increase in trade payables (4,050 2,950) Cash outflow from operations Interest paid Income taxes paid (w (iv)) Net cash outflow from operating activities Cash flows from investing activities Sale of factory Purchase of non-current assets (w (i)) Receipt of government grant (from question) Interest received Net cash from investing activities Cash flows from financing activities Issue of 6% loan notes Redemption of 10% loan notes Repayment of finance leases (w (ii)) Net cash from financing activities Net increase in cash and cash equivalents Cash and cash equivalents at beginning of period Cash and cash equivalents at end of period 800 (4,000) (1,100) (4,300) 1,400 (550) 850 12,000 (2,900) 950 40 10,090 $000 50 2,200 (250) (4,600) (300) (40) 260 (2,680) (700) (500) 1,100 (2,780) (260) (1,350) (4,390) $000

Note: interest paid may also be presented as a financing activity and interest received as an operating cash flow. Workings ($000) (i) Non-current assets: Cost/valuation b/f New finance leases (from question) Disposals Acquisitions balancing figure Cost/valuation c/f Depreciation b/f Disposal Depreciation c/f Charge for year balancing figure Sale of factory: Net book value Proceeds (from question) Profit on sale (ii) Finance lease obligations Balance b/f current over 1 year New leases (from question) Balance c/f current over 1 year Cash repayments balancing figure

20,200 1,500 (8,600) 2,900 16,000 4,400 (1,200) (5,400) (2,200) 7,400 (12,000) (4,600) 800 1,700 1,500 (900) (2,000) 1,100

18

(iii) Government grant: Balance b/f current over 1 year Grants received in year (from question) Balance c/f current over 1 year Difference amortisation credited to income statement (iv) Taxation: Current provision b/f Deferred tax b/f Tax credit in income statement Current provision c/f Deferred tax c/f Tax paid balancing figure (v) Reconciliation of retained earnings Balance b/f Transfer from revaluation reserve Profit for period Balance c/f (b)

400 900 950 (600) (1,400) 250 1,200 500 (50) (100) (200) 1,350 850 1,600 100 2,550

Consideration of the cash flow statement reveals some important information in assessing the change in the financial position of Tabba in the year ended 30 September 2005. There is a huge net cash outflow from operating activities of $4,390,000 despite Tabba reporting a modest operating profit of $270,000. More detailed analysis of this difference reveals some worrying concerns for the future. Many companies experience higher operating cash flows than the underlying operating profit mainly due to depreciation charges being added back to profits to arrive at the cash flows. This is certainly true in Tabbas case, where operating profits have been improved by $22 million during the year in terms of the underlying cash flows. However, the major reconciling difference is the profit on the sale of Tabbas factory of $46 million. This amount has been credited in the income statement and has dramatically distorted the operating profit. If the sale and lease back of the factory had not taken place, Tabbas operating profits would be in a sorry state showing losses of $433 million (ignoring any possible tax effects). When Tabba publishes its financial statements this profit will almost certainly require separate disclosure which should make the effects of the transaction more transparent to the users of the financial statements. A further indication of poor operating profits is that they have been boosted by $300,000 due to an increase in the insurance claim provision (again this is not a cash flow) and $250,000 amortisation of government grants. Many commentators believe that the net cash flow from operating activities is the most important figure in the cash flow statement. This is because it is a measure of expected or maintainable future cash flows. In Tabbas case this highlights a very important point; although Tabba has increased its cash position during the year by $14 million, $12 million has come from the sale of its factory. Clearly this is a one off transaction that cannot be repeated in future years. If the drain on the operating cash flows continues at the current rates, the company will not survive for very long. The tax position is worthy of comment. There is a small tax credit in the income statement, perhaps due to current year trading losses, whereas the cash flow statement shows that tax of $135 million has been paid during the year. This payment of tax is on what must have been a substantial profit for the previous year. This seems to confirm the deteriorating position of the company. Another relevant point is that there has been a very small increase in working capital of $100,000. However, underlying this is the fact that both inventories and trade receivables are showing substantial increases (despite the profit deterioration), which may indicate the presence of bad debts or obsolete inventories, and trade payables have also increased substantially (by $11 million) which may be a symptom of liquidity problems prior to the sale of the factory. On the positive side there has been substantial investment in non-current assets (after allowing for the sale of the factory), but even this is partly due to leasing assets of $15 million (companies often lease assets when they do not have the resources to purchase them outright) and finance from a government grant of $950,000. The company appears to have taken advantage of the proceeds from the sale of the factory to redeem the expensive 10% $4 million loan note (this has partly been replaced by a less expensive 6% $800,000 loan note). In conclusion the cash flow statement reveals some interesting and worrying issues that may indicate a bleak future for Tabba and serves as an illustration of the importance of a cash flow statement to the users of financial statements.

19

(a)

The cruise ship is an example of what can be called a complex asset. This is a single asset that should be treated as if it was a collection of separate assets, each of which may require a different depreciation method/life. In this case the question identifies three components to the cruise ship. The carrying amount of the asset at 30 September 2004 (eight years after acquisition) would be: component cost depreciation carrying value $m $m $m ships fabric 300 96 (300/25 x 8) 204 cabins and entertainment area fittings 150 100 (150/12 x 8) 50 propulsion system 100 75 (100/40,000 x 30,000) 25 550 271 279 Ships fabric This is the most straightforward component. It is being depreciated over a 25 year life and depreciation of $12 million (300/25 years) would be required in the year ended 30 September 2005. The repainting of the ships fabric does not meet the recognition criteria of an asset and should be treated as repairs and maintenance. Cabins and entertainment area and fittings During the year these have had a limited upgrade at a cost of $60 million. This has extended the remaining useful life from four to five years. The costs of the upgrade meet the criteria for recognition as an asset. The original fittings have not been replaced thus the additional $60 million would be added to the cost of the fittings and the new carrying amount of $110 million will be depreciated over the remaining life of five years to give a charge for the year of $22 million. Propulsion system This has been replaced by a new system so the carrying value of the system ($25 million) must be written off and depreciation of the new system for the year ended 30 September 2005 (based on use) would be $14 million (140 million/50,000 x 5,000). Elite Leisure income statement extract year ended 30 September 2005: $m Depreciation ships fabric 12 cabin and entertainment fittings 22 propulsion system 14 Disposal loss propulsion system 25 Repainting ships fabric 20 93 Elite Leisure balance sheet extract as at 30 September 2005 Non-current assets Cruise ship (see working) 406 Workings (in $ million): component ships fabric cabins and entertainment area fittings propulsion system cost $m 300 210 140 650 depreciation $m 108 (300/25 x 9) 122 (110/5 + 100) 14 (140/50,000 x 5,000) 244 carrying value $m 192 88 126 406

(b)

(i)

IAS 24 Related party disclosures says that a party is related to an entity if: the party, directly or indirectly, controls, is controlled by or is under common control with the entity (e.g. parent/subsidiary or subsidiaries of the same group) one party has an interest in another entity that gives it significant influence over the entity (e.g. an associate) or has joint control over the entity (e.g. joint venturers are related parties) In addition members of key management and close family members of related parties are also themselves related parties. In the absence of related party disclosures, users of financial statements would assume that an entity has acted independently and in its own best interests. Principally within this assumption is that all transactions have been entered into willingly and at arms length (i.e. on normal commercial terms at fair value). Where related party relationships and transactions exist, this assumption may not be justified. These relationships and transactions lead to the danger that financial statements may have been distorted or manipulated, both favourably and unfavourably. The most obvious example of this type of transaction would be the sale of goods or rendering of services from one party to another on noncommercial terms (this may relate to the price charged or the credit terms given). Other examples of disclosable transactions are agency, licensing and leasing arrangements, transfer of research and development and the provision of finance, guarantees and collateral. Collectively this would mean there is hardly an area of financial reporting that could not be affected by related party transactions. It is a common misapprehension that related party transactions need only be disclosed if they are not at arms length. This is not the case. For example, a parent may instruct all members of its group to buy certain products or services (on

(ii)

20

commercial terms) from one of its subsidiaries. In the absence of the related party relationships, these transactions may not have occurred. If the parent were to sell the subsidiary, it would be important for the prospective buyer to be aware that the related party transactions would probably not occur in the future. Indeed even where there are no related party transactions, the disclosure of the related party relationship is still important as a subsidiary may obtain custom, receive favourable credit ratings, and benefit from a superior management team simply by being a part of a well respected group. (iii) The subsidiaries of Hideaway are related parties to each other and to Hideaway itself as they are under common control. One of the important aspects of related party relationships is that one of the parties may have its interests subordinated i.e. it may not be able to act in its own best interest. This appears to be the case in this situation. Depret (or at least one of its directors) believes that the price it is charging Benedict is less than it could have achieved by selling the goods to non-connected parties. In effect these sales have not been made at an arms length fair value. The obvious implication of this is that the transactions have moved profits from Depret to Benedict. If the directors figures are accurate Depret would have made a profit on these transactions of $6 million (20 14) rather than the $1 million it has actually made. The transactions will also affect reported revenue and cost of sales and working capital in the individual financial statements of Benedict and Depret. Some might argue that as the profit remains within the group, there is no real overall effect as, in the consolidated financial statements, intra-group transactions are eliminated. This is not entirely true. The implications of these related party sales are serious: Depret has a minority interest of 45% and they have been deprived of their share of the $5 million transferred profit. This could be construed as oppression of the minority and is probably illegal. there is a similar effect on the profit share that the directors of Depret might be entitled to under the group profit sharing scheme as Deprets profits are effectively $5 million lower than they should be. shareholders, independent analysts or even the (independent) managers of Depret would find it difficult to appraise the true performance of Depret. The related party transaction gives the impression that Depret is under-performing. This may lead to the minority selling their shares for a low price (because of poor returns) or calls for the companys closure or some form of rationalisation which may not be necessary. the tax authorities may wish to investigate the transactions under transfer pricing rules. The profit may have been moved to Benedicts financial statements to avoid paying tax in Deprets tax jurisdiction which may have high levels of taxation. in the same way as Deprets results appear poorer due to the effect of the related party transactions, Benedicts results would look better. This may have been done deliberately. Hideaway may intend to dispose of Benedict in the near future and thus its more favourable results may allow Hideaway to obtain a higher sale price for Benedict.

21

Part 2 Examination Paper 2.5(INT) Financial Reporting (International Stream)

December 2005 Marking Scheme

This marking scheme is given as a guide in the context of the suggested answers. Scope is given to markers to award marks for alternative approaches to a question, including relevant comment, and where well-reasoned conclusions are provided. This is particularly the case for written answers where there may be more than one acceptable solution. Marks 5 1 3 3 1 1 1 1 2 3 4 1 1 1 1 1 30 25

goodwill goodwill impairment property, plant and equipment investment in associate other investments inventories and trade receivables cash and bank share capital and premium revaluation reserve retained earnings minority interest deferred consideration deferred tax elimination of 8% loan note trade payables and tax overdraft available Maximum for question

(a)

income statement revenue cost of sales 1 mark per each item commission distribution and administration interest expense income tax

maximum

available maximum (b) balance sheet development costs property, plant and equipment (held for continuing use) plant held for sale (1 mark for separate presentation) inventories trade receivables and cash and bank ordinary shares and share premium retained earnings 6% loan note deferred tax trade payables current tax available maximum (c) basic eps diluted eps maximum Maximum for question

1 6 1 1 1 2 12 10

1 2 2 1 1 1 1 1 1 1 1 13 10 2 3 5 25

23

(a)

(i)

impaired where carrying amounts higher than recoverable amounts discussion of fair value discussion of value in use discussion of CGU goodwill/intangibles with indefinite life tested annually review for indicators of impairment each balance sheet date only test if there is an indication of impairment available maximum impairment loss individual asset: impairment loss applied to carrying value of asset and charged to any previous revaluation surplus then income CGU: Applied to goodwill then pro rata to other assets other assets not reduced below fair value/value in use available maximum depreciation/carrying value 1 April 2004 fair value less costs to sell is disposal value of $20,000, not trade-in value recoverable is therefore $150,000 impairment loss is $50,000 depreciation six months to 30 September 2004 carrying value $112,500 available maximum old brand written off, cannot recognise new brand inventories correct at cost improvement to plant not relevant impairment loss is $5 million land reduced to $9 million plant reduced to $6 million available maximum Maximum for question

Marks 1 2 2 1 1 1 1 9 6 1 2 1 1 1 6 5 2 2 1 1 1 1 8 7 2 2 1 1 1 1 8 7 25

(ii)

(b)

(i)

(ii)

24

(a)

profit before tax depreciation amortisation of government grant profit on sale of factory increase in insurance claim working capital items 1 mark each adjustment for interest receivable/payable interest paid income tax paid sale of factory purchase of non-current assets receipt of government grant interest received redemption of 10% loan issue of 6% loan repayment of finance lease cash b/f and c/f available maximum

Marks 1/ 2 1 1 1 1 3 1/ 2 1 2 1 1 1 1 1 1 2 1 20 17 8 25

(b)

1 mark per relevant point to a

maximum Maximum for question

(a)

principle that each component is treated separately explanation of treatment of each component 1 mark each charges to income statement, 1 each carrying value at 30 September 2005 ships fabric cabins etc propulsion system available maximum

1 3 5 1 2 2 14 12 4 3 1 1 1 1 4 8 6 25

(b)

(i) (ii)

1 mark per relevant point to a 1 mark per relevant point to a

maximum maximum

(iii) effect of related party transaction is to subordinate Deprets interest profit of $5 million has moved from Depret to Benedict also effect sales/cost of sales working capital in consolidated financial statements most transactions eliminated implication for users, 1 mark per point up to available maximum Maximum for question

25

Financial Reporting
(International Stream)
PART 2 THURSDAY 9 JUNE 2005

QUESTION PAPER Time allowed 3 hours This paper is divided into two sections Section A This ONE question is compulsory and MUST be answered THREE questions ONLY to be answered

Section B

Do not open this paper until instructed by the supervisor This question paper must not be removed from the examination hall

The Association of Chartered Certified Accountants

Paper 2.5(INT)

Section A This ONE question is compulsory and MUST be attempted 1 Highveldt, a public listed company, acquired 75% of Samsons ordinary shares on 1 April 2004. Highveldt paid an immediate $350 per share in cash and agreed to pay a further amount of $108 million on 1 April 2005. Highveldts cost of capital is 8% per annum. Highveldt has only recorded the cash consideration of $350 per share. The summarised balance sheets of the two companies at 31 March 2005 are shown below: Highveldt $million $million 420 nil 300 720 133 853 270 80 45 160 190 350 745 nil 108 853 134 76 Samson $million $million 320 40 20 380 91 471 80 40 nil 210 330 60 81 471

Tangible non-current assets (note (i)) Development costs (note (iv)) Investments (note (ii)) Current assets Total assets Equity and liabilities: Ordinary shares of $1 each Reserves: Share premium Revaluation reserve Retained earnings 1 April 2004 year to 31 March 2005 Non-current liabilities 10% inter company loan (note (ii)) Current liabilities Total equity and liabilities The following information is relevant: (i)

Highveldt has a policy of revaluing land and buildings to fair value. At the date of acquisition Samsons land and buildings had a fair value $20 million higher than their book value and at 31 March 2005 this had increased by a further $4 million (ignore any additional depreciation).

(ii) Included in Highveldts investments is a loan of $60 million made to Samson at the date of acquisition. Interest is payable annually in arrears. Samson paid the interest due for the year on 31 March 2005, but Highveldt did not receive this until after the year end. Highveldt has not accounted for the accrued interest from Samson. (iii) Samson had established a line of products under the brand name of Titanware. Acting on behalf of Highveldt, a firm of specialists, had valued the brand name at a value of $40 million with an estimated life of 10 years as at 1 April 2004. The brand is not included in Samsons balance sheet. (iv) Samsons development project was completed on 30 September 2004 at a cost of $50 million. $10 million of this had been amortised by 31 March 2005. Development costs capitalised by Samson at the date of acquisition were $18 million. Highveldts directors are of the opinion that Samsons development costs do not meet the criteria in IAS 38 Intangible Assets for recognition as an asset. (v) Samson sold goods to Highveldt during the year at a profit of $6 million, one-third of these goods were still in the inventory of Highveldt at 31 March 2005. (vi) An impairment test at 31 March 2005 on the consolidated goodwill concluded that it should be written down by $22 million. No other assets were impaired.

Required: (a) Calculate the following figures as they would appear in the consolidated balance sheet of Highveldt at 31 March 2005: (i) goodwill; (8 marks) (ii) minority interest; (4 marks) (iii) the following consolidated reserves: share premium, revaluation reserve and retained earnings. (8 marks) Note: show your workings (b) Explain why consolidated financial statements are useful to the users of financial statements (as opposed to just the parent companys separate (entity) financial statements). (5 marks) (25 marks)

[P.T.O.

Section B THREE questions ONLY to be attempted 2 Reproduced below are the draft financial statements of Harrington, a public company, for the year to 31 March 2005: Income statement Year to 31 March 2005 Sales revenue (note (i)) Cost of sales (note (ii)) Gross profit Operating expenses Loan note interest paid (refer to balance sheet) Profit before tax Income tax expense (note (vi)) Profit for the period Balance Sheet as at 31 March 2005 $000 Property, plant and equipment (note (iii)) Investments (note (iv)) Current assets Inventory Trade receivables Bank Total assets Equity and liabilities: Ordinary shares of 25c each (note (v)) Reserves: Share premium Retained earnings 1 April 2004 Year to 31 March 2005 dividends paid Non-current liabilities 10% loan note (issued 2002) Deferred tax (note (vi)) Current liabilities Trade payables $000 6,270 1,200 7,470 $000 13,700 (9,200) _______ 4,500 (2,400) (25) 2,075 (55) 2,020

1,750 2,450 350

4,550 12,020 2,000 600

2,990 2,020 (500)

4,510 7,110

500 280

780

4,130 12,020

The company policy for ALL depreciation is that it is charged to cost of sales and a full years charge is made in the year of acquisition or completion and none in the year of disposal. The following matters are relevant: (i) Included in sales revenue is $300,000 being the sale proceeds of an item of plant that was sold in January 2005. The plant had originally cost $900,000 and had been depreciated by $630,000 at the date of its sale. Other than recording the proceeds in sales and cash, no other accounting entries for the disposal of the plant have been made. All plant is depreciated at 25% per annum on the reducing balance basis.

(ii) On 31 December 2004 the company completed the construction of a new warehouse. The construction was achieved using the companys own resources as follows: purchased materials direct labour supervision design and planning costs $000 150 800 65 20

Included in the above figures are $10,000 for materials and $25,000 for labour costs that were effectively lost due to the foundations being too close to a neighbouring property. All the above costs are included in cost of sales. The building was brought into immediate use on completion and has an estimated life of 20 years (straightline depreciation). (iii) Details of the other property, plant and equipment at 31 March 2005 are: $000 Land at cost Buildings at cost 4,000 Less accumulated depreciation at 31 March 2004 (800) Plant at cost 5,200 Less accumulated depreciation at 31 March 2004 (3,130) $000 1,000 3,200 2,070 6,270

At the beginning of the current year (1 April 2004), Harrington had an open market basis valuation of its properties (excluding the warehouse in note (ii) above). Land was valued at $12 million and the property at $48 million. The directors wish these values to be incorporated into the financial statements. The properties had an estimated remaining life of 20 years at the date of the valuation (straight-line depreciation is used). Harrington makes a transfer to realised profits in respect of the excess depreciation on revalued assets. Note: depreciation for the year to 31 March 2005 has not yet been accounted for in the draft financial statements. (iv) The investments are in quoted companies that are carried at their stock market values with any gains and losses recorded in the income statement. The value shown in the balance sheet is that at 31 March 2004 and during the year to 31 March 2005 the investments have risen in value by an average of 10%. Harrington has not reflected this increase in its financial statements. (v) On 1 October 2004 there had been a fully subscribed rights issue of 1 for 4 at 60c. This has been recorded in the above balance sheet. (vi) Income tax on the profits for the year to 31 March 2005 is estimated at $260,000. The figure in the income statement is the underprovision for income tax for the year to 31 March 2004. The carrying value of Harringtons net assets is $14 million more than their tax base at 31 March 2005. The income tax rate is 25%. Required: (a) Prepare a restated income statement for the year to 31 March 2005 reflecting the information in notes (i) to (vi) above. (9 marks) (b) Prepare a statement of changes in equity for the year to 31 March 2005. (6 marks)

(c) Prepare a restated balance sheet at 31 March 2005 reflecting the information in notes (i) to (vi) above. (10 marks) (25 marks) 5 [P.T.O.

(a) IFRS 1 First-time Adoption of International Financial Reporting Standards was issued in June 2003. Its main objectives are to ensure high quality information that is transparent and comparable over all periods presented and to provide a starting point for subsequent accounting under International Financial Reporting Standards (IFRS) within the framework of a cost benefit exercise. Required: (i) Describe the circumstances where the presentation of an entitys financial statements is deemed to be the first-time adoption of IFRSs and explain the main financial reporting implementation issues to be addressed in the transition to IFRSs. (7 marks) (ii) Describe IFRS 1s accounting requirements where an entitys previous accounting policies for assets and liabilities do not comply with the recognition and measurement requirements of IFRSs. (8 marks) (b) Transit, a publicly listed holding company, has a reporting date of 31 December each year. Its financial statements include one years comparatives. Transit currently applies local GAAP accounting rules, but is intending to apply IFRSs for the first time in its financial statements (including comparatives) for the year ending 31 December 2005. Its summarised consolidated balance sheet (under local GAAP) at 1 January 2004 is: $000 Property, plant and equipment Goodwill Development costs Current assets Inventory Trade receivables Bank Current liabilities Net current assets $000 1,000 450 400 1,850

150 250 20 420 (320) 100 1,950 (250) (300)

Non-current liabilities Restructuring provision Deferred tax

Issued share capital Retained earnings

(550) 1,400 500 900 1,400

Additional information: (i) Transits depreciation policy for its property, plant and equipment has been based on tax rules set by its government. If depreciation had been based on the most appropriate method under IFRSs, the carrying value of the property, plant and equipment at 1 January 2004 would have been $800,000.

(ii) The development costs originate from an acquired subsidiary of Transit. They do not qualify for recognition under IFRSs. They have a tax base of nil and the deferred tax related to these costs is $100,000. (iii) The inventory has been valued at prime cost. Under IFRSs it would include an additional $30,000 of overheads. (iv) The restructuring provision does not qualify for recognition under IFRSs. (v) Based on IFRSs, the deferred tax provision required at 1 January 2004, including the effects of the development expenditure, is $360,000. Required: Prepare a summarised balance sheet for Transit at the date of transition to IFRSs (1 January 2004) applying the requirements of IFRS 1 to the above items. Note: a reconciliation to previous GAAP is not required. (10 marks) (25 marks)

[P.T.O.

(a) Casino is a publicly listed company. Details of its balance sheets as at 31 March 2005 and 2004 are shown below together with other relevant information: Balance Sheet as at Non-current Assets (note (i)) Property, plant and equipment Intangible assets Current assets Inventory Trade receivables Interest receivable Short term deposits Bank Total assets Share Capital and Reserves Ordinary Shares of $1 each Reserves Share premium Revaluation reserve Retained earnings Non-current liabilities 12% loan note 8% variable rate loan note Deferred tax Current liabilities Trade payables Bank overdraft Taxation Total equity and liabilities The following supporting information is available: (i) Details relating to the non-current assets are: 31 March 2005 Cost/Valuation $m 600 440 31 March 2004 Carrying Carrying Depreciation value Cost/Valuation Depreciation value $m $m $m $m $m 12 588 500 80 420 148 292 445 105 340 880 760 31 March 2005 $m $m 880 400 1,280 350 808 5 32 15 31 March 2004 $m $m 760 510 1,270 420 372 3 120 75

1,210 2,490 300

990 2,260 200

60 112 1,098

1,270 1,570

nil 45 1,165

1,210 1,410

nil 160 90

250

150 nil 75

225

530 125 15

670 2,490

515 nil 110

625 2,260

Property, plant and equipment at:

Land and buildings Plant

Casino revalued the carrying value of its land and buildings by an increase of $70 million on 1 April 2004. On 31 March 2005 Casino transferred $3 million from the revaluation reserve to retained earnings representing the realisation of the revaluation reserve due to the depreciation of buildings. During the year Casino acquired new plant at a cost of $60 million and sold some old plant for $15 million at a loss of $12 million. There were no acquisitions or disposals of intangible assets. 8

(ii) The following extract is from the draft income statement for the year to 31 March 2005: $m $m Operating loss (32) Interest receivable 12 Finance costs (24) Loss before tax (44) Income tax repayment claim 14 Deferred tax charge (15) (1) Loss for the period (45) The finance costs are made up of: Interest expenses Penalty cost for early redemption of fixed rate loan Issue costs of variable rate loan

(16) (6) (2) (24)

(iii) The short term deposits meet the definition of cash equivalents. (iv) Dividends of $25 million were paid during the year. Required: As far as the information permits, prepare a cash flow statement for Casino for the year to 31 March 2005 in accordance with IAS 7 Cash Flow Statements. (20 marks) (b) In recent years many analysts have commented on a growing disillusionment with the usefulness and reliability of the information contained in some companies income statements. Required: Discuss the extent to which a companys cash flow statement may be more useful and reliable than its income statement. (5 marks) (25 marks)

[P.T.O.

Triangle, a public listed company, is in the process of preparing its draft financial statements for the year to 31 March 2005. The following matters have been brought to your attention: (i) On 1 April 2004 the company brought into use a new processing plant that had cost $15 million to construct and had an estimated life of ten years. The plant uses hazardous chemicals which are put in containers and shipped abroad for safe disposal after processing. The chemicals have also contaminated the plant itself which occurred as soon as the plant was used. It is a legal requirement that the plant is decontaminated at the end of its life. The estimated present value of this decontamination, using a discount rate of 8% per annum, is $5 million. The financial statements have been charged with $15 million ($15 million/10 years) for plant depreciation and a provision of $500,000 ($5 million/10 years) has been made towards the cost of the decontamination. (8 marks)

(ii) On 15 May 2005 the companys auditors discovered a fraud in the material requisitions department. A senior member of staff who took up employment with Triangle in August 2004 had been authorising payments for goods that had never been received. The payments were made to a fictitious company that cannot be traced. The member of staff was immediately dismissed. Calculations show that the total amount of the fraud to the date of its discovery was $240,000 of which $210,000 related to the year to 31 March 2005. (Assume the fraud is material). (5 marks) (iii) The company has contacted its insurers in respect of the above fraud. Triangle is insured for theft, but the insurance company maintains that this is a commercial fraud and is not covered by the theft clause in the insurance policy. Triangle has not yet had an opinion from its lawyers. (4 marks) (iv) On 1 April 2004 Triangle sold maturing inventory that had a carrying value of $3 million (at cost) to Factorall, a finance house, for $5 million. Its estimated market value at this date was in excess of $5 million. The inventory will not be ready for sale until 31 March 2008 and will remain on Triangles premises until this date. The sale contract includes a clause allowing Triangle to repurchase the inventory at any time up to 31 March 2008 at a price of $5 million plus interest at 10% per annum compounded from 1 April 2004. The inventory will incur storage costs until maturity. The cost of storage for the current year of $300,000 has been included in trade receivables (in the name of Factorall). If Triangle chooses not to repurchase the inventory, Factorall will pay the accumulated storage costs on 31 March 2008. The proceeds of the sale have been debited to the bank and the sale has been included in Triangles sales revenue. (8 marks) Required: Explain how the items in (i) to (iv) above should be treated in Triangles financial statements for the year to 31 March 2005 in accordance with current international accounting standards. Your answer should quantify the amounts where possible. The mark allocation is shown against each of the four matters above. (25 marks)

End of Question Paper

10

Answers

Part 2 Examination Paper 2.5 (INT) Financial Reporting (International Stream) 1 (a) (i) Goodwill calculation Investments at cost Cash consideration (80 x 75% x $350) Deferred consideration (see below) Less Ordinary shares (75% x 80) Share premium (75% x 40) Pre acq profit (see working) Fair value adjustments: brand (75% x 40) land and buildings (75% x 20) Goodwill on acquisition Impairment at 31 March 2005 (from question) Goodwill at 31 March 2005 $m $m 210 100 310 60 30 87 30 15

June 2005 Answers

(222) 88 (22) 66

The deferred consideration of $108 million must be discounted for one year at a cost of capital of 8% to $100 million (108/108). The $8 million difference is the finance charge in the year to 31 March 2005 (see (iii)). Although the internally generated brand cannot be recognised in Samsons entity financial statements, it should be recognised in the consolidated balance sheet on the acquisition of Samson. This is because the method given in the question is an acceptable method of valuation and thus the brand can be reliably measured. The fair value adjustment for Samsons land and buildings on acquisition is $20 million. The subsequent increase in value of $4 million in the year to 31 March 2005 is treated as a revaluation. The minority interest in the fair value adjustments and revaluation is $16 million (25% of (24 + 40)) see below. (ii) Minority interest Ordinary shares (25% x 80) Share premium (25% x 40) Retained earnings (see working) Fair values (see (i) above) $m $m 20 10 41 16 87 80 48 350 36 6 392 8 22 (30) 362

(iii) Consolidated reserves: Share premium: Highveldt only Revaluation reserve: (45 + (75% x 4)) Retained earnings Highveldt from question Post acq profit of Samson (see working) Interest receivable (see below) Finance cost on deferred consideration (see (i) above) Impairment of goodwill Retained earnings in consolidated balance sheet

The intra group interest has not been recorded by Highveldt. To do so it would credit interest receivable (which increases the profit for the year) and debit cash (in transit). Working (Note: all figures in $million) The pre and post acquisition profits of Samson are calculated as follows: Pre Post Per question 134 76 Apportionment of development costs (18) (22) URP in inventory (6/3) (2) Amortisation of brand (40/10 years) (4) 116 48 Therefore minority interest is 25% x 164 = 41 Pre acquisition earnings are 75% x 116 = 87 Post acquisition earnings are 75% x 48 = 36

at 31 March 2005

164

13

(b)

The objective of consolidated financial statements is to show the financial performance and position of the group as if it was a single economic entity. There is a view that, as the entity financial statements of the parent company contain the investments in subsidiaries as non-current assets, they reflect the assets of the group as a whole. The more traditional view is that entity financial statements do not provide users with sufficient information about subsidiaries for them to make a reliable assessment of the performance of the group as a whole. The following illustrates benefits of consolidated financial statements: they identify the nature and classification of the subsidiarys assets. For example, the investment in a subsidiary may be almost entirely in intangible assets or conversely they may be substantially land and buildings. Such a distinction is of obvious importance to users. the amount of the subsidiarys debt could not be assessed from the parents entity financial statements. In effect the subsidiarys assets and liabilities are netted off when it is shown as an investment. This means group liquidity and gearing cannot be properly assessed. the cost of the investment does not reflect the size of a company. For example a parent company may show an investment in a subsidiary at a cost of $10 million. This may represent the purchase of a subsidiary that has $10 million of assets and no liabilities. Alternatively this could be a subsidiary that has $100 million in assets and $90 million of liabilities. Clearly the latter subsidiary would be a much larger company than the former. the cost of the investment may be a fair representation of its value at the date of purchase, but with the passage of time (assuming the subsidiary is profitable), its value will increase. This increase would not be reflected in the original cost, but it would be reflected in the consolidated net assets of the subsidiary (and the increase in group reserves). the cost of the investment might represent all of the ownership of the subsidiary or only just over half of it i.e. there would be no indication of the minority interest. To summarise, in the absence of a consolidated balance sheet, users would have no information on the current value of a subsidiary, its size, the composition of its net assets and how much of it was owned by the group.

(a)

Harrington: Restated income statement Year to 31 March 2005 Sales revenues (13,700 300 plant sale proceeds) Cost of sales (w (i)) Gross profit Operating expenses Investment income (1,320 1,200) Loan interest (25 + 25) Profit before tax Income tax expense (55 + 260 + (350 280) deferred tax) Profit for the period

$000 13,400 (8,910) 4,490 (2,400) 120 (50) 2,160 (385) 1,775 Ordinary shares $000 1,600 400 Share premium $000 40 560 Total $000 4,630 960 1,775 1,800 nil (500) 8,665

(b)

Statement of Changes in Equity Year to 31 March 2005 Retained Revaluation Profits reserve $000 $000 At 1 April 2004 2,990 nil Rights issue (see below) Profit for period (see (a)) 1,775 Revaluation of property (w (ii)) 1,800 Transfer to realised profit 80 (80) Ordinary dividends paid (500) At 31 March 2005 4,345 1,720

2,000

600

The number of 25c ordinary shares at the year end is 8 million ($2 million x 4). This is after a rights issue of 1 for 4. Thus the number of shares prior to the issue would be 64 million (8 million x 4/5) and the rights issue would have been for 16 million shares. The rights issue price is 60c each which would be recorded as an increase in share capital of $400,000 (16 million x 25c) and an increase in share premium of $560,000 (16 million x 35c).

14

(c)

Balance Sheet as at 31 March 2005 Non-current assets Property, plant and equipment (6,710 + 1,350 (w (ii))) Investments (1,200 x 110%) Current assets Inventory Trade receivables Bank Total assets Equity and liabilities: Ordinary shares of 25c each Reserves (see (b)): Share premium Revaluation reserve (w (ii)) Retained earnings (from (b)) Non-current liabilities 10% loan note (issued 2002) Deferred tax (1,400 x 25%) Current liabilities Trade payables Accrued loan interest ((500 x 10%) 25 paid) Current tax payable Total equity and liabilities Workings (all figures in $000): (i) Cost of sales: per question profit on sale of plant ((900 630) 300) depreciation plant (w (iii)) buildings (w (ii)) capitalised expenses net of error (w (ii))

$000

$000 8,060 1,320 9,380

1,750 2,450 350

4,550 13,930 2,000

600 1,720 4,345

6,665 8,665

500 350 4,130 25 260

850

4,415 13,930

9,200 (30) 450 290 (1,000) 8,910 cost/revaluation 1,000 6,000 7,000 depreciation 50 (20 year life) 240 (see below) 290

(ii)

Land and buildings: Self constructed (see below) Revalued

The carrying value of the land and buildings at 31 March 2005 is $6,710,000 (7,000 290). Depreciation on the building element will be $240 (4,800/20 years). The revaluation of the land and buildings will create a revaluation reserve initially of $1,800 (6,000 (1,000 + (4,000 800)), however a transfer of $80 (1,600/20 building element of the revaluation) to realised profit is required. Self constructed asset: purchased materials direct labour supervision design and planning costs error in construction (10 + 25) 150 800 65 20 (35) 1,000

Note: the cost of the error cannot be capitalised; it must therefore be written off.

15

(iii) Plant per balance sheet disposal

cost 5,200 (900) 4,300

depreciation 31 March 2004 3,130 (630) 2,500

carrying value

1,800

Depreciation for the current year will be $450,000 (25% reducing balance), giving a net book value at 31 March 2005 of $1,350,000.

(a)

(i)

It is important to determine which financial statements constitute the first-time adoption of International Financial Reporting Standards (IFRSs) because IFRS 1 only applies to those financial statements. It does not apply to subsequent financial statements that are prepared under IFRSs as these must be prepared under the whole body of IFRSs (including IASs). A first time adopter is an entity that makes an explicit and unreserved statement that its financial statements comply with (all) IFRSs. Compliance with some, but not all, IFRSs is insufficient as is compliance with all IFRSs without the inclusion of an explicit statement of compliance. The main issues to be addressed in the transition to IFRSs are: deciding the date of transition. selecting accounting policies that comply with IFRSs. preparing (but not presenting) an opening balance sheet at the date of transition to IFRSs as a basis for subsequent accounting. The date of transition to IFRSs is the beginning of the earliest comparative period. If the companys reporting date for its first IFRS financial statements is say 31 December 2005 and it intends to disclose one years comparatives, then the date of transition is 1 January 2004. determining estimates of values for both the opening IFRS balance sheet and all other presented comparative balance sheets; and finally presenting IFRS financial statements (including the required disclosures relating to the transition). Note: some credit would be given for references to practical issues such as required changes to information systems and the recruitment of personnel with IFRS experience. When a company adopts IFRSs for the first time it must use the IFRSs that are in force at the reporting date. These IFRSs should be applied throughout all of the periods presented normally two years. This means that comparative financial statements must comply with the IFRSs in force at the reporting date even if they were not in issue at the date of the comparatives (or different from those that were in issue). The general principle is that the adoption of IFRSs should be applied retrospectively i.e. as if the entity had always applied each IFRS. This is contrary to the specific transitional requirements contained in some individual IFRSs (they often allow prospective application). More specifically an entity must: recognise all assets and liabilities that fall to be recognised by IFRSs (e.g. IFRSs require deferred tax to be recognised on a full provision basis) not recognise assets and liabilities if IFRSs do not permit their recognition (e.g. IFRSs do not permit general provisions to be made and some proposed dividends are not treated as liabilities) reclassify certain items under IFRSs (e.g. convertible debt must be split between its equity and debt components under IFRSs, but in some jurisdictions it may have been classed entirely as debt) apply IFRS rules to measure the value of all recognised assets and liabilities. This may mean for example using a nondiscounted value to measure an item whereas under previous GAAP it may have been discounted give specified reconciliations for equity and reported profits between previous GAAP and IFRSs, together with details of any recognition or reversals of impairments when implementing first time reporting. The resulting adjustments arising from the above must be recognised directly in equity at the date of transition. The Standard contains some specific exemptions and exceptions to the above in the areas of values of property, plant and equipment, employee benefits, translation differences for net investments in foreign operations and derivative instruments. In essence the exemptions and exceptions allow the use of alternative value measures in limited circumstances and where the cost or effort required in determining the value measurements under IFRSs would be excessive. The exemptions are permitted but not required and can be applied in full or individually, whereas exceptions (to retrospective application) are mandatory. A particularly important exemption is that the IFRS rules governing business combinations are not applied retrospectively. However any intangible that was recognised under previous GAAP that cannot be recognised under IFRSs should be reclassified as goodwill and the entity must test for impairment of goodwill at the date of transition.

(ii)

16

(b)

As Transit intends first-time adoption for the year ended 31 December 2005 and to disclose one years comparatives, the date of transition to IFRSs is 1 January 2004. Transit summarised balance sheet at 1 January 2004 $000 Property, plant and equipment (w (i)) Goodwill (450 + 300 (w (ii))) Development costs (w (ii)) Current assets Inventory (150 + 30 (w (i))) Receivables Bank Total assets Issued share capital Retained earnings (w (v)) Current liabilities Trade and other payables Non-current liabilities Restructuring provision (w (iii)) Deferred tax (w (iv)) Total equity and liabilities $000 800 750 nil 1,550

180 250 20

450 2,000 500 820 1,320 320 nil 360 2,000

In addition the goodwill will be tested for impairment and if there is any indication of impairment to the other identifiable assets, they too must be tested for impairment. Workings (all figures in $000) (i) Property, plant and equipment and inventory are restated to their IFRS base valuations of $800,000 and $180,000 respectively. (ii) Acquired intangible assets that do not qualify for recognition under IFRSs must be reclassified as goodwill after allowing for the effect of deferred tax. Thus an amount of $300 (400 100 re deferred tax) is transferred to goodwill and $100 debited to deferred tax. (iii) The restructuring provision does not qualify for recognition under IFRSs. (iv) Summarising the effect on deferred tax: per question 300 re development costs (100) 200 required balance (360) charge to retained earnings (160) (v) Summarising the effect on retained earnings: per question 900 property, plant and equipment (200) inventory 30 elimination of restructuring provision 250 deferred tax (160) 820

17

(a)

Cash Flow Statement of Casino for the Year to 31 March 2005: $m Cash inflows from operating activities Operating loss Adjustments for: Depreciation buildings (w (i)) plant (w (ii)) intangibles (510 400) Loss on disposal of plant (from question) Operating profit before working capital changes Decrease in inventory (420 350) Increase in trade receivables (808 372) Increase in trade payables (530 515) Cash generated from operations Interest paid Income tax paid (w (iii)) Net cash outflow from operating activities Cash flows from investing activities Purchase of land and buildings (w (i)) plant (w (ii)) Sale of plant (w (ii)) Interest received (12 5 + 3) Cash flows from financing activities Issue of ordinary shares (100 + 60) Issue of 8% variable rate loan (160 2 issue costs) Repayments of 12% loan (150 + 6 penalty) Dividends paid (110) (60) 15 10 $m (32) 12 81 110 12

215 183 70 (436) 15 (168) (16) (81) (265)

(145)

137 Net decrease in cash and cash equivalents (273) Cash and cash equivalents at beginning of period (120 + 75) 195 Cash and cash equivalents at end of period (125 (32 +15)) (78) Interest and dividends received and paid may be shown as operating cash flows or as investing or financing activities as appropriate. Workings (in $ million) (i) Land and buildings net book value b/f revaluation gains depreciation for year net book value c/f difference is cash purchases (ii) Plant: cost b/f additions from question balance c/f difference is cost of disposal loss on disposal proceeds difference accumulated depreciation of plant disposed of depreciation b/f less disposal (above) depreciation c/f charge for year

160 158 (156) (25)

420 70 (12) (588) (110) 445 60 (440) 65 (12) (15) 38 105 (38) (148) (81)

18

(iii) Taxation: tax provision b/f deferred tax b/f income statement net charge tax provision c/f deferred tax c/f difference is cash paid (iv) Revaluation reserve: balance b/f revaluation gains transfer to retained earnings balance c/f (v) Retained earnings: balance b/f loss for period dividends paid transfer from revaluation reserve balance c/f (b)

(110) (75) (1) 15 90 (81) 45 70 (3) 112 1,165 (45) (25) 3 1,098

The accruals/matching concept applied in preparing an income statement has the effect of smoothing cash flows for reporting purposes. This practice arose because interpreting raw cash flows can be very difficult and the accruals process has the advantage of helping users to understand the underlying performance of a company. For example if an item of plant with an estimated life of five years is purchased for $100,000, then in the cash flow statement for the five year period there would be an outflow in year 1 of the full $100,000 and no further outflows for the next four years. Contrast this with the income statement where by applying the accruals principle, depreciation of the plant would give a charge of $20,000 per annum (assuming straight-line depreciation). Many would see this example as an advantage of an income statement, however it is important to realise that profit is affected by many subjective items. This has led to accusations of profit manipulation or creative accounting, hence the disillusionment of the usefulness of the income statement. Another example of the difficulty in interpreting cash flows is that counterintuitively a decrease in overall cash flows is not always a bad thing (it may represent an investment in increasing capacity which would bode well for the future), nor is an increase in cash flows necessarily a good thing (this may be from the sale of non-current assets because of the need to raise cash urgently). The advantages of cash flows are: it is difficult to manipulate cash flows, they are real and possess the qualitative characteristic of objectivity (as opposed to subjective profits). cash flows are an easy concept for users to understand, indeed many users misinterpret income statement items as being cash flows. cash flows help to assess a companys liquidity, solvency and financial adaptability. Healthy liquidity is vital to a companys going concern. many business investment decisions and company valuations are based on projected cash flows. the quality of a companys operating profit is said to be confirmed by closely correlated cash flows. Some analysts take the view that if a company shows a healthy operating profit, but has low or negative operating cash flows, there is a suspicion of profit manipulation or creative accounting.

19

(i)

Future decontamination costs must be provided for in full at the time they become unavoidable. Where they are based on future values, they should be discounted to their present value (as has been done in this example). Rather than being immediately written off to the income statement, the decontamination costs are added to the cost of the related asset and amortised over the expected life of the asset. The current treatment of these costs by Triangle is incorrect. The depreciation charge must be based on the full cost of the plant which must include the decontamination costs. Also an imputed finance cost must be applied to the provision (often referred to as unwinding). Applying this, the extracts of the financial statements of Triangle at 31 March 2005 would be: Non-current assets Plant at cost ($15 million + $5 million) Depreciation at 10% per annum $million 200 (20) 180 50 04 54 20 04

Non-current liabilities Provision Accrued finance costs

Income statement Depreciation Accrued finance costs ($5 million x 8%) (ii)

This is an example of an adjusting event after the balance sheet date. To some extent the figures in the draft financial statements already reflect the effects of the fraud (up to the amount at the year end i.e. $210,000) in that presumably the cost of the materials paid for are included in cost of sales. However, the financial statements are incorrect in their presentation. As the fraud is considered material, $210,000 should be removed from the cost of sales and included as an income statement operating expense (perhaps with separate disclosure). This will affect the gross profit and other ratios, though it will not affect the net profit. The further costs beyond the year end of $30,000 should be noted as a non-adjusting event (if material in their own right).

(iii) Triangle is of the opinion that the cost of the fraud may be covered by an insurance claim. However the insurance company is disputing the claim. This appears to be a contingent asset. If a contingent asset is probable it should be noted in the financial statements. However if it is only possible, it should be ignored. As this claim is at an early stage and the company has not yet sought a legal opinion, it would be premature to consider the claim probable. In these circumstances the contingent asset should be ignored and the financial statements will be unaffected. (iv) Although this transaction has been treated as a sale, this is probably not its substance. The clause allowing Triangle to repurchase the inventory makes this a sale and repurchase agreement. Assuming Triangle acts rationally it will repurchase the inventory if its retail value at 31 March 2008 is more than $7,320,500 ($5 million plus compound interest at 10% for four years) plus the accumulated storage costs (as these can be recovered from Factorall in the event that the inventory is not repurchased). There is no indication in the question as to what the inventory is likely to be worth on 31 March 2008. However it is unlikely that a finance company will really want to acquire this inventory (it is not its normal line of business) and thus it would not have entered into the contract unless it believed Triangle would repurchase the inventory. If the above is correct the substance of the transaction is that it is a secured loan rather than a sale. The required adjustments would therefore be: Remove $5 million from sales (debit) and treat this as a long term (4 year) loan. Remove $3 million from cost of sales and treat this as inventory. The receivable for the storage cost should be removed from trade receivables and added to the cost of the inventory. Accrued interest of $500,000 ($5 million x 10%) should be charged to the income statement and added to the carrying value of the loan.

20

Part 2 Examination Paper 2.5 (INT) Financial Reporting (International Stream)

June 2005 Marking Scheme

This marking scheme is given as a guide in the context of the suggested answers. Scope is given to markers to award marks for alternative approaches to a question, including relevant comment, and where well-reasoned conclusions are provided. This is particularly the case for written answers where there may be more than one acceptable solution. 1 (a) (i) goodwill consideration given share capital and premium pre acq profit fair value adjustments goodwill impairment maximum (ii) minority interest share capital and premium retained earnings fair value adjustment maximum (iii) consolidated reserves share premium revaluation reserve retained earnings post acq profit interest receivable finance cost goodwill impairment maximum (b) 1 mark per relevant point to maximum Maximum for question Marks 2 1 2 2 1 8 1 2 1 4 1 2 2 1 1 1 8 5 25

(a)

Restated income statement sales revenue cost of sales operating expenses investment income loan interest income tax available maximum

1 5 1 1 1 2 11 9 1 1 1 2 1 1 7 6

(b)

statement of changes in equity brought forward figures rights issue (restated) profit for the financial year surplus on land and buildings transfer to realised profits dividend paid available maximum

21

Marks (c) Balance sheet land and buildings plant investments inventory and trade receivables bank and trade payables accrued loan interest current tax 10% loan note deferred tax share capital and premium revaluation reserve retained earnings available maximum Maximum for question 1 1 1 1 1 1 1 1 1 1 1 1 12 10 25

(a)

(i) (ii)

one mark per valid point to max one mark per valid point to max

7 8 1 3 1 3 1 2 11 10 25

(b)

Property, plant and equipment development costs/goodwill inventory retained earnings restructuring provision eliminated deferred tax available maximum Maximum for question

(a)

cash flows from operating activities operating loss depreciation and loss on sale adjustments working capital items interest paid income tax investing activities financing 1 mark per item cash and cash equivalents b/f and c/f available maximum

1 4 3 1 2 7 4 1 23 20 5 25

(b)

1 mark per relevant point to

maximum Maximum for question

22

(i)

explanation of treatment of provision cost of plant at $20 million revised depreciation provision initially at $5 million increase by finance cost income statement charges 1 each maximum an example of an adjusting event no overall effect on profit, but presentation incorrect remove from cost of sales and show as an expense $30,000 is a non-adjusting event if material disclose as a note to the financial statements maximum

Marks 2 1 1 1 1 2 8 1 1 1 1 1 5 1 1 2 4 1 1 2 1 2 2 9 8 25

(ii)

(iii) due to the dispute this is an example of a contingent asset describe the treatment of contingent assets not probable therefore ignore, financial statements unchanged maximum (iv) identify it as a sale and repurchase agreement (or financing arrangement) substance is not likely to be a sale will repurchase if value is more than $7,320,500 plus storage costs business of Factorall is financing therefore terms likely to favour repurchase adjustments to sales/loan; cost of sales/inventory; trade receivables/inventory (re storage costs); accrued finance costs/loan available maximum Maximum for question

23

Financial Reporting
(International Stream)
PART 2 THURSDAY 8 JUNE 2006

QUESTION PAPER Time allowed 3 hours This paper is divided into two sections Section A This ONE question is compulsory and MUST be answered THREE questions ONLY to be answered

Section B

Do not open this paper until instructed by the supervisor This question paper must not be removed from the examination hall

The Association of Chartered Certified Accountants

Paper 2.5(INT)

Section A This ONE question is compulsory and MUST be attempted 1 On 1 October 2005 Hydan, a publicly listed company, acquired a 60% controlling interest in Systan paying $9 per share in cash. Prior to the acquisition Hydan had been experiencing difficulties with the supply of components that it used in its manufacturing process. Systan is one of Hydans main suppliers and the acquisition was motivated by the need to secure supplies. In order to finance an increase in the production capacity of Systan, Hydan made a non-dated loan at the date of acquisition of $4 million to Systan that carried an actual and effective interest rate of 10% per annum. The interest to 31 March 2006 on this loan has been paid by Systan and accounted for by both companies. The summarised draft financial statements of the companies are: Income statements for the year ended 31 March 2006 Hydan $000 98,000 (76,000) 22,000 (11,800) 350 (420) 10,130 (4,200) 5,930 Hydan $000 Non-current assets Property, plant and equipment Investments (including loan to Systan) Current assets Total assets Equity and liabilities Ordinary shares of $1 each Share premium Retained earnings Non-current liabilities 7% Bank loan 10% loan from Hydan Current liabilities Total equity and liabilities The following information is relevant: (i) At the date of acquisition, the fair values of Systans property, plant and equipment were $12 million in excess of their carrying amounts. This will have the effect of creating an additional depreciation charge (to cost of sales) of $300,000 in the consolidated financial statements for the year ended 31 March 2006. Systan has not adjusted its assets to fair value. 18,400 16,000 34,400 18,000 52,400 10,000 5,000 20,000 35,000 6,000 nil 11,400 52,400 Systan pre-acquisition post-acquisition $000 $000 24,000 35,200 (18,000) (31,000) 6,000 4,200 (1,200) (8,000) nil nil nil (200) 4,800 (4,000) (1,200) 1,000 3,600 (3,000) Systan $000 9,500 nil 9,500 7,200 16,700 2,000 500 6,300 8,800 nil 4,000 3,900 16,700

Revenue Cost of sales Gross profit Operating expenses Interest income Finance costs Profit/(loss) before tax Income tax (expense)/relief Profit/(loss) for the period Balance sheets as at 31 March 2006

(ii) In the post acquisition period Systans sales to Hydan were $30 million on which Systan had made a consistent profit of 5% of the selling price. Of these goods, $4 million (at selling price to Hydan) were still in the inventory of Hydan at 31 March 2006. Prior to its acquisition Systan made all its sales at a uniform gross profit margin. (iii) Included in Hydans current liabilities is $1 million owing to Systan. This agreed with Systans receivables ledger balance for Hydan at the year end. (iv) An impairment review of the consolidated goodwill at 31 March 2006 revealed that its current value was 125% less than its carrying amount. (v) Neither company paid a dividend in the year to 31 March 2006. Required: (a) Prepare the consolidated income statement for the year ended 31 March 2006 and the consolidated balance sheet at that date. (20 marks) (b) Discuss the effect that the acquisition of Systan appears to have had on Systans operating performance. (5 marks) (25 marks)

[P.T.O.

Section B THREE questions ONLY to be attempted 2 The following trial balance relates to Darius at 31 March 2006: $000 Revenue Cost of sales Closing inventories 31 March 2006 (note (i)) Operating expenses Rental income from investment property Finance costs (note (ii)) Land and building at valuation (note (iii)) Plant and equipment cost (note (iii)) Investment property valuation 1 April 2005 (note (iii)) Accumulated depreciation 1 April 2005 plant and equipment Joint venture (note (iv)) Trade receivables Bank Trade payables Ordinary shares of 25c each 10% Redeemable preference shares of $1 each Deferred tax (note (v)) Revaluation reserve (note (iii)) Retained earnings 1 April 2005 143,800 10,500 22,400 1,200 5,000 63,000 36,000 16,000 16,800 8,000 13,500 900 11,800 20,000 10,000 5,200 21,000 17,500 318,200 $000 213,800

318,200 The following notes are relevant: (i)

An inventory count at 31 March 2006 listed goods with a cost of $105 million. This includes some damaged goods that had cost $800,000. These would require remedial work costing $450,000 before they could be sold for an estimated $950,000.

(ii) Finance costs include overdraft charges, the full years preference dividend and an ordinary dividend of 4c per share that was paid in September 2005. (iii) Non-current assets: Land and building The land and building were revalued at $15 million and $48 million respectively on 1 April 2005 creating a $21 million revaluation reserve. At this date the building had a remaining life of 15 years. Depreciation is on a straight-line basis. Darius does not make a transfer to realised profits in respect of excess depreciation. Plant All plant, including that of the joint venture (note (iv)), is depreciated at 125% on the reducing balance basis. Depreciation on both the building and the plant should be charged to cost of sales. Investment property On 31 March 2006 a qualified surveyor valued the investment property at $135 million. Darius uses the fair value model in IAS 40 Investment property to value its investment property.

(iv) On 1 April 2005 Darius entered into a joint venture with two other entities. Each venturer contributes their own assets and is responsible for their own expenses including depreciation on joint venture assets. Darius is entitled to 40% of the joint ventures total revenues. The joint venture is not a separate entity. Details of Dariuss joint venture transactions are: Plant and equipment at cost Share of joint venture revenue (40% of total sales revenue) Related joint venture cost of sales excluding depreciation Trade receivables Trade payables Net balance included in the above list of balances $000 12,000 (8,000) 5,000 1,500 (2,500) 8,000

(v) The directors have estimated the provision for income tax for the year ended 31 March 2006 at $8 million. The deferred tax provision at 31 March 2006 is to be adjusted (through the income statement) to reflect that the tax base of the companys net assets is $12 million less than their carrying amounts. The rate of income tax is 30%. Required: (a) Prepare the income statement for Darius for the year ended 31 March 2006. (10 marks)

(b) Prepare the statement of recognised income and expense for Darius for the year ended 31 March 2006. (2 marks) (c) Prepare the balance sheet for Darius as at 31 March 2006. Notes to the financial statements are not required. (25 marks) (13 marks)

[P.T.O.

This is a blank page. Question 3 begins on page 7.

(a) The IASBs Framework for the preparation and presentation of financial statements (Framework) sets out the concepts that underlie the preparation and presentation of financial statements that external users are likely to rely on when making economic decisions about an enterprise. Required: Explain the purpose and authoritative status of the Framework. (5 marks)

(b) Of particular importance within the Framework are the definitions and recognition criteria for assets and liabilities. Required: Define assets and liabilities and explain the important aspects of their definitions. Explain why these definitions are of particular importance to the preparation of an entitys balance sheet and income statement. (8 marks) (c) Peterlee is preparing its financial statements for the year ended 31 March 2006. The following items have been brought to your attention: (i) Peterlee acquired the entire share capital of Trantor during the year. The acquisition was achieved through a share exchange. The terms of the exchange were based on the relative values of the two companies obtained by capitalising the companies estimated future cash flows. When the fair value of Trantors identifiable net assets was deducted from the value of the company as a whole, its goodwill was calculated at $25 million. A similar exercise valued the goodwill of Peterlee at $4 million. The directors wish to incorporate both the goodwill values in the companies consolidated financial statements. (4 marks)

(ii) During the year Peterlee acquired an iron ore mine at a cost of $6 million. In addition, when all the ore has been extracted (estimated in 10 years time) the company will face estimated costs for landscaping the area affected by the mining that have a present value of $2 million. These costs would still have to be incurred even if no further ore was extracted. The directors have proposed that an accrual of $200,000 per year for the next ten years should be made for the landscaping. (4 marks) (iii) On 1 April 2005 Peterlee issued an 8% $5 million convertible loan at par. The loan is convertible in three years time to ordinary shares or redeemable at par in cash. The directors decided to issue a convertible loan because a non-convertible loan would have required an interest rate of 10%. The directors intend to show the loan at $5 million under non-current liabilities. The following discount rates are available: Year 1 Year 2 Year 3 Required: Describe (and quantify where possible) how Peterlee should treat the items in (i) to (iii) in its financial statements for the year ended 31 March 2006 commenting on the directors views where appropriate. The mark allocation is shown against each of the three items above. (25 marks) 8% 093 086 079 10% 091 083 075

(4 marks)

[P.T.O.

Shown below are the summarised financial statements for Boston, a publicly listed company, for the years ended 31 March 2005 and 2006, together with some segment information analysed by class of business for the year ended 31 March 2006 only: Income statements: Carpeting $m 90 (30) 60 (25) 35 Hotels $m 130 (95) 35 (15) 20 Total Total House building 31 March 2006 31 March 2005 $m $m $m 280 500 450 (168) (293) (260) 112 207 190 (32) (72) (60) 80 135 130 (60) (50) 75 80 (10) (5) 65 75 (25) (30) 40 45 Total Total House building 31 March 2006 31 March 2005 $m $m $m 200 380 332 75 155 130 275 535 462 15 nil 550 462 100 20 232 352 25 108 65 nil 550 80 nil 192 272 30 115 40 5 462

Revenue Cost of sales (note (i)) Gross profit Operating expenses Segment result Unallocated corporate expenses Profit from operations Finance costs Profit before tax Income tax expense Profit for the period Balance sheets:

Tangible non-current assets Current assets Segment assets Unallocated bank balance Consolidated total assets Ordinary share capital Share premium Retained earnings

Carpeting $m 40 40 80

Hotels $m 140 40 180

Segment current liabilities tax other Unallocated loans Unallocated bank overdraft

4 4

9 51

12 53

Consolidated equity and total liabilities The following notes are relevant (i)

Depreciation for the year to 31 March 2006 was $35 million. During the year a hotel with a carrying amount of $40 million was sold at a loss of $12 million. Depreciation and the loss on the sale of non-current assets are charged to cost of sales. There were no other non-current asset disposals. As part of the companys overall acquisition of new non-current assets, the hotel segment acquired $104 million of new hotels during the year.

(ii) The above figures are based on historical cost values. The fair values of the segment net assets are: Carpeting $m 80 97 Hotels $m 150 240 House building $m 250 265

at 31 March 2005 at 31 March 2006

(iii) The following ratios (which can be taken to be correct) have been calculated based on the overall group results: Year ended: Return on capital employed Gross profit margin Operating profit margin Net assets turnover Current ratio Gearing 31 March 2006 180% 414% 15% 12 times 13:1 156% 31 March 2005 256% 422% 178% 14 times 09:1 128%

(iv) The following segment ratios (which can be taken to be correct) have been calculated for the year ended 31 March 2006 only: Carpeting Hotels House building Segment return on net assets 486% 167% 381% Segment asset turnover (times) 13 11 13 Gross profit margin 667% 269% 40% Net profit margin 389% 154% 286% Current ratio (excluding bank) 5:1 07:1 12:1 Required: (a) Prepare a cash flow statement for Boston for the year ended 31 March 2006. Note: you are not required to show separate segmental cash flows or any disclosure notes. (b) Using the ratios provided, write a report to the Board of Boston analysing the companys financial performance and position for the year ended 31 March 2006. (15 marks) Your answer should make reference to your cash flow statement and the segmental information and consider the implication of the fair value information. (25 marks) (10 marks)

[P.T.O.

(a) Torrent is a large publicly listed company whose main activity involves construction contracts. Details of three of its contracts for the year ended 31 March 2006 are: Contract Date commenced Estimated duration Fixed contract price Estimated costs at start of contract Cost to date: at 31 March 2005 at 31 March 2006 Estimated costs at 31 March 2006 to complete Progress payments received at 31 March 2005 (note (i)) Progress payments received at 31 March 2006 (note (i)) Notes (i) The companys normal policy for determining the percentage completion of contracts is based on the value of work invoiced to date compared to the contract price. Progress payments received represent 90% of the work invoiced. However, no progress payments will be invoiced or received from contract Ceta until it is completed, so the percentage completion of this contract is to be based on the cost to date compared to the estimated total contract costs. Alfa 1 April 2004 3 years $m 20 15 5 125 (note (ii)) 35 54 126 Beta 1 October 2005 18 months Ceta 1 October 2005 2 years $m 12 10 nil 4 6 nil nil

$m 6 75 (note (iii)) nil 2 55 (note (iii)) nil 18

(ii) The cost to date of $125 million at 31 March 2006 for contract Alfa includes $1 million relating to unplanned rectification costs incurred during the current year (ended 31 March 2006) due to subsidence occurring on site. (iii) Since negotiating the price of contract Beta, Torrent has discovered the land that it purchased for the project is contaminated by toxic pollutants. The estimated cost at the start of the contract and the estimated costs to complete the contract include the unexpected costs of decontaminating the site before construction could commence. Required: Prepare extracts of the income statement and balance sheet for Torrent in respect of the above construction contracts for the year ended 31 March 2006 (12 marks) (b) (i) The issued share capital of Savoir, a publicly listed company, at 31 March 2003 was $10 million. Its shares are denominated at 25 cents each. Savoirs earnings attributable to its ordinary shareholders for the year ended 31 March 2003 were also $10 million, giving an earnings per share of 25 cents. Year ended 31 March 2004 On 1 July 2003 Savoir issued eight million ordinary shares at full market value. On 1 January 2004 a bonus issue of one new ordinary share for every four ordinary shares held was made. Earnings attributable to ordinary shareholders for the year ended 31 March 2004 were $13,800,000. Year ended 31 March 2005 On 1 October 2004 Savoir made a rights issue of shares of two new ordinary shares at a price of $100 each for every five ordinary shares held. The offer was fully subscribed. The market price of Savoirs ordinary shares immediately prior to the offer was $240 each. Earnings attributable to ordinary shareholders for the year ended 31 March 2005 were $19,500,000. Required: Calculate Savoirs earnings per share for the years ended 31 March 2004 and 2005 including comparative figures. (9 marks)

10

(ii) On 1 April 2005 Savoir issued $20 million 8% convertible loan stock at par. The terms of conversion (on 1 April 2008) are that for every $100 of loan stock, 50 ordinary shares will be issued at the option of loan stockholders. Alternatively the loan stock will be redeemed at par for cash. Also on 1 April 2005 the directors of Savoir were awarded share options on 12 million ordinary shares exercisable from 1 April 2008 at $150 per share. The average market value of Savoirs ordinary shares for the year ended 31 March 2006 was $250 each. The income tax rate is 25%. Earnings attributable to ordinary shareholders for the year ended 31 March 2006 were $25,200,000. The share options have been correctly recorded in the income statement. Required: Calculate Savoirs basic and diluted earnings per share for the year ended 31 March 2006 (comparative figures are not required). You may assume that both the convertible loan stock and the directors options are dilutive. (4 marks) (25 marks)

End of Question Paper

11

Answers

Part 2 Examination Paper 2.5(INT) Financial Reporting (International Stream) 1 (a) Hydan Consolidated income statement year ended 31 March 2006 $000 Revenue (98,000 + 35,200 30,000 intra-group sales) Cost of sales (w (i)) Gross profit Operating expenses (11,800 + 8,000 + 375 goodwill (w (ii))) Interest receivable (350 200 intra-group (4,000 x 10% x 6/12)) Finance costs Income tax expense (4,200 1,000 tax relief) Profit for the period Attributable to: Equity holders of the parent Minority interest (w (iv))

June 2006 Answers

$000 103,200 (77,500) 25,700 (20,175) 150 (420) 5,255 (3,200) 2,055 3,455 (1,400) 2,055

Consolidated balance sheet as at 31 March 2006 Non-current assets: Property, plant and equipment (18,400 + 9,500 + 1,200 300 depreciation adjustment) Goodwill (3,000 375 (w (ii))) Investment (16,000 10,800 4,000 loan) Current assets (w (v)) Total assets Equity attributable to holders of the parent Ordinary shares of $1 each Share premium Retained earnings (w (iii)) Minority interest (w (iv)) Total equity Non-current liabilities 7% bank loan Current liabilities (w (v)) Total equity and liabilities Workings in $000 (i) Cost of sales Hydan Systan Intra-group sales URP in inventories Additional depreciation re fair values

28,800 2,625 1,200 32,625 24,000 56,625 10,000 5,000 17,525 32,525 3,800 36,325 6,000 14,300 56,625

76,000 31,000 (30,000) 200 300 77,500 10,800 2,000 500 9,300 1,200 13,000 x 60%

(ii)

Goodwill/Cost of control in Systan: Investment at cost (2,000 x 60% x $9) Less ordinary shares of Systan share premium pre-acquisition reserves (6,300 + 3,000 post acq loss) fair value adjustment

Goodwill on consolidation Goodwill is impaired by 125% of its carrying amount =

(7,800) 3,000 375

15

(iii) Consolidated reserves: Hydans reserves Systans post acquisition losses (see below) (3,500 x 60%) Goodwill impairment (w (ii))

20,000 (2,100) (375) 17,525 6,300 (200) (300) (500) 5,800 3,000 500 3,500 x 40% = 1,400 2,500 5,800 1,200 9,500 x 40% = 3,800

The adjusted profits of Systan are: Per question Adjustments URP in inventories (4,000 x 5%) additional depreciation

(iv) Minority interest in income statement Systans post acquisition loss after tax Adjustments from (w (iii)) Adjusted losses Minority interest in balance sheet Ordinary shares and premium of Systan Adjusted profits (w (iii)) Fair value adjustments

(v)

Current assets and liabilities Current assets: Hydan Systan URP in inventories Intra-group balance

18,000 7,200 (200) (1,000) 24,000 11,400 3,900 (1,000) 14,300

Current liabilities: Hydan Systan Intra-group balance

(b)

Although Systans revenue has increased since its acquisition by Hydan, its operating performance appears to have deteriorated markedly. Its gross profit margin has fallen from 25% (6m/24m) in the six months prior to the acquisition to only 119% (42m/352m) in the post-acquisition period. The decline in gross profit is worsened by a huge increase in operating expenses in the post-acquisition period. These have gone from $12 million pre-acquisition to $8 million post-acquisition. Taking into account the effects of interest and tax a $36 million first half profit (pre-acquisition) has turned into a $3 million second half loss (post-acquisition). Whilst it is possible that some of the worsening performance may be due to market conditions, the major cause is probably due to the effects of the acquisition. As the question states Hydan has acquired a controlling interest in Systan and thus the two companies are related parties. Since the acquisition most of Systans sales have been to Hydan. This is not surprising as Systan was acquired to secure supplies to Hydan. The terms under which the sales are made are now determined by the management of Hydan, whereas they were previously determined by the management of Systan. The question says sales to Hydan yield a consistent gross profit of only 5%. This is very low and much lower than the profit margin on sales to Hydan prior to the acquisition and also much lower than the few sales that were made to third parties in the post acquisition period. It may also be that Hydan has shifted the burden of some of the group operating expenses to Systan this may explain the large increase in Systans post acquisition operating expenses. The effect of these (transfer pricing) actions would move profits from Systans books into those of Hydan. The implications of this are quite significant. Initially there may be a tendency to think the effect is not important as on consolidation both companies results are added together, but other parties are affected by these actions. The most obvious is the significant (40%) minority interest, they are effectively having some of their share of Systans profit and balance sheet value taken from them. It may also be that the management and staff of Systan may be losing out on profit related bonuses. Finally, any party using Systans entity financial statements, for whatever purpose, would be basing any decisions they make on potentially misleading information.

16

(a)

Darius income statement for the year ended 31 March 2006 $000 Revenue (w (i)) Cost of sales (w (i)) Gross profit Operating expenses Investment income Loss on investment property (16,000 13,500 w (ii)) Financing cost (5,000 3,200 ordinary dividend (w (v)) Profit before tax Income tax expense (w (iii)) Profit for the period $000 221,800 (156,200) 65,600 (22,400) 1,200 (2,500) (1,800) 40,100 (6,400) 33,700

(b)

Statement of recognised income and expense for the year ended 31 March 2006 Unrealised surplus on land and building Profit for the period Total recognised income and expense for the period

21,000 33,700 54,700

(c)

Darius balance sheet as at 31 March 2006 Non-current assets Property, plant and equipment (w (iv)) Investment property (w (ii)) Current assets Inventories (10,500 300 (w (i))) Trade receivables (13,500 + 1,500 JV) Total assets Equity and liabilities: Ordinary shares of 25c each Reserves: Revaluation Retained earnings (w (v)) Non-current liabilities Deferred tax (w (iii)) Redeemable preference shares of $1 each Current liabilities Trade payables (11,800 + 2,500 JV) Bank overdraft Current tax payable Total equity and liabilities Workings in $000 (i) Revenue Per question Joint venture revenue

87,100 13,500 100,600 10,200 15,000

25,200 125,800 20,000

21,000 48,000

69,000 89,000

3,600 10,000 14,300 900 8,000

13,600

23,200 125,800

213,800 8,000 221,800 143,800 300 5,000 3,200 3,900 156,200

Cost of sales: Per question Closing inventories adjustment (see below) Joint venture costs Depreciation (w (iv)) building plant

The damaged inventories will require expenditure of $450,000 to repair them and then have an expected selling price of $950,000. This gives a net realisable value of $500,000, as their cost was $800,000, a write down of $300,000 is required.

17

(ii)

The fair value model in IAS 40 Investment property requires investment properties to be included in the balance sheet at their fair value (in this case taken to be the open market value). Any surplus or deficit is recorded in income. 8,000 (1,600) 6,400

(iii) Taxation: Provision for year Deferred tax (see below)

Taxable temporary differences are $12 million. At a rate of 30% this would require a balance sheet provision for deferred tax of $36 million. The opening provision is $52 million, thus a credit of $16 million will be made in the income statement. (iv) Non-current assets Land and building Depreciation of the building for the year ended 31 March 2006 will be (48,000/15 years) Plant and equipment Per trial balance Joint venture plant Accumulated depreciation 1 April 2005 Carrying amount prior to charge for year Depreciation year ended 31 March 2006 at 125% Carrying amount at 31 March 2006 Summarising: Land and building Plant and equipment Property, plant and equipment (v) cost/valuation 63,000 48,000 111,000 3,200 36,000 12,000 48,000 (16,800) 31,200 (3,900) 27,300 accumulated depreciation carrying amount 3,200 59,800 20,700 27,300 23,900 87,100 17,500 33,700 (3,200) 48,000

Retained earnings Balance b/f Profit for period Ordinary dividends paid (20,000 x 4 x 4c)

(a)

The purpose of the Framework is to assist the various bodies and users that may be interested in the financial statements of an entity. It is there to assist the IASB itself, other standard setters, preparers, auditors and users of financial statements and any other party interested in the work of the IASB. More specifically: to assist the Board in the development of new and the review of existing standards. It is also believed that the Framework will assist in promoting harmonisation of the preparation of financial statements and also reduce the number of alternative accounting treatments permitted by IFRSs national standard setters that have expressed a desire for local standards to be compliant with IFRS will be assisted by the Framework the Framework will help preparers to apply IFRS more effectively if they understand the concepts underlying the Standards, additionally the Framework should help in dealing with new or emerging issues which are, as yet, not covered by an IFRS the above is also true of the work of the auditor, in particular the Framework can assist the auditor in determining whether the financial statements conform to IFRS users should be assisted by the Framework in interpreting the performance of entities that have complied with IFRS.

It is important to realise that the Framework is not itself an accounting standard and thus cannot override a requirement of a specific standard. Indeed, the Board recognises that there may be (rare) occasions where a particular IFRS is in conflict with the Framework. In these cases the requirements of the standard should prevail. The Board believes that such conflicts will diminish over time as the development of new and (revised) existing standards will be guided by the Framework and the Framework itself may be revised based on the experience of working with it.

18

(b)

Definitions assets: The IASBs Framework defines assets as a resource controlled by an entity as a result of past events and from which future economic benefits are expected to flow to the entity. The first part of the definition puts the emphasis on control rather than ownership. This is done so that the balance sheet reflects the substance of transactions rather than their legal form. This means that assets that are not legally owned by an entity, but over which the entity has the rights that are normally conveyed by ownership, are recognised as assets of the entity. Common examples of this would be finance leased assets and other contractual rights such as aircraft landing rights. An important aspect of control of assets is that it allows the entity to restrict the access of others to them. The reference to past events prevents assets that may arise in future from being recognised early. liabilities: The IASBs Framework defines liabilities as a present obligation of the entity arising from past events, the settlement of which is expected to result in an outflow from the entity of resources embodying economic benefits. Many aspects of this definition are complementary (as a mirror image) to the definition of assets, however the IASB stresses that the essential characteristic of a liability is that the entity has a present obligation. Such obligations are usually legally enforceable (by a binding contract or by statute), but obligations also arise where there is an expectation (by a third party) of an entity assuming responsibility for costs where there is no legal requirement to do so. Such obligations are referred to as constructive (by IAS 37 Provisions, contingent liabilities and contingent assets). An example of this would be repairing or replacing faulty goods (beyond any warranty period) or incurring environmental costs (e.g. landscaping the site of a previous quarry) where there is no legal obligation to do so. Where entities do incur constructive obligations it is usually to maintain the goodwill and reputation of the entity. One area of difficulty is where entities cannot be sure whether an obligation exists or not, it may depend upon a future uncertain event. These are more generally known as contingent liabilities. Importance of the definitions of assets and liabilities: The definitions of assets and liabilities are fundamental to the Framework. Apart from forming the obvious basis for the preparation of a balance sheet, they are also the two elements of financial statements that are used to derive the equity interest (ownership) which is the residue of assets less liabilities. Assets and liabilities also have a part to play in determining when income (which includes gains) and expenses (which include losses) should be recognised. Income is recognised (in the income statement) when there is an increase in future economic benefits relating to increases in assets or decreases in liabilities, provided they can be measured reliably. Expenses are the opposite of this. Changes in assets and liabilities arising from contributions from, and distributions to, the owners are excluded from the definitions of income and expenses. Currently there is a great deal of concern over off balance sheet finance. This is an aspect of what is commonly referred to as creative accounting. Many recent company failure scandals have been in part due to companies having often massive liabilities that have not been included on the balance sheet. Robust definitions, based on substance, of assets and liabilities in particular should ensure that only real assets are included on the balance sheet and all liabilities are also included. In contradiction to the above point, there have also been occasions where companies have included liabilities on their balance sheets where they do not meet the definition of liabilities in the Framework. Common examples of this are general provisions and accounting for future costs and losses (usually as part of the acquisition of a subsidiary). Companies have used these general provisions to smooth profits i.e. creating a provision when the company has a good year (in terms of profit) and releasing them to boost profits in a bad year. Providing for future costs and losses during an acquisition may effectively allow them to bypass the income statement as they would become part of the goodwill figure.

(c)

(i)

Whilst it is acceptable to value the goodwill of $25 million of Trantor (the subsidiary) on the basis described in the question and include it in the consolidated balance sheet, the same treatment cannot be afforded to Peterlees own goodwill. The calculation may indeed give a realistic value of $4 million for Peterlees goodwill, and there may be no difference in nature between the goodwill of the two companies, but it must be realised that the goodwill of Peterlee is internal goodwill and IFRSs prohibit such goodwill appearing in the financial statements. The main basis of this conclusion is one of reliable measurement. The value of acquired (purchased) goodwill can be evidenced by the method described in the question (there are also other acceptable methods), but this method of valuation is not acceptable as a basis for recognising internal goodwill. Accruing for future costs such as this landscaping on an annual basis may seem appropriate and was common practice until recently. However, it is no longer possible to account for this type of future cost in this manner, therefore the directors suggestion is unacceptable. IAS 37 Provisions, contingent liabilities and contingent assets requires such costs to be accounted for in full as soon as they become unavoidable. The Standard says that the estimate of the future cost should be discounted to a present value (as in this example at $2 million). The accounting treatment is rather controversial; the cost should be included in the balance sheet as a provision (a credit entry/balance), but the debit is to the cost of the asset to give an initial carrying amount of $8 million. This has the effect of grossing up the balance sheet by including the landscaping costs as both an asset and a liability. As the asset is depreciated on a systematic basis ($800,000 per annum assuming straight-line depreciation), the landscaping costs are charged to the income statement over the life of the asset. As the discount is unwound (and charged as a finance cost) this is added to the balance sheet provision such that, at the date when the liability is due to be settled, the provision is equal to the amount due (assuming estimates prove to be accurate).

(ii)

(iii) The directors suggestion that the convertible loan should be recorded as a liability of the full $5 million is incorrect. The reason why a similar loan without the option to convert to equity shares (such that it must be redeemed by cash only) carries a higher interest rate is because of the value of the equity option that is contained within the issue proceeds of the $5 million. If the company performs well over the period of the loan, the value of its equity shares should rise and

19

thus it would (probably) be beneficial for the loan note holders to opt for the equity share alternative. IAS 32 and 39 dealing with financial instruments require the value of the option is to be treated as equity rather than debt. The calculation of value of the equity is as follows: Year 1 400 x 091 Year 2 400 x 083 Year 3 5,400 x 075 Present value of the cash flows Proceeds of issue Difference is value of equity $000 364 332 4,050 4,746 (5,000) 254

Initially the loan would be shown at $4,746,000. The Income statement would show: Loan interest paid ($5m x 8%) Accrued finance costs (balance) $000 400 75 $000 475 (i.e. $4746m x 10%)

At 31 March 2006 the loan would have a carrying amount of $4,821,000 ($4,746,000 + $75,000)

(a)

Boston Cash Flow Statement for the year ended 31 March 2006: Cash flows from operating activities Note: figures in brackets are in $000 Profit before tax Adjustments for: depreciation of non-current assets loss on sale of hotel interest expense increase in current assets (155 130) decrease in other current liabilities (115 108) Cash generated from operations Interest paid (see note) Income taxes paid Net cash flow from operating activities Cash flows from investing activities: purchase of non-current assets (see below) sale of non-current assets (40 12) Net cash used in investing activities Cash flows from financing activities Issue of ordinary shares (20 + 20) Issue of loans (65 40) Net cash from financing activities Net increase in cash and cash equivalents Cash and cash equivalents at beginning of period Cash and cash equivalents at end of period Workings Non-current assets carrying amount Balance b/f Disposal Depreciation for year Balance c/f Cost of assets acquired $000 332 (40) (35) (380) (123) 40 25 65 20 (5) 15 (123) 28 (95) $000 65 35 12 10 122 (25) (7) 90 (10) (30) 50 $000

Note: interest paid may also be presented as a cash flow from financing activities.

20

(b)

Report on the financial performance of Boston for the year ended 31 March 2006 To: The Board of Boston From: A N Other Date: Profitability (note figures are rounded to 1 decimal place) The most striking feature of the current years performance is the deterioration in the ROCE, down from 256% to only 180%. This represents an overall fall in profitability of 30% ((256 180)/256 x 100). An examination of the other ratios provided shows that this is due to a decline in both profit margins and asset utilisation. A closer look at the profit margins shows that the decline in gross margin is relatively small (422% down to 414%), whereas the fall in the operating profit margin is down by 28%, this represents a 157% decline in profitability (i.e. 28% on 178%). This has been caused by increases in operating expenses of $12m and unallocated corporate expenses of $10m. These increases represent more than half of the net profit for the period and further investigation into the cause of these increases should be made. The company is generating only $120 of sales per $1 of net balance sheet assets this year compared to a figure of $140 in the previous year. This decline in asset utilisation represents a fall of 143% ((14 1.2)/14 x 100). Liquidity/solvency From the limited information provided a poor current ratio of 09:1 in 2005 has improved to 13:1 in the current year. Despite the improvement, it is still below the accepted norm. At the same time gearing has increased from 128% to 156%. Information from the cash flow statement shows the company has raised $65 million in new capital ($40m in equity and $25m in loans). The disproportionate increase in the loans is the cause of the increase in gearing, however, at 156% this is still not a highly geared company. The increase in finance has been used mainly to purchase new non-current assets, but it has also improved liquidity, mainly by reversing an overdraft of $5 million to a bank balance in hand of $15 million. A common feature of new investment is that there is often a delay between making the investment and benefiting from the returns. This may be the case with Boston, and it may be that in future years the increased investment will be rewarded with higher returns. Another aspect of the investment that may have caused the lower return on assets is that the investment is likely to have occurred part way through the year (maybe even near the year end). This means that the income statement may not include returns for a full year, whereas in future years it will. Segment issues Segment information is intended to help the users to better assess the performance of an enterprise by looking at the detailed contribution made by the differing activities that comprise the enterprise as a whole. Referring to the segment ratios it appears that the carpeting segment is giving the greatest contribution to overall profitability achieving a 486% return on its segment assets, whereas the equivalent return for house building is 381% and for hotels it is only 167%. The main reason for the better return from carpeting is due to its higher segment net profit margin of 389% compared to hotels at 154% and house building at 286%. Carpetings higher segment net profit is in turn a reflection of its underlying very high gross margin (667%). The segment net asset turnover of the hotels (11 times) is also very much lower than the other two segments (13 times). It seems that the hotel segment is also responsible for the groups fairly poor liquidity ratios (ignoring the bank balances) the segment current liabilities are 50% greater than its current assets ($60m compared to $40m); the opposite of this would be a more acceptable current ratio. These figures are based on historical values. Most commentators argue that the use of fair values is more consistent and thus provides more reliable information on which to base assessments (they are less misleading than the use of historical values). If fair values are used all segments understandably show lower returns and poorer performance (as fair values are higher than historical values), but the figures for the hotels are proportionately much worse, falling by a half of the historic values (as the fair values of the hotel segment are exactly double the historical values). Fair value adjusted figures may even lead one to question the future of the hotel activities. However, before jumping to any conclusions an important issue should be considered. Although the reported profit of the hotels is poor, the market values of its segment assets have increased by a net $90 million. New net investment in hotel capital expenditure is $64 million ($104m $40m disposal); this leaves an increase in value of $26 million. The majority of this appears to be from market value increases (this would be confirmed if the statement of recognised income and expense was available). Whilst this is not a realised profit, it is nevertheless a significant and valuable gain (equivalent to 65% of the group reported net profit). Conclusion Although the companys overall performance has deteriorated in the current year, it is clear that at least some areas of the business have had considerable new investment which may take some time to bear fruit. This applies to the hotel segment in particular and may explain its poor performance, which is also partly offset by the strong increase in the market value of its assets. Yours A N other Appendix Further segment ratios Return on net assets at fair values (35/97 x 100) Asset turnover on fair values (times) (90/97) Carpeting 361% 09 Hotels 83% 05 House building 302% 11

Note: workings have been shown for the figures for the carpeting segment only, the other segments figures are based on equivalent calculations.

21

(a)

Income statement for the year ended 31 March 2006 Alfa $m 8 (7) 1 24 (13) 125 25 (126) 24 35 (15) (15) (18) (13) 40 08 (nil) 48 Beta $m 20 (35) (15) Ceta $m 48 (40) 08 48 Total $m 148 (145) 03 72 (13) 200 18 (15) (144) 72 (13)

Revenue Cost of sales Profit/(loss) Balance sheet as at 31 March 2006 Gross amounts due from customers (see below) Gross amounts due to customers (see below) Gross amounts from and to customers: Contract cost incurred Recognised profits less (losses) Provision for losses to date Payments received Due from customers Due to customers (contract liability) Workings (in $m): Alfa Work invoiced (54/90%) Cost of sales (balancing figure)

at 31 March 2005 at 31 March 2006 Year ended 31 March 2006 60 (126/90%) 140 8 (45) (115) (7) Profit (see below) 15 25 1 Percentage complete (6/20 x 100) 30% (14/20 x 100) 70% Attributable profit ($5m x 30%) 15 (($5m x 70%) $1m rectification) 25 Prior to the rectification costs (which must be charged to the year in which they are incurred), the estimated total profit on the contract is $5 million ($20m $15m). Beta Due to the increase in the estimated cost Beta is a loss-making contract and the whole of the loss must be provided for as soon as it is can be anticipated. The loss is expected to be $15 million ($75m $6m). The sales value of the contract at 31 March 2006 is $2 million ($18/90%), thus the cost of sales must be recorded as $35 million. As costs to date are $2 million, this means a provision of $15 million is required. Ceta Based on the costs to date at 31 March 2006 of $4 million and the total estimated costs of $10 million, this contract is 40% complete. The estimated profit is $2 million ($12m $10m); therefore the profit at 31 March 2006 is $08 million ($2m x 40%). This gives an imputed sales (and receivable) value of $48 million. (b) (i) Savoir EPS year ended 31 March 2004: The issue on 1 July 2003 at full market value needs to be weighted: New shares 40m x 3/12 = 8m 48m x 9/12 = 10m 36m 46m

Without the bonus issue this would give an EPS of 30c ($138m/46m x 100). The bonus issue of one for four would result in 12 million new shares giving a total number of ordinary shares of 60 million. The dilutive effect of the bonus issue would reduce the EPS to 24c (30c x 48m/60m). The comparative EPS (for 2003) would be restated at 20c (25c x 48m/60m).

22

EPS year ended 31 March 2005: The rights issue of two for five on 1 October 2004 is half way through the year. The theoretical ex rights value can be calculated as: Holder of Subscribes for Now holds Weighting: Rights issue (2 for 5) New total Weighted average 60m x 6/12 x 240/200 = 24m 84m x 6/12 = 36 million 42 million 78 million 100 shares worth $240 = 40 shares at $1 each = 140 worth (in theory) $240 $40 $280

i.e. $2 each.

EPS is therefore 25c ($195m/78m x 100). The comparative (for 2004) would be restated at 20c (24c x 200/240). (ii) The basic EPS for the year ended 31 March 2006 is 30c ($252m/84m x 100). Dilution Convertible loan stock On conversion loan interest of $12 million after tax would be saved ($20 million x 8% x (100% 25%)) and a further 10 million shares would be issued ($20m/$100 x 50). Directors options Options for 12 million shares at $150 each would yield proceeds of $18 million. At the average market price of $250 per share this would purchase 72 million shares ($18m/$250). Therefore the bonus element of the options is 48 million shares (12m 72m). Using the above figures the diluted EPS for the year ended 31 March 2006 is 267c ($252m + $12m)/(84m + 10m + 48m)).

23

Part 2 Examination Paper 2.5(INT) Financial Reporting (International Stream)

June 2006 Marking Scheme

This marking scheme is given as a guide in the context of the suggested answers. Scope is given to markers to award marks for alternative approaches to a question, including relevant comment, and where well-reasoned conclusions are provided. This is particularly the case for written answers where there may be more than one definitive solution. Marks 1 (a) Income statement: revenue cost of sales operating expenses including 1 mark for goodwill interest receivable/payable income tax minority interest Balance sheet: goodwill tangible non-current assets investments current assets/current liabilities 7% bank loan elimination of 10% intra group loan minority interest share capital and share premium retained earnings available maximum ( b ) 1 mark per relevant point to maximum Maximum for question 2 3 2 1 1 2 3 2 1 2 1 1 2 1 1 25 20 5 25

(a)

Income statement revenue cost of sales operating costs investment income loss on investment finance costs income tax available maximum

1 4 1 1 1 2 2 12 10

(b)

Statement of recognised income and expense surplus on land and buildings profit for period maximum

1 1 2

(c)

Balance sheet property, plant and equipment investment property inventories trade receivables share capital revaluation reserve retained earnings (1 for dividend deduction) overdraft trade payables current tax payable deferred tax preference shares (shown as a liability) available maximum Maximum for question

4 1 1 1 1 1 2 1 1 1 1 1 16 13 25

25

(a) (b) (c)

one mark per valid point to max one mark per valid point to max (i) acceptable method to value goodwill of subsidiary although no difference in nature of goodwill; cannot use this method to value goodwill of parent IFRS specifically prohibits recognition of internal goodwill main issue is reliable measurement available maximum (ii) IAS 37 requires immediate recognition (as a provision) of such costs future costs discounted to present value added to cost of the asset (increases depreciation) unwinding of discount is a finance cost maximum (iii) directors suggestion is incorrect, part of proceeds is equity IFRS require equity component to be calculated (residual equity method) Debt is $4,746,000; equity is $254,000 Income statement charge is $475,000 Non-current liability at 31 March 2006 $4,821,000 available maximum Maximum for question

Marks 5 8 1 1 1 1 1 5 4 1 1 1 1 4 1 1 1 1 1 5 4 25

(a)

profit before tax depreciation loss on sale of hotel increase in current assets decrease in current liabilities interest paid income taxes paid purchase of non-current assets sale of non-current assets share issue issue of loan cash and cash equivalents b/f and c/f available maximum

1 1 1 1 1 1 1 1 1 1 1 1 12 10 15 25

(b)

one mark per valid point to max Maximum for question

26

Marks 5 (a) Income statement revenue cost of sales/profit balance sheet gross amounts due from customers provision available maximum (b) (i) EPS year year year year ended ended ended ended 31 31 31 31 March March March March 2004 2004 comparative 2005 2005 comparative available maximum (ii) EPS year ended 31 March 2006 basic EPS dilution effect of options effect of convertible loan stock calculation maximum Maximum for question 4 4 4 1 13 12 4 1 4 1 10 9 1 1 1 1 4 25

27

Financial Reporting
(International Stream)
PART 2 THURSDAY 7 DECEMBER 2006

QUESTION PAPER Time allowed 3 hours This paper is divided into two sections Section A This ONE question is compulsory and MUST be answered THREE questions ONLY to be answered

Section B

Do not open this paper until instructed by the supervisor This question paper must not be removed from the examination hall

The Association of Chartered Certified Accountants

Paper 2.5(INT)

Section A This ONE question is compulsory and MUST be attempted 1 Hosterling purchased the following equity investments: On 1 October 2005: 80% of the issued share capital of Sunlee. The acquisition was through a share exchange of three shares in Hosterling for every five shares in Sunlee. The market price of Hosterlings shares at 1 October 2005 was $5 per share. On 1 July 2006: 6 million shares in Amber paying $3 per share in cash and issuing to Ambers shareholders 6% (actual and effective rate) loan notes on the basis of $100 loan note for every 100 shares acquired. The summarised income statements for the three companies for the year ended 30 September 2006 are: Hosterling $000 105,000 (68,000) 37,000 400 (4,000) (7,500) (1,200) 24,700 (8,700) 16,000 Sunlee $000 62,000 (36,500) 25,500 nil (2,000) (7,000) (900) 15,600 (2,600) 13,000 Amber $000 50,000 (61,000) (11,000) nil (4,500) (8,500) nil (24,000) 4,000 (20,000)

Revenue Cost of sales Gross profit/(loss) Other income (note (i)) Distribution costs Administrative expenses Finance costs Profit/(loss) before tax Income tax (expense)/credit Profit/(loss) for the period The following information is relevant: (i)

The other income is a dividend received from Sunlee on 31 March 2006.

(ii) The details of Sunlees and Ambers share capital and reserves at 1 October 2005 were: Sunlee $000 20,000 18,000 Amber $000 15,000 35,000

Equity shares of $1 each Retained earnings

(iii) A fair value exercise was carried out at the date of acquisition of Sunlee with the following results: carrying amount $000 18,000 17,000 30,000 fair value $000 22,000 20,000 35,000 remaining life (straight line)

Intellectual property Land Plant

still in development not applicable five years

The fair values have not been reflected in Sunlees financial statements. Plant depreciation is included in cost of sales. No fair value adjustments were required on the acquisition of Amber. (iv) In the year ended 30 September 2006 Hosterling sold goods to Sunlee at a selling price of $18 million. Hosterling made a profit of cost plus 25% on these sales. $75 million (at cost to Sunlee) of these goods were still in the inventories of Sunlee at 30 September 2006. (v) Impairment tests for both Sunlee and Amber were conducted on 30 September 2006. They concluded that the goodwill of Sunlee should be written down by $16 million and, due to its losses since acquisition, the investment in Amber was worth $215 million. (vi) All trading profits and losses are deemed to accrue evenly throughout the year.

Required: (a) Calculate the goodwill arising on the acquisition of Sunlee at 1 October 2005. (5 marks)

(b) Calculate the carrying amount of the investment in Amber at 30 September 2006 under the equity method prior to the impairment test. (4 marks) (c) Prepare the consolidated income statement for the Hosterling Group for the year ended 30 September 2006. (16 marks) (25 marks)

[P.T.O.

Section B THREE questions ONLY to be attempted 2 The following trial balance relates to Tadeon, a publicly listed company, at 30 September 2006: $000 Revenue Cost of sales Operating expenses Loan interest paid (note (i)) Rental of vehicles (note (ii)) Investment income 25 year leasehold property at cost (note (iii)) Plant and equipment at cost Investments at amortised cost Accumulated depreciation at 1 October 2005 leasehold property plant and equipment Equity shares of 20 cents each fully paid Retained earnings at 1 October 2005 2% Loan note (note (i)) Deferred tax balance 1 October 2005 (note (iv)) Trade receivables Inventories at 30 September 2006 Bank Trade payables Suspense account (note (v)) 118,000 40,000 1,000 6,200 2,000 225,000 181,000 42,000 36,000 85,000 150,000 18,600 50,000 12,000 53,500 33,300 1,900 18,700 48,000 700,000 $000 277,800

700,000

The following notes are relevant: (i) The loan note was issued on 1 October 2005. It is redeemable on 30 September 2010 at a large premium (in order to compensate for the low nominal interest rate). The finance department has calculated that the effective interest rate on the loan is 55% per annum.

(ii) The rental of the vehicles relates to two separate contracts. These have been scrutinised by the finance department and they have come to the conclusion that $5 million of the rentals relate to a finance lease. The finance lease was entered into on 1 October 2005 (the date the $5 million was paid) for a four year period. The vehicles had a fair value of $20 million (straight-line depreciation should be used) at 1 October 2005 and the lease agreement requires three further annual payments of $6 million each on the anniversary of the lease. The interest rate implicit in the lease is to be taken as 10% per annum. (Note: you are not required to calculate the present value of the minimum lease payments.) The other contract is an operating lease and should be charged to operating expenses. Other plant and equipment is depreciated at 121/2% per annum on the reducing balance basis. All depreciation of property, plant and equipment is charged to cost of sales. (iii) On 30 September 2006 the leasehold property was revalued to $200 million. The directors wish to incorporate this valuation into the financial statements. (iv) The directors have estimated the provision for income tax for the year ended 30 September 2006 at $38 million. At 30 September 2006 there were $74 million of taxable temporary differences, of which $20 million related to the revaluation of the leasehold property (see (iii) above). The income tax rate is 20%. (v) The suspense account balance can be reconciled from the following transactions: The payment of a dividend in October 2005. This was calculated to give a 5% yield on the companys share price of 80 cents as at 30 September 2005. The net receipt in March 2006 of a fully subscribed rights issue of one new share for every three held at a price of 32 cents each. The expenses of the share issue were $2 million and should be charged to share premium. Note: the cash entries for these transactions have been correctly accounted for. 4

Required: Prepare for Tadeon: (a) An income statement for the year ended 30 September 2006; and (b) A balance sheet as at 30 September 2006. Note: A statement of changes in equity is not required. Disclosure notes are not required. (25 marks) (8 marks) (17 marks)

[P.T.O.

(a) Recording the substance of transactions, rather than their legal form, is an important principle in financial accounting. Abuse of this principle can lead to profit manipulation, non-recognition of assets and substantial debt not being recorded on the balance sheet. Required: Describe how the use of off balance sheet financing can mislead users of financial statements. Note: your answer should refer to specific user groups and include examples where recording the legal form of transactions may mislead them. (9 marks) (b) Angelino has entered into the following transactions during the year ended 30 September 2006: (i) In September 2006 Angelino sold (factored) some of its trade receivables to Omar, a finance house. On selected account balances Omar paid Angelino 80% of their book value. The agreement was that Omar would administer the collection of the receivables and remit a residual amount to Angelino depending upon how quickly individual customers paid. Any balance uncollected by Omar after six months will be refunded to Omar by Angelino. (5 marks)

(ii) On 1 October 2005 Angelino owned a freehold building that had a carrying amount of $75 million and had an estimated remaining life of 20 years. On this date it sold the building to Finaid for a price of $12 million and entered into an agreement with Finaid to rent back the building for an annual rental of $13 million for a period of five years. The auditors of Angelino have commented that in their opinion the building had a market value of only $10 million at the date of its sale and to rent an equivalent building under similar terms to the agreement between Angelino and Finaid would only cost $800,000 per annum. Assume any finance costs are 10% per annum. (6 marks) (iii) Angelino is a motor car dealer selling vehicles to the public. Most of its new vehicles are supplied on consignment by two manufacturers, Monza and Capri, who trade on different terms. Monza supplies cars on terms that allow Angelino to display the vehicles for a period of three months from the date of delivery or when Angelino sells the cars on to a retail customer if this is less than three months. Within this period Angelino can return the cars to Monza or can be asked by Monza to transfer the cars to another dealership (both at no cost to Angelino). Angelino pays the manufacturers list price at the end of the three month period (or at the date of sale if sooner). In recent years Angelino has returned several cars to Monza that were not selling very well and has also been required to transfer cars to other dealerships at Monzas request. Capris terms of supply are that Angelino pays 10% of the manufacturers price at the date of delivery and 1% of the outstanding balance per month as a display charge. After six months (or sooner if Angelino chooses), Angelino must pay the balance of the purchase price or return the cars to Capri. If the cars are returned to the manufacturer, Angelino has to pay for the transportation costs and forfeits the 10% deposit. Because of this Angelino has only returned vehicles to Capri once in the last three years. (5 marks) Required: Describe how the above transactions and events should be treated in the financial statements of Angelino for the year ended 30 September 2006. Your answer should explain, where relevant, the difference between the legal form of the transactions and their substance. Note: The mark allocation is shown against each of the three transactions above. (25 marks)

This is a blank page. Question 4 begins on page 8.

[P.T.O.

Minster is a publicly listed company. Details of its financial statements for the year ended 30 September 2006, together with a comparative balance sheet, are: Balance Sheet at Non-current assets (note (i)) Property, plant and equipment Software Investments at fair value through profit and loss Current assets Inventories Trade receivables Amounts due from construction contracts Bank Total assets Equity and liabilities Equity shares of 25 cents each Reserves Share premium (note (ii)) Revaluation reserve Retained earnings Non-current liabilities 9% loan note Environmental provision Deferred tax Current liabilities Trade payables Bank overdraft Current tax payable Total equity and liabilities Income statement for the year ended 30 September 2006 Revenue Cost of sales Gross profit Operating expenses Finance costs (note (i)) Investment income and gain on investments Profit before tax Income tax expense Profit for the year The following supporting information is available: (i) Included in property, plant and equipment is a coal mine and related plant that Minster purchased on 1 October 2005. Legislation requires that in ten years time (the estimated life of the mine) Minster will have to landscape the area affected by the mining. The future cost of this has been estimated and discounted at a rate of 8% to a 30 September 2006 $000 $000 1,280 135 150 1,565 480 270 80 nil 510 380 55 35 30 September 2005 $000 $000 940 nil 125 1,065

830 2,395 500

980 2,045 300

150 60 950

1,160 1,660

85 25 965

1,075 1,375

120 162 18 350 25 60

300

nil nil 25 555 40 50

25

435 2,395

645 2,045 1,397 (1,110) 287 (125) 162 (40) 20 142 (57) 85

present value of $150,000. This cost has been included in the carrying amount of the mine and, together with the unwinding of the discount, has also been treated as a provision. The unwinding of the discount is included within finance costs in the income statement. Other land was revalued (upward) by $35,000 during the year. Depreciation of property, plant and equipment for the year was $255,000. There were no disposals of property, plant and equipment during the year. The software was purchased on 1 April 2006 for $180,000. The market value of the investments had increased during the year by $15,000. There have been no sales of these investments during the year. (ii) On 1 April 2006 there was a bonus (scrip) issue of equity shares of one for every four held utilising the share premium reserve. A further cash share issue was made on 1 June 2006. No shares were redeemed during the year. (iii) A dividend of 5 cents per share was paid on 1 July 2006. Required: (a) Prepare a cash flow statement for Minster for the year to 30 September 2006 in accordance with IAS 7 Cash flow statements. (15 marks) (b) Comment on the financial performance and position of Minster as revealed by the above financial statements and your cash flow statement. (10 marks) (25 marks)

[P.T.O.

(a) (i)

State the definition of both non-current assets held for sale and discontinued operations and explain the usefulness of information for discontinued operations. (4 marks)

Partway is in the process of preparing its financial statements for the year ended 31 October 2006. The companys main activity is in the travel industry mainly selling package holidays (flights and accommodation) to the general public through the Internet and retail travel agencies. During the current year the number of holidays sold by travel agencies declined dramatically and the directors decided at a board meeting on 15 October 2006 to cease marketing holidays through its chain of travel agents and sell off the related high-street premises. Immediately after the meeting the travel agencies staff and suppliers were notified of the situation and an announcement was made in the press. The directors wish to show the travel agencies results as a discontinued operation in the financial statements to 31 October 2006. Due to the declining business of the travel agents, on 1 August 2006 (three months before the year end) Partway expanded its Internet operations to offer car hire facilities to purchasers of its Internet holidays. The following are Partways summarised income statement results years ended: 31 October 2006 Internet travel agencies car hire $000 $000 $000 23,000 14,000 2,000 (18,000) (16,500) (1,500) 5,000 (2,500) 500 (1,000) (1,500) (100) 4,000 (4,000) 400 31 October 2005 total total $000 $000 39,000 40,000 (36,000) (32,000) 3,000 8,000 (2,600) (2,000) 400 6,000

Revenue Cost of sales Gross profit/(loss) Operating expenses Profit/(loss) before tax

The results for the travel agencies for the year ended 31 October 2005 were: revenue $18 million, cost of sales $15 million and operating expenses of $15 million. Required: (ii) Discuss whether the directors wish to show the travel agencies results as a discontinued operation is justifiable. (4 marks) (iii) Assuming the closure of the travel agencies is a discontinued operation, prepare the (summarised) income statement of Partway for the year ended 31 October 2006 together with its comparatives. Note: Partway discloses the analysis of its discontinued operations on the face of its income statement. (6 marks) (b) (i) Describe the circumstances in which an entity may change its accounting policies and how a change should be applied. (5 marks)

The terms under which Partway sells its holidays are that a 10% deposit is required on booking and the balance of the holiday must be paid six weeks before the travel date. In previous years Partway has recognised revenue (and profit) from the sale of its holidays at the date the holiday is actually taken. From the beginning of November 2005, Partway has made it a condition of booking that all customers must have holiday cancellation insurance and as a result it is unlikely that the outstanding balance of any holidays will be unpaid due to cancellation. In preparing its financial statements to 31 October 2006, the directors are proposing to change to recognising revenue (and related estimated costs) at the date when a booking is made. The directors also feel that this change will help to negate the adverse effect of comparison with last years results (year ended 31 October 2005) which were better than the current years. Required: (ii) Comment on whether Partways proposal to change the timing of its recognition of its revenue is acceptable and whether this would be a change of accounting policy. (6 marks) (25 marks) End of Question Paper 10

Answers

Part 2 Examination Paper 2.5(INT) Financial Reporting (International Stream) 1 (a) Cost of control in Sunlee: Consideration Shares (20,000 x 80% x 3/5 x $5) Less Equity shares Pre acq reserves Fair value adjustments (4,000 + 3,000 + 5,000)

December 2006 Answers

$000

$000 48,000

20,000 18,000 12,000 50,000 x 80%

Goodwill (b) Carrying amount of Amber 30 September 2006 (prior to impairment loss): At cost Cash (6,000 x $3) 6% loan notes (6,000 x $100/100) Less Post acquisition losses (20,000 x 40% x 3/12)

(40,000) 8,000

$000 18,000 6,000 24,000 (2,000) 22,000

(c)

Hosterling Group Consolidated income statement for the year ended 30 September 2006 Revenue (105,000 + 62,000 18,000 intra group) Cost of sales (see working) Gross profit Distribution costs (4,000 + 2,000) Administrative expenses (7,500 + 7,000) Finance costs (1,200 + 900) Impairment losses: Goodwill Investment in associate (22,000 21,500) Share of loss from associate (20,000 x 40% x 3/12) Profit before tax Income tax expense (8,700 + 2,600) Profit for the period Attributable to: Equity holders of the parent Minority Interest ((13,000 1,000 depreciation adjustment) x 20%) $000 149,000 (89,000) 60,000 (6,000) (14,500) (2,100) (1,600) (500) (2,000) 33,300 (11,300) 22,000 19,600 2,400 22,000 $000 68,000 36,500 (18,000) 1,000 1,500 89,000

Note: the dividend from Sunlee is eliminated on consolidation. Working Cost of sales Hosterling Sunlee Intra group purchases Additional depreciation of plant (5,000/5 years) Unrealised profit in inventories (7,500 x 25%/125%)

13

(a)

Tadeon Income statement Year to 30 September 2006 $000 Revenue Cost of sales (w (i)) Gross profit Operating expenses (40,000 + 1,200 (w (ii))) Investment income Finance costs finance lease (w (ii)) loan (w (iii)) Profit before tax Income tax expense (w (iv)) Profit for the period

$000 277,800 (144,000) 133,800 (41,200) 2,000 (4,250) 90,350 (36,800) 53,550

(1,500) (2,750)

(b)

Tadeon Balance Sheet as at 30 September 2006 Non-current assets Property, plant and equipment (w (v)) Investments at amortised cost Current assets Inventories Trade receivables Total assets Equity and liabilities Capital and reserves: Equity shares of 20 cents each fully paid (w (vi)) Reserves Share premium (w (vi)) Revaluation reserve (w (v)) Retained earnings (w (vii)) $000 $000 299,000 42,000 341,000

33,300 53,500

86,800 427,800

200,000 28,000 16,000 42,150

86,150 286,150

Non-current liabilities 2% Loan note (w (iii)) Deferred tax (w (iv)) Finance lease obligation (w (ii)) Current liabilities Trade payables Accrued lease finance costs (w (ii)) Finance lease obligation (w (ii)) Bank overdraft Income tax payable (w (iv)) Total equity and liabilities Workings (note figures in brackets are in $000) (i) Cost of sales: Per trial balance Depreciation (12,000 + 5,000 + 9,000 w (v))

51,750 14,800 10,500 18,700 1,500 4,500 1,900 38,000

77,050

64,600 427,800 $000 118,000 26,000 144,000

(ii)

Vehicle rentals/finance lease: The total amount of vehicle rentals is $62 million of which $12 million are operating lease rentals and $5 million is identified as finance lease rentals. The operating rentals have been included in operating expenses. Finance lease Fair value of vehicles First rental payment 1 October 2005 Capital outstanding to 30 September 2006 Accrued interest 10% (current liability) Total outstanding 30 September 2006 $000 20,000 (5,000) 15,000 1,500 16,500

14

In the year to 30 September 2007 (i.e. on 1 October 2006) the second rental payment of $6 million will be made, of this $15 million is for the accrued interest for the previous year, thus $45 million will be a capital repayment. The remaining $105 million (16,500 (4,500 + 1,500)) will be shown as a non-current liability. (iii) Although the loan has a nominal (coupon) rate of only 2%, amortisation of the large premium on redemption, gives an effective interest rate of 55% (from question). This means the finance charge to the income statement will be a total of $275 million (50,000 x 55%). As the actual interest paid is $1 million an accrual of $175 million is required. This amount is added to the carrying amount of the loan in the balance sheet. (iv) Income tax and deferred tax The income statement charge is made up as follows: Current years provision Deferred tax (see below) $'000 38,000 (1,200) 36,800

There are $74 million of taxable temporary differences at 30 September 2006. With an income tax rate of 20%, this would require a deferred tax liability of $148 million (74,000 x 20%). $4 million ($20m x 20%) is transferred to deferred tax in respect of the revaluation of the leasehold property (and debited to the revaluation reserve), thus the effect of deferred tax on the income statement is a credit of $12 million (14,800 4,000 12,000 b/f). (v) Non-current assets/depreciation: Non-leased plant This has a carrying amount of $96 million (181,000 85,000) prior to depreciation of $12 million at 121/2% reducing balance to give a carrying amount of $84 million at 30 September 2006. The leased vehicles will be included in non-current assets at their fair value of $20 million and depreciated by $5 million (four years straight-line) for the year ended 30 September 2006 giving a carrying amount of $15 million at that date. The 25 year leasehold property is being depreciated at $9 million per annum (225,000/25 years). Prior to its revaluation on 30 September 2006 there would be a further years depreciation charge of $9 million giving a carrying amount of $180 million (225,000 (36,000 + 9,000)) prior to its revaluation to $200 million. Thus $20 million would be transferred to a revaluation reserve. The question says the revaluation gives rise to $20 million of the deductible temporary differences, at a tax rate of 20%, this would give a credit to deferred tax of $4 million which is debited to the revaluation reserve to give a net balance of $16 million. Summarising: cost/valuation $,000 200,000 181,000 20,000 401,000 accumulated depreciation $,000 nil 97,000 5,000 102,000 carrying amount $,000 200,000 84,000 15,000 299,000

25 year leasehold property Non-leased plant Leased vehicles

(vi) Suspense account The called up share capital of $150 million in the trial balance represents 750 million shares (150m/02) which have a market value at 1 October 2005 of $600 million (750m x 80 cents). A yield of 5% on this amount would require a $30 million dividend to be paid. A fully subscribed rights issue of one new share for every three shares held at a price of 32c each would lead to an issue of 250 million (150m/02 x 1/3). This would yield a gross amount of $80 million, and after issue costs of $2 million, would give a net receipt of $78 million. This should be accounted for as $50 million (250m x 20 cents) to equity share capital and the balance of $28 million to share premium. The receipt from the share issue of $78 million less the payment of dividends of $30 million reconciles the suspense account balance of $48 million. (vii) Retained earnings At 1 October 2005 Year to 30 September 2006 less dividends paid (w (vi)) $,000 18,600 53,550 (30,000) 42,150

(a)

Most forms of off balance sheet financing have the effect of what is, in substance, debt finance either not appearing on the balance sheet at all or being netted off against related assets such that it is not classified as debt. Common examples would be structuring a lease such that it fell to be treated as an operating lease when it has the characteristics of a finance lease, complex financial instruments classified as equity when they may have, at least in part, the substance of debt and controlled entities having large borrowings (used to benefit the group as a whole), that are not consolidated because the financial structure avoids the entities meeting the definition of a subsidiary.

15

The main problem of off balance sheet finance is that it results in financial statements that do not faithfully represent the transactions and events that have taken place. Faithful representation is an important qualitative characteristic of useful information (as described in the Framework for the preparation and presentation of financial statements). Financial statements that do not faithfully represent that which they purport to lack reliability. A lack of reliability may mean that any decisions made on the basis of the information contained in financial statements are likely to be incorrect or, at best, suboptimal. The level of debt on a balance sheet is a direct contributor to the calculation of an entitys balance sheet gearing, which is considered as one of the most important financial ratios. It should be understood that, to a point, the use of debt financing is perfectly acceptable. Where balance sheet gearing is considered low, borrowing is relatively inexpensive, often tax efficient and can lead to higher returns to shareholders. However, when the level of borrowings becomes high, it increases risk in many ways. Off balance sheet financing may lead to a breach of loan covenants (a serious situation) if such debt were to be recognised on the balance sheet in accordance with its substance. High gearing is a particular issue to equity investors. Equity (ordinary shares) is sometimes described as residual return capital. This description identifies the dangers (to equity holders) when an entity has high gearing. The dividend that the equity shareholders might expect is often based on the level of reported profits. The finance cost of debt acts as a reduction of the profits available for dividends. As the level of debt increases, higher interest rates are also usually payable to reflect the additional risk borne by the lender, thus the higher the debt the greater the finance charges and the lower the profit. Many off balance sheet finance schemes also disguise or hide the true finance cost which makes it difficult for equity investors to assess the amount of profits that will be needed to finance the debt and consequently how much profit will be available to equity investors. Furthermore, if the market believes or suspects an entity is involved in creative accounting (and off balance sheet finance is a common example of this) it may adversely affect the entitys share price. An entitys level of gearing will also influence any decision to provide further debt finance (loans) to the entity. Lenders will consider the nature and value of the assets that an entity owns which may be provided as security for the borrowings. The presence of existing debt will generally increase the risk of default of interest and capital repayments (on further borrowings) and existing lenders may have a prior charge on assets available as security. In simple terms if an entity has high borrowings, additional borrowing is more risky and consequently more expensive. A prospective lender to an entity that already has high borrowings, but which do not appear on the balance sheet is likely to make the wrong decision. If the correct level of borrowings were apparent, either the lender would not make the loan at all (too high a lending risk) or, if it did make the loan, it would be on substantially different terms (e.g. charge a higher interest rate) so as to reflect the real risk of the loan. Some forms of off balance sheet financing may specifically mislead suppliers that offer credit. It is a natural precaution that a prospective supplier will consider the balance sheet strength and liquidity ratios of the prospective customer. The existence of consignment inventories may be particularly relevant to trade suppliers. Sometimes consignment inventories and their related current liabilities are not recorded on the balance sheet as the wording of the purchase agreement may be such that the legal ownership of the goods remains with the supplier until specified events occur (often the onward sale of the goods). This means that other suppliers cannot accurately assess an entitys true level of trade payables and consequently the average payment period to suppliers, both of which are important determinants in deciding whether to grant credit. (b) (i) Debt factoring is a common method of entities releasing the liquidity of their trade receivables. The accounting issue that needs to be decided is whether the trade receivables have been sold, or whether the income from the finance house for their sale should be treated as a short term loan. The main substance issue with this type of transaction is to identify which party bears the risks (i.e. of slow and non-payment by the customer) relating to the asset. If the risk lies with the finance house (Omar), the trade receivables should be removed from the balance sheet (derecognised in accordance with IAS 39). In this case it is clear that Angelino still bears the risk relating to slow and non-payment. The residual payment by Omar depends on how quickly the receivables are collected; the longer it takes, the less the residual payment (this imputes a finance cost). Any balance uncollected by Omar after six months will be refunded by Angelino which reflects the non-payment risk. Thus the correct accounting treatment for this transaction is that the cash received from Omar (80% of the selected receivables) should be treated as a current liability (a short term loan) and the difference between the gross trade receivables and the amount ultimately received from Omar (plus any amounts directly from the credit customers themselves) should be charged to the income statement. The classification of the charge is likely to be a mixture of administrative expenses (for Omar collecting receivables), finance expenses (reflecting the time taken to collect the receivables) and the impairment of trade receivables (bad debts). (ii) This is an example of a sale and leaseback of a property. Such transactions are part of normal commercial activity, often being used as a way to improve cash flow and liquidity. However, if an asset is sold at an amount that is different to its fair value there is likely to be an underlying reason for this. In this case it appears (based on the opinion of the auditor) that Finaid has paid Angelino $2 million more than the building is worth. No (unconnected) company would do this knowingly without there being some form of compensating transaction. This sale is linked to the five year rental agreement. The question indicates the rent too is not at a fair value, being $500,000 per annum ($1,300,000 $800,000) above what a commercial rent for a similar building would be. It now becomes clear that the excess purchase consideration of $2 million is an in substance loan (rather than sales proceeds the legal form) which is being repaid through the excess ($500,000 per annum) of the rentals. Although this is a sale and leaseback transaction, as the building is freehold and has an estimated remaining life (20 years) that is much longer than the five year leaseback period, the lease is not a finance lease and the building should be treated as sold and thus derecognised.

16

The correct treatment for this item is that the sale of the building should be recorded at its fair value of $10 million, thus the profit on disposal would be $25 million ($10 million $75 million). The excess of $2 million ($12 million $10 million) should be treated as a loan (non-current liability). The rental payment of $13 million should be split into three elements; $800,000 building rental cost, $200,000 finance cost (10% of $2 million) and the remaining $300,000 is a capital repayment of the loan. (iii) The treatment of consignment inventory depends on the substance of the arrangements between the manufacturer and the dealer (Angelino). The main issue is to determine if and at what point in time the cars are sold. The substance is determined by analysing which parties bear the risks (e.g. slow moving/obsolete inventories, finance costs) and receive the benefits (e.g. use of inventories, potential for higher sales, protection from price increases) associated with the transaction. Supplies from Monza Angelino has, and has actually exercised, the right to return the cars without penalty (or been required by Monza to transfer them to another dealer), which would indicate that it has not bought the cars. There are no finance costs incurred by Angelino, however Angelino would suffer from any price increases that occurred during the three month holding/display period. These factors seem to indicate that the substance of this arrangement is the same as its legal form i.e. Monza should include the cars in its balance sheet as inventory and therefore Angelino will not record a purchase transaction until it becomes obliged to pay for the cars (three months after delivery or until sold to customers if sooner). Supplies from Capri Although this arrangement seems similar to the above, there are several important differences. Angelino is bearing the finance costs of 1% per month (calling it a display charge is a distraction). The option to return the cars should be ignored because it is not likely to be exercised due to commercial penalties (payment of transport costs and loss of deposit). Finally the purchase price is fixed at the date of delivery rather than at the end of six months. These factors strongly indicate that Angelino bears the risks and rewards associated with ownership and should recognise the inventory and the associated liability in its financial statements at the date of delivery.

17

(a)

Cash Flow Statement of Minster for the Year ended 30 September 2006: Cash flows from operating activities Profit before tax Adjustments for: Depreciation of property, plant and equipment Amortisation of software (180 135) Investment income Finance costs Working capital adjustments Decrease in trade receivables (380 270) Increase in amounts due from construction contracts (80 55) Decrease in inventories (510 480) Decrease in trade payables (555 350) Cash generated from operations Interest paid (40 12 re unwinding of environmental provision) Income taxes paid (w (ii)) Net cash from operating activities Cash flows from investing activities Purchase of property, plant and equipment (w (i)) software investments (150 (15 + 125)) Investment income received (20 15 gain on investments) Net cash used in investing activities Cash flows from financing activities Proceeds from issue of equity shares (w (iii)) Proceeds from issue of 9% loan note Dividends paid (500 x 4 x 5 cents) Net cash from financing activities Net decrease in cash and cash equivalents Cash and cash equivalents at beginning of period (40 35) Cash and cash equivalents at end of period 265 120 (100) 285 (20) (5) (25) (410) (180) (10) 5 (595) $000 $000 142

255 45

300 (20) 40 462

110 (25) 30 (205)

(90) 372 (28) (54) 290

Note: interest paid may be presented under financing activities and dividends paid may be presented under operating activities. Workings (in $000) (i) Property, plant and equipment: carrying amount b/f non-cash environmental provision revaluation depreciation for period carrying amount c/f difference is cash acquisitions (ii) Taxation: tax provision b/f deferred tax b/f income statement charge tax provision c/f deferred tax c/f difference is cash paid (iii) Equity shares balance b/f bonus issue (1 for 4) balance c/f difference is cash issue

940 150 35 (255) (1,280) (410) (50) (25) (57) 60 18 (54) (300) (75) 500 125

18

Share premium balance b/f bonus issue (1 for 4) balance c/f difference is cash issue Therefore the total proceeds of cash issue of shares are $265,000 (125 + 140). (b) Report on the financial position of Minster for the year ended 30 September 2006 To: From: Date:

(85) 75 150 140

Minster shows healthy operating cash inflows of $372,000 (prior to finance costs and taxation). This is considered by many commentators as a very important figure as it is often used as the basis for estimating the companys future maintainable cash flows. Subject to (inevitable) annual expected variations and allowing for any changes in the companys structure this figure is more likely to be repeated in the future than most other figures in the cash flow statements which are often one-off cash flows such as raising loans or purchasing non-current assets. The operating cash inflow compares well with the underlying profit before tax $142,000. This is mainly due to depreciation charges of $300,000 being added back to the profit as they are a non-cash expense. The cash inflow generated from operations of $372,000 together with the reduction in net working capital of $90,000 is more than sufficient to cover the companys taxation payments of $54,000, interest payments of $28,000 and the dividend of $100,000 and leaves an amount to contribute to the funding of the increase in non-current assets. It is important that these short term costs are funded from operating cash flows; it would be of serious concern if, for example, interest or income tax payments were having to be funded by loan capital or the sale of non-current assets. There are a number of points of concern. The dividend of $100,000 gives a dividend cover of less than one (85/100 = 085) which means the company has distributed previous years profits. This is not a tenable situation in the long-term. The size of the dividend has also contributed to the lower cash balances (see below). There is less investment in both inventory levels and trade receivables. This may be the result of more efficient inventory control and better collection of receivables, but it may also indicate that trading volumes may be falling. Also of note is a large reduction in trade payable balances of $205,000. This too may be indicative of lower trading (i.e. less inventory purchased on credit) or pressure from suppliers to pay earlier. Without more detailed information it is difficult to come to a conclusion in this matter. Investing activities: The cash flow statement shows considerable investment in non-current assets, in particular $410,000 in property, plant and equipment. These acquisitions represent an increase of 44% of the carrying amount of the property, plant and equipment as at the beginning of the year. As there are no disposals, the increase in investment must represent an increase in capacity rather than the replacement of old assets. Assuming that this investment has been made wisely, this should bode well for the future (most analysts would prefer to see increased investment rather than contraction in operating assets). An unusual feature of the required treatment of environmental provisions is that the investment in non-current assets as portrayed by the cash flow statement appears less than if balance sheet figures are used. The balance sheet at 30 September 2006 includes $150,000 of non-current assets (the discounted cost of the environmental provision), which does not appear in the cash flow figures as it is not a cash cost. A further consequence is that the unwinding of the discounting of the provision causes a financing expense in the income statement which is not matched in the cash flow statement as the unwinding is not a cash flow. Many commentators have criticised the required treatment of environmental provisions because they cause financing expenses which are not (immediate) cash costs and no loans have been taken out. Viewed in this light, it may be that the information in the cash flow statement is more useful than that in the income statement and balance sheet. Financing activities: The increase in investing activities (before investment income) of $600,000 has been largely funded by an issue of shares at $265,000 and raising a 9% $120,000 loan note. This indicates that the companys shareholders appear reasonably pleased with the company's past performance (or they would not be very willing to purchase further shares). The interest rate of the loan at 9% seems quite high, and virtually equal to the companys overall return on capital employed of 91% (162/(1,660 + 120)). Provided current profit levels are maintained, it should not reduce overall returns to shareholders. Cash position: The overall effect of the years cash flows has worsened the companys cash position by an increased net cash liability of $20,000. Although the companys short term borrowings have reduced by $15,000, the cash at bank of $35,000 at the beginning of the year has now gone. In comparison to the cash generation ability of the company and considering its large investment in non-current assets, this $20,000 is a relatively small amount and should be relieved by operating cash inflows in the near future.

19

Summary The above analysis shows that Minster has invested substantially in new non-current assets suggesting expansion. To finance this, the company appears to have no difficulty in attracting further long-term funding. At the same time there are indications of reduced inventories, trade receivables and payables which may suggest the opposite i.e. contraction. It may be that the new investment is a change in the nature of the companys activities (e.g. mining) which has different working capital characteristics. The company has good operating cash flow generation and the slight deterioration in short term net cash balance should only be temporary. Yours ..

(a)

(i)

IFRS 5 Non-current assets held for sale and discontinued operations defines non-current assets held for sale as those assets (or a group of assets) whose carrying amounts will be recovered principally through a sale transaction rather than through continuing use. A discontinued operation is a component of an entity that has either been disposed of, or is classified as held for sale and: (i) represents a separate major line of business or geographical area of operations (ii) is part of a single co-ordinated plan to dispose of such, or (iii) is a subsidiary acquired exclusively for sale. IFRS 5 says that a component of an entity must have operations and cash flows that can be clearly distinguished from the rest of the entity and will in all probability have been a cash-generating unit (or group of such units) whilst held for use. This definition also means that a discontinued operation will also fall to be treated as a disposal group as defined in IFRS 5. A disposal group is a group of assets (possibly with associated liabilities) that it is intended will be disposed of in a single transaction by sale or otherwise (closure or abandonment). Assets held for disposal (but not those being abandoned) must be presented separately (at the lower of cost or fair value less costs to sell) from other assets and included as current assets (rather than as non-current assets) and any associated liabilities must be separately presented under liabilities. The results of a discontinued operation should be disclosed separately as a single figure (as a minimum) on the face of the income statement with more detailed figures disclosed either also on the face of the income statement or in the notes. The intention of this requirement is to improve the usefulness of the financial statements by improving the predictive value of the (historical) income statement. Clearly the results from discontinued operations should have little impact on future operating results. Thus users can focus on the continuing activities in any assessment of future income and profit.

(ii)

The timing of the board meeting and consequent actions and notifications is within the accounting period ended 31 October 2006. The notification of staff, suppliers and the press seems to indicate that the sale will be highly probable and the directors are committed to a plan to sell the assets and are actively locating a buyer. From the financial and other information given in the question it appears that the travel agencies operations and cash flows can be clearly distinguished from its other operations. The assets of the travel agencies appear to meet the definition of non-current assets held for sale; however the main difficulty is whether their sale and closure also represent a discontinued operation. The main issue is with the wording of a separate major line of business in part (i) of the above definition of a discontinued operation. The company is still operating in the holiday business, but only through Internet selling. The selling of holidays through the Internet compared with through high-street travel agencies requires very different assets, staff knowledge and training and has a different cost structure. It could therefore be argued that although the company is still selling holidays the travel agencies do represent a separate line of business. If this is the case, it seems the announced closure of the travel agencies appears to meet the definition of a discontinued operation.

20

(iii) Partway income statement year ended: 31 October 2006 $000 Continuing operations Revenue Cost of sales Gross profit Operating expenses Profit/(loss) from continuing operations Discontinued operations Profit/(loss) from discontinued operations Profit for the period Analysis of discontinued operations Revenue Cost of sales Gross profit/(loss) Operating expenses Profit/(loss) from discontinued operations Note: other presentations may be acceptable. (b) (i) Comparability is one of the four principal qualitative characteristics of useful financial information. It is a vital attribute when assessing the performance of an entity over time (trend analysis) and to some extent with other similar entities. For information to be comparable it should be based on the consistent treatment of transactions and events. In effect a change in an accounting policy breaks the principle of consistency and should generally be avoided. That said there are circumstances where it becomes necessary to change an accounting policy. These are mainly where it is required by a new or revised accounting standard, interpretation or applicable legislation or where the change would result in financial statements giving a more reliable and relevant representation of the entitys transactions and events. It is important to note that the application of a different accounting policy to transactions or events that are substantially different to existing transactions or events or to transactions or events that an entity had not previously experienced does NOT represent a change in an accounting policy. It is also necessary to distinguish between a change in an accounting policy and a change in an estimation technique. In an attempt to limit the problem of reduced comparability caused by a change in an accounting policy, the general principle is that the financial statements should be prepared as if the new accounting policy had always been in place. This is known as retrospective application. The main effect of this is that comparative financial statements should be restated by applying the new policy to them and adjusting the opening balance of each component of equity affected in the earliest prior period presented. IAS 8 Accounting policies, changes in accounting estimates and errors says that a change in accounting policy required by a specific Standard or Interpretation should be dealt with under the transitional provisions (if any) of that Standard or Interpretation (normally these apply the general rule of retrospective application). There are some limited exemptions (mainly on the grounds of impracticality) to the general principle of retrospective application in IAS 8. (ii) This issue is one of the timing of when revenue should be recognised in the income statement. This can be a complex issue which involves identifying the transfer of significant risks, reliable measurement, the probability of receiving economic benefits, relevant accounting standards and legislation and generally accepted practice. Applying the general guidance in IAS 18 Revenue, the previous policy, applied before cancellation insurance was made a condition of booking, seemed appropriate. At the time the holiday is taken it can no longer be cancelled, all monies would have been received and the flights and accommodation have been provided. There may be some compensation costs involved if there are problems with the holiday, but this is akin to product warranties on normal sales of goods which may be immaterial or provided for based on previous experience of such costs. The appendix to IAS 18 specifically refers to payments in advance of the delivery of goods and says that revenue should be recognised when the goods are delivered. Interpreting this for Partways transaction would seem to confirm the appropriateness of its previous policy. The directors of Partway wish to change the timing of recognition of sales because of the change in circumstances relating to the compulsory cancellation insurance. The directors are apparently arguing that the new transactions and events are substantially different to previous transactions therefore the old policy should not apply. Even if this does justify revising the timing of the recognition of revenue, it is not a change of accounting policy because of the reasons outlined in (i) above. An issue to consider is whether compulsory cancellation insurance represents a substantial change to the risks that Partway experiences. An analysis of past experience of losses caused by uninsured cancellations may help to assess this, but even if the past losses were material (and in future they wont be), it is unlikely that this would override the general guidance in the appendix to IAS 18 relating to payments made in advance of delivery. It seems the main motivation for the proposed change is to improve the profit for the year ended 31 October 2006 so that it compares more favourably with that of the previous period. 25,000 (19,500) 5,500 (1,100) 4,400 (4,000) 400 14,000 (16,500) (2,500) (1,500) (4,000) 31 October 2005 $000 22,000 (17,000) 5,000 (500) 4,500 1,500 6,000 18,000 (15,000) 3,000 (1,500) 1,500

21

To summarise, it is unlikely that the imposition of compulsory cancellation insurance justifies recognising sales at the date of booking when a deposit is received, and, even if it did, it would not be a change in accounting policy. This means that comparatives would not be restated (which is something that would actually suit the suspected objectives of the directors).

22

Part 2 Examination Paper 2.5(INT) Financial Reporting (International Stream)

December 2006 Marking Scheme

This marking scheme is given as a guide in the context of the suggested answers. Scope is given to markers to award marks for alternative approaches to a question, including relevant comment, and where well-reasoned conclusions are provided. This is particularly the case for written answers where there may be more than one acceptable solution. Marks 1 (a) Goodwill of Sunlee: consideration equity shares pre acquisition reserves fair value adjustments maximum (b) Carrying amount and impairment of Amber: cash paid 6% loan note post acquisition loss maximum (c) Income statement: revenue cost of sales distribution costs and administrative expenses finance costs impairment of goodwill impairment of associate share of associates loss income tax minority interests eliminate dividend from Sunlee maximum Maximum for question 1 1 1 2 5

1 1 2 4

2 4 1 1 1 1 2 1 2 1 16 25

(a)

Income statement revenue cost of sales operating expenses investment income finance costs income tax expense available maximum

1 2 1 1 2 3 10 8

(b)

Balance sheet property, plant and equipment investment inventory and trade receivables share capital and premium revaluation reserve retained earnings (including 1 mark for the dividend) loan note deferred tax lease obligation (1 for current, 1 for long-term) trade payables and overdraft accrued lease finance costs income tax payable available maximum Maximum for question

3 1 1 3 2 2 2 1 2 1 1 1 20 17 25

23

(a) (b)

1 mark per relevant point to a (i) (ii) 1 mark per relevant point to a

maximum maximum

Marks 9 5 1 1 1 1 1 1 6 2 2 2 6 5 25

sale price not at fair value raises substance issues leaseback is not a finance lease treat building as sold (derecognise) at a profit of $25 million rental treated as: $800,000 rental cost $200,000 finance cost $300,000 loan repayment maximum

(iii) general discussion of risks and rewards re consignment goods Issues and accounting treatment relating to supplies from Monza Issues and accounting treatment relating to supplies from Capri available maximum Maximum for question

(a)

cash flow from operating activity profit before tax adjusted for investment income and finance cost depreciation/amortisation working capital items finance costs income taxes paid investing activities (including 1 for investment income) financing issue of ordinary shares issue of 9% loan dividend paid cash and cash equivalents b/f and c/f available maximum 1 mark per relevant point Maximum for question

1 2 2 2 2 4 1 1 1 1 17 15 10 25

(b)

(a)

(i)

definitions usefulness of information maximum discussion of whether a discontinued operation conclusion maximum for for for for for for revenue from continuing operations (2005 and 2006) revenue from discontinued operations (2005 and 2006) cost of sales from continuing operations (2005 and 2006) cost of sales from discontinued operations (2005 and 2006) profit from continuing operations (2005 and 2006) profit from discontinued operations (2005 and 2006) maximum maximum maximum Maximum for question

2 2 4 3 1 4 1 1 1 1 1 1 6 5 6 25

(ii)

(iii) figures figures figures figures figures figures

(b)

(i) (ii)

1 mark per relevant point 1 mark per relevant point

24

Financial Reporting
(International Stream)
PART 2 THURSDAY 7 JUNE 2007

QUESTION PAPER Time allowed 3 hours This paper is divided into two sections Section A This ONE question is compulsory and MUST be answered THREE questions ONLY to be answered

Section B

Do not open this paper until instructed by the supervisor This question paper must not be removed from the examination hall

The Association of Chartered Certified Accountants

Paper 2.5(INT)

Section A This ONE question is compulsory and MUST be attempted 1 Parentis, a public listed company, acquired 600 million equity shares in Offspring on 1 April 2006. The purchase consideration was made up of: a share exchange of one share in Parentis for two shares in Offspring the issue of $100 10% loan note for every 500 shares acquired; and a deferred cash payment of 11 cents per share acquired payable on 1 April 2007. Parentis has only recorded the issue of the loan notes. The value of each Parentis share at the date of acquisition was 75 cents and Parentis has a cost of capital of 10% per annum. The balance sheets of the two companies at 31 March 2007 are shown below: Parentis $ million $ million Assets Property, plant and equipment (note (i)) Investments Intellectual property (note (ii)) Current assets Inventory (note (iii)) Trade receivables (note (iii)) Bank Total assets Equity and liabilities Equity shares of 25 cents each Retained earnings 1 April 2006 year ended 31 March 2007 Non-current liabilities 10% loan notes Current liabilities Trade payables (note (iii)) Current tax payable Overdraft Total equity and liabilities The following information is relevant: (i) At the date of acquisition the fair values of Offsprings net assets were approximately equal to their carrying amounts with the exception of its properties. These properties had a fair value of $40 million in excess of their carrying amounts which would create additional depreciation of $2 million in the post acquisition period to 31 March 2007. The fair values have not been reflected in Offsprings balance sheet. 130 45 25 640 120 nil 760 76 84 nil 22 44 4 Offspring $ million $ million 340 nil 30 370

160 920 300

70 440 200

210 90

300 600 120

120 20

140 340 20

200 920

57 23 nil

80 440

(ii) The intellectual property is a system of encryption designed for internet use. Offspring has been advised that government legislation (passed since acquisition) has now made this type of encryption illegal. Offspring will receive $10 million in compensation from the government. (iii) Offspring sold Parentis goods for $15 million in the post acquisition period. $5 million of these goods are included in the inventory of Parentis at 31 March 2007. The profit made by Offspring on these sales was $6 million. Offsprings trade payable account (in the records of Parentis) of $7 million does not agree with Parentiss trade receivable account (in the records of Offspring) due to cash in transit of $4 million paid by Parentis. (iv) Due to the impact of the above legislation, Parentis has concluded that the consolidated goodwill has been impaired by $27 million.

Required: Prepare the consolidated balance sheet of Parentis as at 31 March 2007. (25 marks)

[P.T.O.

Section B THREE questions ONLY to be attempted 2 The summarised draft financial statements of Wellmay are shown below. Income statement year ended 31 March 2007: Revenue (note (i)) Cost of sales (note (ii)) Gross profit Operating expenses Investment property rental income Finance costs Profit before tax Income tax Profit for the period Balance sheet as at 31 March 2007: $000 Assets Non-current assets Property, plant and equipment (note (iii)) Investment property (note (iii)) Current assets Total assets Equity and liabilities Equity Equity shares of 50 cents each (note (vii)) Reserves: Revaluation reserve Retained earnings (note (iv)) Non-current liabilities 8% Convertible loan note (2010) (note (v)) Deferred tax (note (vi)) Current liabilities Total equity and liabilities The following information is relevant to the draft financial statements: (i) Revenue includes $500,000 for the sale on 1 April 2006 of maturing goods to Westwood. The goods had a cost of $200,000 at the date of sale. Wellmay can repurchase the goods on 31 March 2008 for $605,000 (based on achieving a lenders return of 10% per annum) at which time the goods are estimated to have a value of $750,000. $000 $000 4,200 (2,700) 1,500 (470) 20 (55) 995 (360) 635

4,200 400 4,600 1,400 6,000

1,200 350 2,850

3,200 4,400

600 180

780 820 6,000

(ii) Past experience shows that in the post balance sheet period the company often receives unrecorded invoices for materials relating to the previous year. As a result of this an accrued charge of $75,000 for contingent costs has been included in cost of sales and as a current liability.

(iii) Non-current assets: Wellmay owns two properties. One is a factory (with office accommodation) used by Wellmay as a production facility and the other is an investment property that is leased to a third party under an operating lease. Wellmay revalues all its properties to current value at the end of each year and uses the fair value model in IAS 40 Investment property. Relevant details of the fair values of the properties are: Factory $000 1,200 1,350 Investment property $000 400 375

Valuation 31 March 2006 Valuation 31 March 2007

The valuations at 31 March 2007 have not yet been incorporated into the financial statements. Factory depreciation for the year ended 31 March 2007 of $40,000 was charged to cost of sales. As the factory includes some office accommodation, 20% of this depreciation should have been charged to operating expenses. (iv) The balance of retained earnings is made up of: balance b/f 1 April 2006 profit for the period dividends paid during year ended 31 March 2007 $000 2,615 635 (400) 2,850

(v) 8% Convertible loan note (2010) On 1 April 2006 an 8% convertible loan note with a nominal value of $600,000 was issued at par. It is redeemable on 31 March 2010 at par or it may be converted into equity shares of Wellmay on the basis of 100 new shares for each $200 of loan note. An equivalent loan note without the conversion option would have carried an interest rate of 10%. Interest of $48,000 has been paid on the loan and charged as a finance cost. The present value of $1 receivable at the end of each year, based on discount rates of 8% and 10% are: End of year 1 2 3 4 8% 093 086 079 073 10% 091 083 075 068

(vi) The carrying amounts of Wellmays net assets at 31 March 2007 are $600,000 higher than their tax base. The rate of taxation is 35%. The income tax charge of $360,000 does not include the adjustment required to the deferred tax provision which should be charged in full to the income statement. (vii) Bonus/scrip issue: On 15 March 2007, Wellmay made a bonus issue from retained earnings of one share for every four held. The issue has not been recorded in the draft financial statements. Required: Redraft the financial statements of Wellmay, including a statement of changes in equity, for the year ended 31 March 2007 reflecting the adjustments required by notes (i) to (vii) above. Note: Calculations should be made to the nearest $000. (25 marks)

[P.T.O.

(a) A trainee accountant has been assisting in the preparation of the financial statements of Toogood for the year ended 31 March 2007. He has observed that the corresponding figures (i.e. for the year ended 31 March 2006) in the financial statements for the year ended 31 March 2007 do not agree in several instances with the equivalent figures that were published in the companys financial statements for year ended 31 March 2006. In particular: consolidated goodwill (gross figure before impairment) appears to have been recalculated, several other non-current assets have been revised, the brought forward retained earnings have been restated and; several income statement line items are also different. The trainee accountant has also noted that even when the revised earnings figure for the year ended 31 March 2006 is divided by the weighted average number of shares in issue during that year, it still does not agree with the comparative earnings per share figure (i.e. for the year ended 31 March 2006) reported in the financial statements for the year ended 31 March 2007. Required: Explain three circumstances where accounting standards require previously reported financial statement figures to be amended when they are reproduced as corresponding amounts. Note: It may help to consider, among other things, the items mentioned by the trainee accountant. (12 marks) (b) The trainee accountant has been reading some literature written by a qualified surveyor on the values of leasehold property located in the area where Toogood owns leasehold property. The main thrust is that historically, annual increases in property prices more than compensate for the fall in the carrying amount caused by annual amortisation until a leasehold property has less than 10 years of remaining life. Therefore the trainee accountant suggests that the company should adopt a policy of carrying its leasehold properties at cost until their remaining lives are 10 years and then amortising them on a straight-line basis over 10 years. This would improve the companys reported profit and cash flows as well as showing a faithful representation of the value of the leasehold properties. Required: Comment on the validity and acceptability of the trainee accountants suggestion. (7 marks)

(c) The trainee accountant notes that Toogood acquired the Trilogy group during the year ended 31 March 2007. The Trilogy group consists of Trilogy itself and two wholly owned subsidiaries. Toogood has only consolidated Trilogy and one subsidiary with the other subsidiary being shown as a current asset. The trainee accountant wonders if this is because the non-consolidated subsidiary is making losses. Required: Explain why the two subsidiaries may require the different treatments that Toogood has applied. (6 marks) (25 marks)

This is a blank page. Question 4 begins on page 8.

[P.T.O.

Greenwood is a public listed company. During the year ended 31 March 2007 the directors decided to cease operations of one of its activities and put the assets of the operation up for sale (the discontinued activity has no associated liabilities). The directors have been advised that the cessation qualifies as a discontinued operation and has been accounted for accordingly. Extracts from Greenwoods financial statements are set out below. Note: the income statement figures down to the profit for the period from continuing operations are those of the continuing operations only. Income statements for the year ended 31 March: Revenue Cost of sales Gross profit Operating expenses Finance costs Profit before taxation Income tax expense Profit for the period from continuing operations Profit/(Loss) from discontinued operations Profit for the period Analysis of discontinued operations: Revenue Cost of sales Gross profit/(loss) Operating expenses Profit/(loss) before tax Tax (expense)/relief Loss on measurement to fair value of disposal group Tax relief on disposal group Profit/(Loss) from discontinued operations 2007 $000 27,500 (19,500) 8,000 (2,900) 5,100 (600) 4,500 (1,000) 3,500 (1,500) 2,000 7,500 (8,500) (1,000) (400) (1,400) 300 (1,100) (500) 100 (1,500) 2006 $000 21,200 (15,000) 6,200 (2,450) 3,750 (250) 3,500 (800) 2,700 320 3,020 9,000 (8,000) 1,000 (550) 450 (130) 320 320

Balance Sheets as at 31 March $000 Non-current assets Current assets Inventory Trade receivables Bank Assets held for sale (at fair value) Total assets Equity and liabilities Equity shares of $1 each Retained earnings Non-current liabilities 5% loan notes Current liabilities Bank overdraft Trade payables Current tax payable Total equity and liabilities 1,150 2,400 950 1,500 2,000 nil 6,000

2007 $000 17,500 $000

2006 $000 17,600

9,500 27,000 10,000 4,500 14,500 8,000

1,350 2,300 50 nil

3,700 21,300 10,000 2,500 12,500 5,000

4,500 27,000

nil 2,800 1,000

3,800 21,300

Note: the carrying amount of the assets of the discontinued operation at 31 March 2006 was $63 million. Required: Analyse the financial performance and position of Greenwood for the two years ended 31 March 2007. Note: Your analysis should be supported by appropriate ratios (up to 10 marks available) and refer to the effects of the discontinued operation. (25 marks)

[P.T.O.

(a) The following is an extract of Errseas balances of property, plant and equipment and related government grants at 1 April 2006. cost $000 240 accumulated depreciation $000 180 carrying amount $000 60 30 10

Property, plant and equipment Non-current liabilities Government grants Current liabilities Government grants

Details including purchases and disposals of plant and related government grants during the year are: (i) Included in the above figures is an item of plant that was disposed of on 1 April 2006 for $12,000 which had cost $90,000 on 1 April 2003. The plant was being depreciated on a straight-line basis over four years assuming a residual value of $10,000. A government grant was received on its purchase and was being recognised in the income statement in equal amounts over four years. In accordance with the terms of the grant, Errsea repaid $3,000 of the grant on the disposal of the related plant.

(ii) An item of plant was acquired on 1 July 2006 with the following costs: Base cost Modifications specified by Errsea Transport and installation $ 192,000 12,000 6,000

The plant qualified for a government grant of 25% of the base cost of the plant, but it had not been received by 31 March 2007. The plant is to be depreciated on a straight-line basis over three years with a nil estimated residual value. (iii) All other plant is depreciated by 15% per annum on cost (iv) $11,000 of the $30,000 non-current liability for government grants at 1 April 2006 should be reclassified as a current liability as at 31 March 2007. (v) Depreciation is calculated on a time apportioned basis. Required: Prepare extracts of Errseas income statement and balance sheet in respect of the property, plant and equipment and government grants for the year ended 31 March 2007. Note: Disclosure notes are not required. (10 marks) (b) In the post balance sheet period, prior to authorising for issue the financial statements of Tentacle for the year ended 31 March 2007, the following material information has arisen. (i) The notification of the bankruptcy of a customer. The balance of the trade receivable due from the customer at 31 March 2007 was $23,000 and at the date of the notification it was $25,000. No payment is expected from the bankruptcy proceedings. (3 marks)

(ii) Sales of some items of product W32 were made at a price of $540 each in April and May 2007. Sales staff receive a commission of 15% of the sales price on this product. At 31 March 2007 Tentacle had 12,000 units of product W32 in inventory included at cost of $6 each. (4 marks) (iii) Tentacle is being sued by an employee who lost a limb in an accident while at work on 15 March 2007. The company is contesting the claim as the employee was not following the safety procedures that he had been instructed to use. Accordingly the financial statements include a note of a contingent liability of $500,000 for personal injury damages. In a recently decided case where a similar injury was sustained, a settlement figure of $750,000 was awarded by the court. Although the injury was similar, the circumstances of the accident in the decided case are different from those of Tentacles case. (4 marks) 10

(iv) Tentacle is involved in the construction of a residential apartment building. It is being accounted for using the percentage of completion basis in IAS 11 Construction contracts. The recognised profit at 31 March 2007 was $12 million based on costs to date of $3 million as a percentage of the total estimated costs of $6 million. Early in May 2007 Tentacle was informed that due to very recent industry shortages, building materials will cost $15 million more than the estimate of total cost used in the calculation of the percentage of completion. Tentacle cannot pass on any additional costs to the customer. (4 marks) Required: State and quantify how items (i) to (iv) above should be treated when finalising the financial statements of Tentacle for the year ended 31 March 2007. Note: The mark allocation is shown against each of the four items above. (25 marks)

End of Question Paper

11

Answers

Part 2 Examination Paper 2.5(INT) Financial Reporting (International Stream) 1 Consolidated Balance Sheet of Parentis as at 31 March 2007: $ million Assets Non-current assets Property, plant and equipment (640 + 340 + 40 2) Intangible Consolidated goodwill (135 (w (i)) 27 impairment) Current assets Inventory (76 + 22 2 URP) Trade receivables (84 + 44 11 intra-group) Receivable re intellectual property Bank Total assets Equity and liabilities Equity attributable to equity holders of the parent Equity shares 25c each (w (i)) Reserves: Share Premium (w (i)) Retained earnings (w (ii)) Minority interest (w (iii)) Total equity Non-current liabilities 10% loan notes (120 + 20) Current liabilities Trade payables (130 + 57 7 intra-group) Cash consideration due 1 April 2007 (60 + 6 interest) Overdraft (25 4 CIT) Taxation (45 + 23) Total equity and liabilities Workings (Note: all figures in $ million) (i) 180 66 21 68

June 2007 Answers

$ million

1,018 108 1,126 96 117 10 4

227 1,353

375 150 264

414 789 89 878 140

335 1,353

Goodwill: The acquisition of 600 million shares represents 75% of Offsprings 800 million shares ($200m/25c). The share exchange of 300 million (i.e. 1 for 2) at $075 each will result in an increase in equity share capital of $75 million (the nominal value) and create a share premium balance of $150 million (i.e. $050 premium on 300 million shares). Consideration: Equity shares (600/2 x $075) 10% loan notes (see below) Cash (600 x $011/11 i.e. discounted at 10%) Acquired: Equity shares (600m x 25c) Pre acquisition retained earnings (120 x 75%) Fair value adjustment to properties (40 x 75%) Goodwill 225 120 60 405 150 90 30

(270) 135

The issue of the 10% loan notes is calculated as 600 million/500 x $100 = $120 million.

15

(ii)

Retained earnings: Parentis Interest on deferred consideration (60 x 10%) Goodwill impairment (from question) Offspring URP in inventory (see below) Additional depreciation (from question) Write down intellectual property (30 10) Pre acquisition

300 (6) (27) 140 (2) (2) (20) (120) (4)

x 75%

(3) 264

The unrealised profit in inventory (URP) is $5m/$15m of the profit of $6 million made by Offspring. (iii) Minority interest Offspring net assets at 31 March 2007 Fair value adjustment URP in inventory Additional depreciation Write down intellectual property (30 10) 340 40 (2) (2) (20) 356

x 25%

89

Wellmay Income Statement year ended 31 March 2007: $000 Revenue (4,200 500 (w (i))) Cost of sales (w (ii)) Gross profit Operating expenses (470 + 8 depreciation) Investment property rental income fair value loss (400 375) Finance costs (w (iii)) Profit before tax Income tax (360 + 30 (w (v))) Profit for the period Statement of changes in equity year ended 31 March 2007 Equity shares Equity option $000 $000 Balances at 1 April 2006 1,200 Equity conversion option (w (iv)) 40 Bonus issue (1 for 4) 300 Revaluation of factory (w (vi)) Profit for the period Dividends Balances at 31 March 2007 1,500 40 $000 3,700 (2,417) 1,283 (478) (5) (113) 687 (390) 297 Total $000 4,165 40 190 297 (400) 4,292

20 (25)

Revaluation reserve $000 350

Retained earnings $000 2,615 (300)

190 540 297 (400) 2,212

16

Balance sheet as at 31 March 2007: $000 Non-current assets Property, plant and equipment (w (vi)) Investment property (w (vi)) Current assets (1,400 + 200 inventory (w (i))) Total assets Equity and liabilities (see statement of changes in equity above) Equity shares of 50 cents each Equity option (w (iv)) Reserves: Revaluation reserve Retained earnings Non-current liabilities Deferred tax (w (v)) 8% Convertible loan note ((560 + 8) (w (iv))) Current liabilities (820 75 (w (ii))) Loan from Westwood (500 + 50 accrued interest (w (i))) Total equity and liabilities Workings (note: all figures in $000) (i) The sale to Westwood is, in substance, a secured loan. The repurchase price is the cost of sale plus compound interest at 10% for two years. The correct accounting treatment is to reverse the sale with the goods going back into inventory and the proceeds treated as a loan with accrued interest of 10% ($50,000) for the current year. Cost of sales: From draft financial statements Sale of goods added back to inventory (see above) Reversal of contingency provision (see below) Depreciation transferred to operating costs (40 x 20%) 2,700 (200) (75) (8) 2,417 $000 4,390 375 4,765 1,600 6,365 1,500 40 1,540 540 2,212

2,752 4,292

210 568 745 550

778

1,295 6,365

(ii)

General or non-specific provisions do not meet the definition of a liability in IAS 37 Provisions, contingent liabilities and contingent assets and must therefore be reversed. (iii) Finance costs: From draft financial statements Additional accrued interest on convertible loan (w (iv)) Finance cost on in-substance loan (500 x 10%) 55 8 50 113

(iv) Convertible Loan: This is a compound financial instrument that contains an element of debt and an element of equity (the option to convert). IAS 32 Financial instruments: disclosure and presentation requires that the substance of such instruments should be applied to the reporting of them. The value of the debt element is calculated by discounting the future cash flows (at 10%). The residue of the issue proceeds is recorded as the value of the equity option: year year year year 1 2 3 4 interest interest interest interest, redemption premium and capital Cash flows 48 48 48 648 factor at 10% 091 083 075 068 present value $000 436 398 360 4406 5600 6000 400

total value of debt component proceeds of the issue equity component (residual amount)

For the year ended 31 March 2007, the interest cost for the convertible loan in the income statement should be increased from $48,000 to $56,000 (10% x 560) by accruing $8,000, which should be added to the carrying value of the debt.

17

(v)

Taxation: The required deferred tax balance is $210,000 (600 x 35%), the current balance is $180,000, and thus a further transfer of $30,000 (via the income statement) is required.

(vi) Properties: The fair value model in IAS 40 Investment property requires the loss of $25,000 on the fair value of investment properties to be reported in the income statement. This differs from revaluations of other properties. IAS 16 Property, plant and equipment requires surpluses and deficits to be recorded as movements in equity (a revaluation reserve). After depreciation of $40,000 for the year ended 31 March 2007, the factory (used by Wellmay) would have a carrying amount of $1,160,000 (1,200 40). The valuation of $1,350,000 at 31 March 2007 would give a further revaluation surplus of $190,000 (1,350 1,160) and a carrying amount of property, plant and equipment of $4,390,000 (4,200 + 190) at that date.

(a)

As a general principle the reported financial statements for (in this case) the year ended 31 March 2006 should become (without amendment) the corresponding amounts in the financial statements ended 31 March 2007. However, there are a number of circumstances where this general principle is modified: (i) As part of a business combination it is necessary to determine the goodwill as the residual amount in the process of allocating the consideration paid to the identifiable assets, liabilities and contingent liabilities acquired. IFRS 3 Business combinations requires the allocation process to be completed within a period of 12 months from the date of acquisition. Within this allowed period it is likely that the parent company will have to produce annual financial statements. Thus it may be necessary to determine (some) fair values on a provisional basis. The consequences are that in the following accounting period the provisional values may need to be restated. IFRS 3 requires the confirmed values to be recognised as from the date of acquisition. This would mean restating the goodwill and any asset or liability whose provisional estimate has been revised in the corresponding amounts of the financial statements following the year of acquisition. This would mean that the corresponding figures presented would not agree with the original presentation of those financial statements. Depreciation charges (and possible impairments of goodwill) may also need revision. It is important to note that any adjustment to provisional values must reflect conditions that existed at the date of acquisition. Fair value changes as a result of subsequent events are not part of this process. Another situation where corresponding amounts are restated is for the correction of errors (discovered subsequent to publication of the financial statements). The error may be one of recognition, measurement or presentation. Such errors must be either material or immaterial and deliberate (i.e. done intentionally to improve the appearance of the financial statements). The revision of an accounting estimate is not a prior period error. IAS 8 Accounting policies, changes in accounting estimates and errors requires prior period errors to be accounted for retrospectively. This is achieved by restating any affected comparative amounts for the prior periods presented or by restating opening balances of assets, liabilities and equity if the error occurred before the earliest prior period presented.

(ii)

(iii) Comparability is an important characteristic of financial statements. If a company changes an accounting policy this is likely to impair comparability because the current years financial statements (applying a new accounting policy) will have been prepared on a different basis to the corresponding amounts (using the previous policy). In order to minimise the effect of this IAS 8 requires changes in accounting policy to be applied retrospectively. This means that the financial statements presented (including corresponding amounts) should be presented as if the new accounting policy had always been in place. This will mean that the corresponding financial statements will be different to when they were originally published. (iv) All of the above examples can lead to a revision of the profit shown in the corresponding financial statements. This in turn would cause the corresponding eps figure to be revised. The example in the question says that even allowing for the revised profit, the eps does not compute correctly. The probable reason for this is there has been a bonus issue of shares (or an issue of shares containing a bonus element) in the current year (i.e. ended 31 March 2007 in Toogoods case). In order to preserve comparability of the trend shown by the eps figures, any previously reported eps presented must be adjusted for the dilutive effect of any bonus issues. Thus in Toogoods case the corresponding eps will have been recalculated based on the revised earnings and then adjusted for the effects of the bonus issue made in the year to 31 March 2007. Note: only three examples were required and other examples may be acceptable. (b) The trainee accountant is getting confused with valuation and the purpose of amortisation/depreciation. What the surveyor says in relation to the value of leasehold properties may be correct, but it does not remove the need to amortise properties with a life of more than 10 years. The purpose of amortisation (and depreciation) is to spread the cost of non-current assets over the period they give benefits. It is in essence a cost allocation process in compliance with the accruals/matching principle; depreciation is not a valuation model. Thus the suggestion by the trainee is unacceptable; each accounting period must bear a charge for amortisation of the leaseholds reflecting the proportion (measured in time) of the lease that has expired. If it is considered important that the balance sheet reflects the current value of leaseholds, then under IAS 16 Property, plant and equipment, Toogood may revalue its leasehold property (the gain going to equity) to fair (market) value. This course of action would still require amortisation to be charged, but it would now be on the revalued amounts. Ironically this would lead to higher charges for amortisation and thus reduce profit. The trainee accountants comment that non-amortisation of the leasehold would improve cash flows is misguided; decreasing (or increasing) amortisation has no effect on cash flows.

18

(c)

IAS 27 Consolidated and separate financial statements normally requires that when a parent acquires control of an entity, the entity must be consolidated from the date of acquisition/control. This is the treatment applied to Trilogy and one subsidiary. An acquired subsidiary cannot be excluded from consolidation simply because it is making losses. In this case it seems likely that Toogood intends to sell the non-consolidated subsidiary (perhaps because it is making losses). This means that Toogoods control of the subsidiary will be temporary. In these circumstances IAS 27 used to allow an exemption from consolidation (such that they were treated as available for sale investments under IAS 39 Financial instruments: recognition and measurement) and it seems that Toogood has applied this exemption. However IFRS 5 Non-current assets held for sale and discontinued operations has removed this exemption from IAS 27. IFRS 5 requires that a subsidiary acquired with a view to sale is classified as a disposal group (of assets and liabilities). The effect of the new treatment is not very different from that required by IAS 27. The assets of the subsidiary should be shown as held for sale within current assets and the liabilities of the subsidiary should be shown as held for sale within current liabilities (under IAS 27 they appeared net). The other notable difference is that under IFRS 5 the non-current assets of a disposal group are not depreciated, instead the subsidiarys assets should be measured at the lower of cost and fair value less cost to sell.

Note IFRS 5 uses the term discontinued operation. The answer below also uses this term, but it should be realised that the assets of the discontinued operation are classed as held for sale and not yet sold. In some literature this may be described as a discontinuing operation. Profitability/utilisation of assets An important feature of the companys performance in the year to 31 March 2007 is to evaluate the effect of the discontinued operation. When using an entitys recent results as a basis for assessing how the entity may perform in the future, emphasis should be placed on the results from continuing operations as it is these that will form the basis of future results. For this reason most of the ratios calculated in the appendix are based on the results from continuing operations and ratio calculations involving net assets/capital employed generally exclude the value of the assets held for sale. On this basis, it can be seen that the overall efficiency of Greenwood (measured by its ROCE) has declined considerably from 335% to 297% (a fall of 113%). The fall in the asset turnover (from 189 to 167 times) appears to be mostly responsible for the overall decline in efficiency. In effect the companys assets are generating less sales per $ invested in them. The other contributing factors to overall profitability are the companys profit margins. Greenwood has achieved an impressive increase in headline sales revenues of nearly 30% (63m on 212m) whilst being able to maintain its gross profit margin at around 29% (no significant change from 2006). This has led to a substantial increase in gross profit, but this has been eroded by an increase in operating expenses. As a percentage of sales, operating expenses were 105% in 2007 compared to 116% in 2006 (they appear to be more of a variable than a fixed cost). This has led to a modest improvement in the profit before interest and tax margin which has partially offset the deteriorating asset utilisation. The decision to sell the activities which are classified as a discontinued operation is likely to improve the overall profitability of the company. In the year ended 31 March 2006 the discontinued operation made a modest pre tax profit of $450,000 (this would represent a return of around 7% on the activitys assets of $63 million).This poor return acted to reduce the companys overall profitability (the continuing operations yielded a return of 335%). The performance of the discontinued operation continued to deteriorate in the year ended 31 March 2007 making a pre tax operating loss of $14 million which creates a negative return on the relevant assets. Despite incurring losses on the measurement to fair value of the discontinued operations assets, it seems the decision will benefit the company in the future as the discontinued operation showed no sign of recovery. Liquidity and solvency Superficially the current ratio of 211 in 2007 seems reasonable, but the improvement from the alarming current ratio in 2006 of 097 is more illusory than real. The ratio in the year ended 31 March 2007 has been distorted (improved) by the inclusion of assets of the discontinued operation under the heading of held for sale. These have been included at fair value less cost to sell (being lower than their cost a requirement of IFRS 5). Thus the carrying amount should be a realistic expectation of the net sale proceeds, but it is not clear whether the sale will be cash (they may be exchanged for shares or other assets) or how Greenwood intends to use the disposal proceeds. What can be deduced is that without the assets held for sale being classified as current, the companys liquidity ratio would be much worse than at present (at below 1 for both years). Against an expected norm of 1, quick ratios (acid test) calculated on the normal basis of excluding inventory (and in this case the assets held for sale) show an alarming position; a poor figure of 062 in 2006 has further deteriorated in 2007 to 044. Without the proceeds from the sale of the discontinued operation (assuming they will be for cash) it is difficult to see how Greenwood would pay its creditors (and tax liability), given a year end overdraft of $1,150,000. Further analysis of the current ratios shows some interesting changes during the year. Despite its large overdraft Greenwood appears to be settling its trade payables quicker than in 2006. At 68 days in 2006 this was rather a long time and the reduction in credit period may be at the insistence of suppliers not a good sign. Perhaps to relieve liquidity pressure, the company appears to be pushing its customers to settle early. It may be that this has been achieved by the offer of early settlement discounts, if so the cost of this would have impacted on profit. Despite holding a higher amount of inventory at 31 March 2007 (than in 2006), the company has increased its inventory turnover; given that margins have been held, this reflects an improved performance. Gearing The additional borrowing of $3 million in loan notes (perhaps due to liquidity pressure) has resulted in an increase in gearing from 286% to 356% and a consequent increase in finance costs. Despite the increase in finance costs the borrowing is acting in the shareholders favour as the overall return on capital employed (at 297%) is well in excess of the 5% interest cost.

19

Summary Overall the companys performance has deteriorated in the year ended 31 March 2007. Managements action in respect of the discontinued operation is a welcome measure to try to halt the decline, but more needs to be done. The companys liquidity position is giving cause for serious concern and without the prospect of realising $6 million from the assets held for sale it would be difficult to envisage any easing of the companys liquidity pressures. Appendix ROCE: continuing operations (4,500 + 400)/(14,500 + 8,000 6,000) 2007 297% 2006 335%

(3,500 + 250)/(12,500 + 5,000 6,300)

The return has been taken as the profit before interest (on loan notes only) and tax from continuing operations. The capital employed is the normal equity plus loan capital (as at the year end), but less the value of the assets held for sale. This is because the assets held for sale have not contributed to the return from continuing operations. Gross profit percentage (8,000/27,500) Operating expense percentage of sales revenue (2,900/27,500) Profit before interest and tax margin (5,100/27,500) Asset turnover (27,500/16,500) Current ratio (9,500:4,500) Current ratio (excluding held for sale) (3,500:4,500) Quick ratio (excluding held for sale) (2,000:4,500) Inventory (closing) turnover (19,500/1,500) Receivables (in days) (2,000/27,500) x 365 Payables/cost of sales (in days) (2,400/19,500) x 365 Gearing (8,000/8,000 + 14,500) 291% 105% 185% 167 211 077 044 130 265 449 356% (6,200/21,200) (2,450/21,200) (3,750/21,200) (21,200/11,200) (3,700:3,800) not applicable (2,350:3,800) (15,000/1,350) (2,300/21,200) x 365 (2,800/15,000) x 365 (5,000/5,000 + 12,500) 292% 116% 177% 189 097 062 111 396 681 286%

(a)

Errsea income statement extracts year ended 31 March 2007 Loss on disposal of plant see note below ((90,000 60,000) 12,000) Depreciation for year (wkg (i)) Government grants (a credit item) see note below and (wkg (iv)) $ 18,000 75,000 (19,000)

Note: the repayment of government grant of $3,000 may instead have been included as an increase of the loss on disposal of the plant. Errsea balance sheet extracts as at 31 March 2007 cost $ 360,000 accumulated depreciation $ 195,000 carrying amount $ 165,000 39,000 27,000

Property, plant and equipment (wkg (v)) Non-current liabilities Government grants (wkg (iv)) Current liabilities Government grants (wkg (iv)) Workings (i)

Depreciation for year ended 31 March 2007 On acquired plant (wkg (ii)) Other plant (wkg (iii))

$ 52,500 22,500 75,000

(ii)

The cost of the acquired plant is recorded at $210,000 being its base cost plus the costs of modification and transport and installation. Annual depreciation over three years will be $70,000. Time apportioned for year ended 31 March 2007 by 9/12 = $52,500. $ 22,500

(iii) The other remaining plant is depreciated at 15% on cost (b/f 240,000 90,000 (disposed of) x 15%) (iv) Government grants Transferred to income for the year ended 31 March 2007: From current liability in 2006 (10,000 3,000 (repaid)) From acquired plant (see below):

$ 7,000 12,000 19,000

20

Non-current liability b/f transferred to current on acquired plant (see below)

$ 30,000 (11,000) 20,000 39,000

Grant on acquired plant is 25% of base cost only = $48,000 This will be treated as: To income in year ended 31 March 2007 (48,000/3 x 9/12) Classified as current liability (48,000/3) Classified as a non-current liability (balance)

12,000 16,000 20,000 48,000

Note: government grants are accounted for from the date they are receivable (i.e. when the qualifying conditions for the grant have been met). Current liability Transferred from non-current (per question) On acquired plant (see above) 11,000 16,000 27,000 cost $ Property, plant and equipment Balances b/f 240,000 Disposal (90,000 ) Addition (w (ii)) 210,000 Other plant depreciation for year (wkg (iii)) Balances at 31 March 2007 360,000 (b) (i) accumulated depreciation $ 180,000 (60,000 ) 52,500 22,500 195,000 carrying amount $ 60,000 (30,000 ) 157,500 (22,500) 165,000

(v)

This is an example of an adjusting event within IAS 10 Events after the balance sheet date. This means that an impairment of trade receivables of $23,000 must be recognised (and charged to income). The increase in the receivable after the year end should be written off in the following years financial statements. Sales of the year-end inventory in the following accounting period may provide evidence that the inventorys net realisable value has fallen below its cost. This appears to be the case for product W32 and is another example of an adjusting event. With a selling price of $540 and after paying a 15% commission, the net realisable value of W32 is $459 each. Assuming that the fall in selling price is not due to circumstances that occurred after the year end and that the selling price is typical of what the remainder of the product will sell for, inventory should be written down (via a charge to the income statement) by $16,920 ((600 459) x 12,000 units).

(ii)

(iii) Tentacle has correctly treated the outstanding litigation as a contingent liability. The settlement of a court case after the balance sheet date may confirm (or otherwise) the existence of an obligation at the year end and would be an example of an adjusting event. This would then require that either the disclosure note of the contingency is removed or the obligation should be provided for dependent on the outcome of the litigation. However, this is not quite the case in Tentacles example. The circumstances of the claim against Tentacle are different from those of the recently settled case. So this settlement does not appear to have any effect on the likelihood of Tentacle losing the case. What it does (potentially) affect is the estimated amount of the liability. IAS 10 refers to this situation as an updating disclosure. The only required change to the financial statements would be to update the disclosure note on the contingent liability to reflect that the potential liability has increased from $500,000 to $750,000. (iv) Normally the effect of price increases of materials after the balance sheet date would be a matter for the following years financial statements as such increases do not affect the costs as they existed at the balance sheet date (i.e. they would not be an adjusting event). However, Tentacles method of recognising profit (using a cost basis to determine the percentage of completion) requires an estimate (at 31 March 2007) of the future costs of the contract. This estimate directly determines the amount of profit recognised at 31 March 2007. Therefore the information indicating that the total estimated costs of the contract have increased should be taken as providing additional evidence of conditions that existed at the year end. Thus this is an adjusting event which requires the recognised profit to be recalculated. The original estimate of the recognised profit at 31 March 2007 of $12 million would be half of the estimated total profit of $24 million (percentage of completion is 50% i.e. $3 million/$6 million). The increase in the costs of $15 million means the revised estimated total profit is only $900,000 (24m 15m). The revised total costs are $75 million (6m + 15m). Thus the recognised profit on the contract should be recalculated as $360,000 (900,000 x 3m/75m) with appropriate amendments to the income statement and balance sheet figures.

21

Part 2 Examination Paper 2.5(INT) Financial Reporting (International Stream)

June 2007 Marking Scheme

This marking scheme is given as a guide in the context of the suggested answers. Scope is given to markers to award marks for alternative approaches to a question, including relevant comment, and where well-reasoned conclusions are provided. This is particularly the case for written answers where there may be more than one acceptable solution. Marks 2 7 1 1 1 1/ 2 1 1 4 3 1 1 1 1 1/ 2 26 25

non-current assets goodwill (1 for impairment) inventory trade receivables receivable re intellectual property bank equity shares share premium retained earnings minority interest 10% loan notes trade payables deferred consideration overdraft tax liability available Maximum for question

Income statement revenue cost of sales operating expenses rental income loss on investment property (in income statement) finance costs income tax Changes in equity balances b/f equity option bonus issue revaluation profit for period dividends Balance sheet property, plant and equipment investment property current assets (re inventory) deferred tax 8% loan note loan from Westwood current liabilities available Maximum for question

1 3 1 1 1 3 2 1 1 1 1 1 1 2 1 1 1 2 2 1 28 25

(a) (b) (c)

up to 4 marks for each example 1 mark per point to a 1 mark per point to a

maximum maximum maximum Maximum for question

12 7 6 25

23

up to 10 marks for relevant ratios up to 5 marks for effect of discontinued operation up to 1 mark per relevant interpretive comment Maximum for question

Marks 10 5 10 25

(a)

loss on disposal depreciation for year government grant to income ($19,000) property, plant and equipment current liability non-current liability available maximum

1 2 2 3 2 2 12 10 1 2 3 1 1 2 4 1 2 1 1 5 4 1 1 1 2 5 4 25

(b)

(i)

adjusting event impairment of $23,000 maximum adjusting event NRV is $459 impairment loss $16,920 maximum

(ii)

(iii) settlement of court case normally adjusting event this case does not alter nature of contingency example of updating disclosure revise amount of contingency disclosure to $750,000 available maximum (iv) after date increase not normally adjusting event as future costs are part of profit calculation this is adjusting event calculation of new estimate of total profit ($900,000) recalculation of profit for year ($360,000) available maximum Maximum for question

24

You might also like